You are on page 1of 87

Cambridge International

A and AS level Mathematics


Statistics Practice book
The University of Cambridge Local Examinations Syndicate bears no responsibility for the example answers to
questions taken from its past question papers which are contained in this publication.

5 (i)
S1 Statistics No. people (x)
1
Frequency (f)
42
xf
42

S1 Statistics 1 Answers
1 Answers 2
3
15
6
30
18
4 7 28
1 Exploring data 5 1 5

Exercise 1.1 6 0 0

5 + 3 + 8 + 1 + 12 + 3 + 2 + 3 + 7 7 4 28
1 Mean =
44
9
∑ f = 75 ∑ xf = 151
= 
= 4.8 or 4.89 (3 s.f.)
9
(ii) Mean =
∑ xf =
151
= 2.01 (3 s.f.)
1  2  3  3  3  5  7  8  12 ∑f 75

Median is 3 Median = 1

Mode is 3 Mode = 1

Median is good as it is not influenced by 6 No. siblings (x) Frequency (f) xf


outlier of 12. Arguments for all three! 0 7 0×7=0
90 1 18 1 × 18 = 18
2 Mean = = 7.5
12
2 12 2 × 12 = 24
5 6 6 7 7 7 8 8 8 8 9 11
3 9 3 × 9 = 27
Median is 7.5 4 4 4 × 4 = 16

Mode is 8 ∑ f = 50 ∑ xf = 85
3 (i) Discrete ∑ xf 85
x= = = 1.7
(ii) Mean =
164
= 10.93 = 10.9 (3 s.f.) ∑f 50
15
50 + 1  51
3  4  5  5  6  6  6  7  8  9  10  11  12  18  54 The median is the  = = 25.5th value
 2   2

Median = 7
Hence the median = 1 + 2 = 1.5
2
Mode = 6
The mode is 1 (it occurs 18 times).
(iii) 
Median – it is not influenced by the
7 (i) Test
outlier (54)
score Frequency, f midpoint, x xf
4 (i) Discrete 0–4 7 2 14

(ii) Continuous 5–9 18 7 126

10–14 14 12 168
(iii) Discrete
15–20 11 17.5 192.5
(iv) Category
∑ f = 50 ∑ xf = 500.5
(v) Discrete

(vi) Continuous (ii) Mean =


∑ xf =
500.5
= 10.01
∑f 50

(iii) 5–9 class


1
∑ xB ∑x
S1 8 6.50 =
18
⇒ ∑ x = 117
B
14 37.8 =
30
A
⇒ ∑ x A = 1134

8=
∑ xG
⇒ ∑ x = 96 24.2 =
∑ xB ⇒ ∑ x B = 605
12 G 25
x=
117 + 96 213
= = $7.10 19.5 =
∑ xC ⇒ ∑ x C = 312
30 30 16
1134 + 605 + 312 2051
9 Length Midpoint Frequency x= = = 28.9 (3 s.f.)
71 71
0x2 1 9
2x4 3 21
Exercise 1.2
S1 Statistics 1 Answers

4x8 6 f
1 (i) Range = 18 – 2 = 16
  8  x  10 9 6
IQR = 13 – 2.5 = 10.5
1× 9 + 3 × 21+ 6 × f + 9 × 6
= 4.125
9 + 21+ f + 6
2 + 3 + 5 + 8 + 18
126 + 6f
= 4.1
125 x= = 7.2
5
36 + f
126 + 6f = 4.125(36 + f ) 22 + 32 + 52 + 82 + 182
s= − 7.22
126 + 6f = 148.5 + 4.125f 5
1.875f
  1 = 22.5 = 5.78 (3 s.f.)
           f = 12
(ii) Range = 140 – 21 = 119
so the total number of earthworms is 48.
IQR = 131 – 36 = 95
∑x ⇒
10 86.5 =
18
∑ x = 1557 x = 21 + 25 + 36 + ... + 140 = 92.8
10
86 =
∑x ⇒ ∑ x = 1634
19 212 + 252 + 362 + ... + 1402
s= − 92.82
10
New player’s weight = 1634 – 1557 = 77 kg
= 45.4 (3 s.f.)

11 191 =
∑x 1
⇒ ∑ x 1 = 955
5 2 Range = 21 − 0 = 21
193 =
∑ x2 ⇒ ∑ x 2 = 965
5 Arranging in order,
The new player is 10 cm taller than the substituted 0  2  2  5  6  7  9  21
player so 198 cm tall.
LQ Median UQ
310
12 x 1 = = 12.4  min
25
496 IQR = UQ − LQ = 8 − 2 = 6
x2 = = 12.4  min ⇒ n = 40
n
s=
∑x2 − x 2

n
13 Time, x Frequency, f xf
02 + 22 + 22 + 52 + 62 + 72 + 92 + 212
2 28 56 = − 6.52
8
2.5 22 55
640
= − 6.52
3 n 3n 8
Total 50 + n 111 + 3n = 37.75


x=
∑ xf ⇒ 2.4 =
111 + 3n = 6.14 (3 s.f.)

∑f 50 + n
3 h = ∑ =
h 9150
111 + 3n = 152.5 cm
    2.4 = n 60
50 + n
∑ (h − h )
2

   2.4 (50 + n) = 111 + 3n sh = =


9077.4
= 12.3 cm
n 60
   120 + 2.4n = 111 + 3n
   9 = 0.6n
     n = 15

2
4 (i) No. Frequency ∑ y = −240
faults (x) (f) xf x 2f
9 (i) Let y = x − 50 so
S1
0 22 0 0

y=
∑y =
−240
= −2.4
n 100
1 34 34 34 ⇒ x = −2.4 + 50 = 47.6
2 25 50 100 (ii) Since sd(y) = sd(x)
3 12 36 108

sy =
∑y 2

− y2 ⇒ 3.5 =
∑y 2

− (−2.4)2
4 7 28 112 n 100
∑ f = 100 ∑ xf = 148 ∑ x f = 354 2
⇒ ∑ y = ( 3.5 + (−2.4) ) × 100 = 1801
2 2 2

∑ ( x − 50) = 1801
2

148

S1 Statistics 1 Answers
(ii) Mean = = 1.48
100 ∑x = 2 ⇒
10 x =
4
∑x = 8
354
(iii) s = − 1.482 = 1.16 (3 s.f.)
100
x new =
∑x new
= 3 ⇒ ∑ x new = 15
(iv) Range = 4 − 0 = 4 5
so the num
mber added is 7
IQR = 2 − 1 = 1
s=
∑x 2

− 22 = 4
5 (i) 4

 
∑x 2

− 22 = 16
Time (min) Frequency, f x xf x 2f 4
0x6 6 3 18 54   ∑x 2
= 80
 ∑x 2
= 80 + 7 = 1292
 6  x  12 8 9 72 648 new

129 2
12  x  20 24 16 384 6144   snew = − 3 = 4.10 (3 s..f.)
5
20  x  30 7 25 175 4375
11 (i) Since the standard deviation is 0, all the
30  x  40 3 35 105 3675
activities Lara did must cost the same, which is

∑ f = 48 ∑ xf = 754 ∑x f 2
the mean of $12.
= 14 896 6 × 12 + 4 x
(ii) xs = = 11
10
754 72 + 4 x = 110 ⇒ 4x = 38 ⇒ x = $9.50
(ii) Mean = x = = 15.7  min (3 s.f.)
48
∑x f − x
2

14 896 s= 2

(iii) s =
48
− 15.72 = 7.97  min (3 s.f.) ∑f
122 × 4 + 122 × 2 + 9.52 × 4
s= − 112
6 (i) 10
Score f x xf x 2f
1225
0–4 7 2 14 28 = − 112 = 1.22 (3 s.f.)
10
5–9 18 7 126 882
10–14 14 12 168 2016 12 Standard
15–20 11 17.5 192.5 3368.75 Mean deviation Years
∑ f =  50 ∑ xf =  500.5 ∑ x f 2
City A 195 14 10
= 6294.75
City B 143 10 20
500.5
(ii) Mean ≈ = 10.01 1950 + 2860
50 Mean (A and B) = = 160.3
30
6294.75
s= − 10.012 = 5.07 (3 s.f.) ∑A 2
∑A 2
50 sA = − A2 ⇒ 14 = − 1952
n 10

7 Mean = x = 1170 = 32.5 min ⇒ ∑ A = (142 2


+ 1952 )  × 10 = 382 210
36
38 370 sB =
∑B − B 2
2
⇒ 10 =
∑B 2

− 1432
s= − 32.52 = 3.10 min (3 s.f.) n 20
36
⇒ ∑B 2
= (102 + 1432 ) × 20 = 410 980
8 Let Y = T − 100
s A+ B =
∑ A + ∑B 2 2

− A+B
2

∑ Y = 24 and ∑ Y 2 = 330 n
382210 + 410 980
24 = − 160.3 2
Y= = 2 ⇒ T = 100 + 2 = 102° 30
12 = 27.1 (3 s.f.)

330
sY = − 22 = 4.85° (3 ss.f.) ⇒ sT = 4.85°
12 3
13 Let y = x − 50 2 (i) Team A Team B
S1 (i) y=
496
= 6.2 so x = 6.2 + 50 = 56.2 km/h LQ 82.5 62
80
Median 89 73
(ii) Let z = x − 40 UQ 97.5 80

z = x − 40 = 16.2
∑ z = 16.2 ⇒ (ii)
z =
80
∑ z = 1296 Weights of players
so  ∑ ( x − 40) = 1296

Team B
S1 Statistics 1 Answers

(iii) s x = s y = s z = 6.7

s z = 6.7 =
∑z 2

−z2
n
Team A

6.7 =
∑z 2

− 16.2
2

80

6.72 =
∑z 2

− 16.22
40 50 60 70 80
weight (kg)
90 100 110 120

80
∑ z 2 = 80 (6.72 + 16.22 ) = 24586.4 (iii) •  The median weight for Team A is higher
than Team B. In general, the weight of the
∑ ( x − 40)
2
= 24586.4
people in Team A is greater.

14 (i) y=
∑ y = −189 = −5.4 min •  T
 he range of weights in Team B is greater
n 35
than Team A.
x = −5.4 + 120 = 114.6 min
•  The IQR is broadly similar for both teams.
(ii) sx = sy = 12.8
3

12.8 =
∑y 2

− (−5.4)2
Cookie frequency
30
35
∑ y2 = 35 × (12.82 + 5.42 ) = 6755 25
∑ (x − 120)
2
= 6755

20
frequency

Exercise 1.3
15
1 (i) 16 0 3 4
15
14 4 10
13 0 4 5 8
12 2 7
5
11 2 6 9
10 9
9 5 0
12 13 14 15 16
(ii) Median = 130 number of cookies

UQ = 144 No. cookies (x) Frequency (f) Angle


4
LQ = 116 12 4 × 360 = 20°
72
(iii) 29
13 29 × 360 = 145°
length of DVDs 72
21
14 21 × 360 = 105°
72
10
15 10 × 360 = 50°
72
8
90 100 110 120 130 140 150 160 170 16 8 × 360 = 40°
72
time (min)

4
Cookie frequency 5 A side-by-side bar chart would work well here. Two
12
pie charts, one for boys and one for girls, would also S1
16 6% work. There are many ways to represent this data.
11%
Students’ opinions
15
14% 13 24
40% 22
20
14
18
29%
16 boys

frequency
14 girls
12
4 ·  The scale on the vertical axis does not start at zero. 10

S1 Statistics 1 Answers
8
·  The axes are not labelled and the 3-D perspective 6
makes the ‘Yes’ category look larger. 4
2
·  Line graph is not appropriate and the scale on the 0
yes no undecided
horizontal axis is not uniform.
Boys’ opinions Girls’ opinions
·  Using 3-D objects is not appropriate: making the
un-
width and the height twice as large makes the 2009
decided
barrel appear much more than twice the ‘size’. (49°) un-
decided
(103°) yes
·  The scale on the vertical axis is not uniform. no yes
(196°) (231°)
(115°)
no
(26°)

Mice weights
6 (i)

150 170 190 210 230 250 270 290 310 330 350
weight (grams)

(ii) Minimum 120 g, range of 180 g. Maximum is 300 g.

LQ = 210 g, IQR = 60 g UQ = 270 g


Mice weight

100 150 200 250 300 350


weight (grams)

(iii) ·  The median of the second sample of mice is substantially lower than the first.

·  The range of weights for the second sample is larger than the first.

·  The interquartile range of the second sample is much smaller than the first.

·  In general, the weights of the first sample were higher than the weights
of the second sample.

·  The weights in the first sample are skewed towards the higher values.

·  The weights in the second sample are skewed towards the lower values.

5
Stretch and challenge
4 x = ∑ ⇒ 15 = ∑ ⇒ ∑ x = 60
x x
S1 n n n 4
∑ ( xi − x ) 2
∑ xi 2
The lowest possible score will occur when the other
1 i =1
= i =1
− x2
n n
three students all score the maximum mark of 20,
LHS :
n giving a total of 60 marks. The lowest possible mark
∑(x i − x )2 is therefore 0.
i =1
n
n 5 It isn’t possible if you take the average to be the
∑( x i
2
− 2xi x + x 2 ) mean. But if you use the median it is quite possible.
= i =1

n The diagram gives an example with seven boys and


S1 Statistics 1 Answers

n n n

∑ xi 2 − 2x ∑ xi + ∑ x 2 seven girls. Each light grey dot represents a male


= i =1 i =1 i =1
and each dark grey dot represents a female. A line
n
n
connecting two dots indicates that they dated. The
∑ xi 2 − 2x × nx + nx 2
= i =1 boy’s total is fourteen, for a mean of two. Obviously
n
n the girls must have the same total (since every line
∑ xi 2 − 2nx 2 + nx 2 has two ends) so the girl’s mean is also two. But
= i =1

n the median is a different story. Sorting the boy’s


n

∑ xi 2
− nx 2
data gives us 0 1 1 3 3 3 3 for a median of 3 dating
= i =1
partners. After the girl’s data is sorted we have 1 1 1
n
n
1 2 3 5 for a median of 1 dating partner. The medians
∑ xi 2
= i =1
−x 2 do not necessarily have to be equal.
n

2 (i) w and y

(ii) z

3 x =
∑ x ⇒ 124 = ∑ x and 125 = ∑ x + 159
n n n +1

124n = ∑ x so 125 =
124n + 159 ∑x ⇒ 8 = ∑x ⇒
n+1
6 x =
n 11
∑ x = 88
125(n + 1) = 124n + 159
125n + 125 = 124n + 159 s=
∑x 2

− x2 ⇒ 2=
∑x 2

− 82 ⇒ ∑ x 2 = 748
n 11
n = 34
∑ x = 4216 If we remove one 8, ∑ x = 88 − 8 = 80

126 =
∑x 1
⇒ ∑ x1 = 4536
x=
80
=8
10
36
Score needed = 4536 − ( 4216 + 159) = 161 ∑x 2
= 748 − 82 = 684
684
s= − 82 = 2.10 (3 s.ff.)
10

6
Exam focus 5 Let the boys’ marks be b and the girls’ marks be g.

∑ xf ∑b ∑b S1
1 (i) x= =
410
= 6.83 (3 s.f.) b=
n
⇒ 28 =
12
⇒ ∑ b = 28 × 12 = 336
∑f 60
∑g ∑g
(ii) The modal class is 4  x  6.
g=
n
⇒ 30 =
18
⇒ ∑ g = 30 × 18 = 540
∑ b + ∑ g = 336 + 540 = 29.2
(iii) s =
∑x f − x2 =
2
3682
− 6.832 = 3.83 (3 s.f.)
b+g =
n 30
∑f 60
∑ b2
s = − b2
2 Let y = x − 70 b n

∑ y ⇒ 4 = 108 ⇒ n = 108 = 27 ⇒ 3.1 =


∑b 2

− 282
(i) y= 12
n n 4
⇒ ∑ b = (3.12 + 282 ) × 12 = 9523.32

S1 Statistics 1 Answers
2

∑y 2
3025
(ii) s y =
n
− y2 =
27
− 42 = 9.80 (3 s.f.)
sg =
∑ g − g2 2

n
Let z = x − 74
⇒ 2.5 =
∑g 2

− 302
Since s x = s y = sz 18
⇒ ∑g 2
= (2.52 + 302 ) × 18 = 16 312.5

sz =
∑z 2

− z 2 ⇒ 9.80 =
∑z 2

− 02 ⇒ ∑ z 2 = 2593 ∑b + ∑g 2 2
2
s = −b+g
n 27 b+g n
∑ ( x − 74)
2
= 2593 9523.32 + 16 312.5
   = − 29.22
30
   = 2.92 (3 s.f.)
3 x = ∑ =
x 4308
= $215.40
n 20
6 (i) Let y = x − k
s=
∑x 2

− x2 =
948 551
− 215.4
402 = $32.10
n 20

y=
∑y ⇒y=
252
= 5.6
n 45
4 Let y = x – 340. Then ∑ y = −200 and ∑ y 2
= 962
∑y 2
1895
y=
∑ y = −200 = −4 ml so sx = sy =
n
− y2 =
45
− 5.62 = 3.28 (3 s.f.)
n 50
x = −4 + 340 = 336 ml (ii) 78 − k = 5.6 so k = 72.4

sy =
∑y 2

− y2 =
962
− (−4)2 = 1.8 ml
n 50
Since the y values have the same spread as the x
values, the standard deviation (and variance) of both
sets of numbers is the same.

So s x = s y = 1.8 ml

7 (i) Median = 33.5, LQ = 25, UQ = 43

(ii)
Class A

Class B

0 10 20 30 40 50
score

7
S1 2 Representing and 2 (i)

interpreting data Score f Boundaries Width f.d. c.f.

Exercise 2.1 0–4 7  − 0.5  x  4.5 5 1.4 7

1 (i) 5–9 18 4.5 < x  9.5 5 3.6 25

Frequency Cumulative 10–14 14 9.5 < x  14.5 5 2.8 39


Length (x  cm) Frequency, f density frequency
15–20 11 14.5 < x  20.5 6 1.83 50
0<x2 2 1 2
S1 Statistics 1 Answers

15 7.5 17 Histogram of test scores


2<x4 (ii)
4
4<x8 22 5.5 39

frequency density
3
8 < x < 10 7 3.5 46

∑f = 46 2

(ii) 1

Histogram of earthworm lengths


8
0 5 10 15 20
7 test score

6
frequency density

(iii) Cumulative frequency of test scores


5 50
4 cumulative frequency 40
3
30
2
20
1
10
0 2 4 6 8 10
length (cm) 0
5 10 15 20
test score
(iii)
Median ≈ 9.5  LQ ≈ 6.2  UQ ≈ 13.8
Cumulative frequency of Earthworm lengths
50 (iv) 19
cumulative frequency

40
(v) 20 students
30

20 3 (i)
Frequency Cumulative
10
Time (min) Frequency, f density frequency
0 0<x6 6 1 6
1 2 3 4 5 6 7 8 9 10
length (cm)
6 < x  12 8 1.33 14
(iv) LQ = 3.3 12 < x  20 24 3 38
Median = 5.1 20 < x  30 7 0.7 45

UQ = 7.1 30 < x < 40 3 0.3 48

(v) 46 − 9 = 37 ∑f = 48

8
(ii) (iii)

3
Histogram of times on phone
50
Cumulative frequency of time on phone S1
frequency density

cumulative frequency
40
2

30

1
20

0 10
5 10 15 20 25 30 35 40
time (mins)

S1 Statistics 1 Answers
0 5 10 15 20 25 30 35 40
time (mins)

(iv) Median ≈ 15.2

(v) 48 −11 = 37 spend more than 10 minutes.

4 (i)  
Time
5  t < 20 20  t < 30 30  t < 35 35  t < 45 45  t < 60 60  t < 80
(t minutes)

Frequency 5 30 67 50 24 8

(ii) Mean =
∑ xf =
5 × 12.5 + 30 × 25 + 67 × 32.5 + 50 × 40 + 24 × 52.5 + 8 × 70
∑f 5 + 30 + 67 + 50 + 24 + 8
6810
= = 37.0  minutess (3 s.f.)
184

6
5 (i) 1991: there were 710 000 − 600 000 so 110 000 7 (i) =2
(a) Frequency density =
3
more births 2 1
(b) Frequency density = =
(ii) (a) 6 3
1 2
Height = 2 × = cm
3 3
Proportion
Proportion of mothers (ii) (0.5, 0) and (3.5, 6)
of mothers giving
Median Interquartile giving birth aged (iii) (a)
age range birth aged 35 or
Year (years) (years) below 25 above Number
Hours of of days Midpoint
1991 27.5 7.3 33% 9% sunshine (f) (m) fm fm2
2001 29–29.5 7.5–9.5 23%–26.3% 18% 1–3 6 2 12 24

(b) 
Older 4–6 9 5 45 225

7–9 4 8 32 256
  •  Median is greater in 2001
10–15 2 12.5 25 312.5
  •  % of older mothers is greater in 2001
21 114 817.5
  •  % of younger mothers is less in 2001
114
6 Frequency Cumulative Mean = = 5.43 hours
21
Runs Frequency density frequency
817.5
0–4 8 1.6 8 SD = − 5.432 = 3.08 hours
21

5–25 21 1 29

26–49 48 2 77

50–99 20 0.4 97

100–120 3 0.14 100

100
9
(b)
• The values are measured to the Histogram of CO level
S1 nearest hour.
12

• We don’t know the exact values, we 10

frequency density
are only using the midpoint of each
8
interval.
6
8 (i) Median = 4.07°
4
IQR = 4.3 − 3.8 = 0.5°
2
(ii)
S1 Statistics 1 Answers

0
Temperature 3.0 < t 3.4  t 3.8  t 4.2  t 4.6  t 2 4 6 8 10 12 14 16 18 20
(t degrees < 3.4 < 3.8 < 4.2 < 4.6 < 5.0 CO level (ppm)
Celsius)
(ii) (a) 
25 125 243 157 50
Frequency Cumulative
CO level Frequency frequency
(iii) Mean
0<x2 2 2
25 × 3.2 + 125 × 3.6 + 243 × 4.0 + 157 × 4.4 + 50 × 4.8
≈ 2<x4 12 14
600
2432.8
= 4<x6 20 34
600
= 4.05 6 < x  10 14 48
The number of readings
(iv) Mean = 1.8 × 4.05 + 32 = 39.3 F 10 < x  14 8 56 up to and including the
6 < x  10 class is
Standard deviation = 1.8 × 0.379 = 0.6822 F 14 < x < 18 4 60 2 + 2 + 20 + 14 = 48

Stretch and challenge The last value should be the


same as the total frequency
1 (1) D
CO levels
(2) A 60
cumulative frequency

(3) B 50

40
(4) E
30
(5) C
20

10

Exam focus 0
0 2 4 6 8 10 12 14 16 18
1 (i) Frequency LQ median UQ 90th percentile
CO level Frequency f density CO level (ppm)

2
0<x2 2 =1 (b) 
From the graph, the median ≈ 5.6, LQ ≈
2
4.1, UQ ≈ 9.
12
2<x4 12 =6
2 (c) 
90th percentile ≈ 13.2
20
4<x6 20 = 10 (d) 
Drawing a line from 5 ppm to the curve
2
and across to the vertical axis gives a value
14
6 < x  10 14 = 3.5 of about 23. So the number of days when
4
the reading is above 5 ppm is 60 − 23 = 37
8
10 < x  14 8 =2
4 The graph can be drawn with curved lines
4 also…
14 < x < 18 4 =1
4

10
CO levels Time Midpoint
60
(x min) f (m) fm fm2 S1
cumulative frequency

50
30 < x  40 5 35 175 6 125
40
40 < x  45 29 42.5 1232.5 52 381.25
30
45 < x  50 55 47.5 2612.5 124 093.75
20

10 50 < x  60 43 55 2365 130 075

0 132 6385 312 675


0 2 4 6 8 10 12 14 16 18
CO level (ppm) (ii) k = 49.2 min

S1 Statistics 1 Answers
6385
2 (iii) Mean = = 48.4 min
132
No. Frequency, Class Class Frequency 312 675
cars, c f boundaries width density SD = − 48.42 = 5.38 min
132
6
1–4 6 0.5 < c  4.5 4 = 1.5 4
4
10 Time Midpoint
5–9 10 4.5 < c  9.5 5 =2
5 (t hours) fd Frequency (m) fm fm2
25 0<x1 8 8 0.5 4 2
10–19 25 9.5 < c  19.5 10 = 2.5
10
1<x2 20 20 1.5 30 45
9
20–25 9 19.5 < c  25.5 6 = 1.5
6 2<x4 25 50 3 150 450

Histogram of car numbers 4<x6 12 24 5 120 600


3
6 < x  10 4.5 18 8 144 1152
2.5 120 448 2249
frequency density

2 448
(i) Mean = = 3.73 hours
120
1.5 2249
(ii) SD = − 3.73...2 = 2.19 hours
120
1

0.5

0 5 10 15 20 25 30
number of cars

3 (i)
140
130
120
110
100
cumulative frequency

90
80
70
60
50
40
30
20
10
0
30 35 40 45 50 55 60
time (x min)
11
3 Probability Since P(S and T)  0
S1 S and T are not mutually exclusive.
Exercise 3.1
13 1 4 (i) 0.4  0.6 = 0.24
1 (i) =
52 4
4 1
(ii) 0.6 × (1 - 0.4) = 0.36
(ii) =
52 13
(iii) P(D or C) = P(D) + P(C) - P(D and C)
1
(iii)
52 = 0.4 + 0.6 - 0.24
16 4
(iv) = = 0.76
S1 Statistics 1 Answers

52 13
8 2 (iv) No they are not mutually exclusive as
(v) =
52 13
P(D and C)  0.
2 (i) 1 2 3 4 5 6 If two events are independent then
H H1 H2 H3 H4 H5 H6 P(D and C) = P(D) × P(C) so
T T1 T2 T3 T4 T5 T6
P(D and C)  0, so if the events are
1 independent they cannot be mutually
(ii)
12 exclusive.
(iii) 7
12 5 (i) P(A and B) = 0.32 × 0.5 = 0.16
3 1
(iv) = (ii) P(A or B) = P(A) + P(B) - P(A and B)
12 4
= 0.32 + 0.5 - 0.16
3 (i)
Die 1
= 0.66
M 1 2 3 4 5 6
6 (i) If F and G are independent then
1 1 2 3 4 5 6
P(F  G) = P(F) × P(G)
2 2 4 6 8 10 12
0.1 = 0.25 × P(G)
3 3 6 9 12 15 18
Die 2

0.1
P(G) = = 0.4
4 4 8 12 16 20 24 0.25
(ii) P(F  G) = P(F) + P(G) - P(F  G)
5 5 10 15 20 25 30
 = 0.25 + 0.4 - 0.1
6 6 12 18 24 30 36
 = 0.55
14 7
(ii) =
36 18 7 (i) 1, 1, 2;  1, 2, 1;  2, 1, 1
19
(iii) (ii) Total no. outcomes = 6 × 6 × 6 = 216
36
3 1
11 so P(total of 4) = =
(iv) 216 72
36
26 13
(iii) No. ways of getting 6:
(v) =
36 18
114, 141, 411, 123, 132, 213, 231, 312, 321
P(S or T) = P(S) + P(T) - P(S and T) 9 1
so P(total of 6) = 216 = 24
1 19 11
= + −
2 36 36
8 (i)
13 − 1 2 3 4
=
18 1 0 1 2 3
(vi) If independent then 2 1 0 1 2
P(S and T) = P(S) × P(T) 3 2 1 0 1
11 1 19 4 3 2 1 0
but ≠ ×
36 2 36
so S and T are not independent.

12
10 5 Alternately, let the probability of Cody winning be p.
(ii) P(difference less than 2) = =
16 8
The probability she wins on her first turn is
1
.
S1
6 3 18
(iii) P(difference at least 2) = =
16 8
The probability that no one wins on the first
3
No. games won = 8 × 40 = 15 1 2 5
× × =
5
round is
1 1 2 3 6 18
9 P(A) = 6 and P(B) = 6
The game then starts again so
If independent then 1 5
p = + p
18 18
P(A and B) = P(A) × P(B) 13 1
p=
1 1 1 18 18
= ×

S1 Statistics 1 Answers
36 6 6 1
p=
so A and B are independent. 13

Since P(A and B)  0 the events are not 12 (i) P(HH) = 0.04 so P(H) = 0.2, P(T) = 0.8
mutually exclusive.
P(TT) = 0.82 = 0.64
10 P(Bryan wins)
(ii) P(HT or TH)
=  ×  +  × × ×  +  × × × × × 
5 1 5 5 5 1 5 5 5 5 5 1
= p(1 − p) + (1 − p) p = 2p(1 − p)
6 6 6 6 6 6 6 6 6 6 6 6
5 5
3 5
5 2p(1 − p) = 0.42
= 62 + 64 + 66 + ...
2p − 2p2 = 0.42
5
This is a geometric series with a =   and 0 = 2p2 − 2p + 0.42
36
5
3
5 5
2
25 p = 0.3 orr 0.7
r= 4 ÷ 2 = 2 =

6 6 6 36
13 (i) The first packet bought obviously contains
5
a 5 a new card. The probability that the second
S∞ = = 36 =
1 − r 1 − 25 11 packet bought contains a different card is 2 so
36 3
it takes, on average, 3 packets to get the next
2
11 Probability that Cody wins on her first turn card. Similarly, after Olivia has the first two
1 2 1 1 cards, the probability that any randomly chosen
= × × =
2 3 6 18 packet has the last card is 1 so on average she
3
Probability that she wins on her second turn, would need to buy 3 packets.
1 2 5 1 2 1 1 5 5 3 1
= × × × × × = × = The total number of packets is 1 + +3=5
2 3 6 2 3 6 18 18 324 2 2
Continuing the pattern we get (ii)  U
 sing the same logic the number of packets
 1 needed on average to collect all 6 cards is
5 
2
1  1
×  +  ×    + ...
5
= +
18  18 18   18  18   6 6 6 6 6
1 + + + + + = 14.7
5 4 3 2 1
1  
2
1 + +   + ...
5 5
=
18  18  18   (iii) Looking at the expression above
6 6 6 6 6
  1+ + + + + can be written as
1  1  5 4 3 2 1
=
18  1 − 5 
  6  1 + 1 + 1 + 1 + 1 + 1
18  6 5 4 3 2 
1 18 So in general the number of packets needed for
= ×
18 13
a full set of n cards is
1
=
13
n  1 + 1 + 1 + 1 + ... 1 
 2 3 4 n

13
Exercise 3.2 (ii) No. days worked = 45 × 5 = 225
S1 1 (i) 3 W
No. days late = 0.24 × 225 = 54
6
5 (i) 0.15
Late
W
4 C'
7 3 0.98
6 B
0.85 On-time

0.02
S1 Statistics 1 Answers

4 W C
3 6
7
(ii) 0.98 × 0.85 = 0.833
B
(iii) 0.02 + 0.98 × 0.15 = 0.167
2
6 B or 1 - 0.833 = 0.167

4 3 2
(ii) (a)  7 × 6 = 7 Exercise 3.3
4 3 3 4 4
(b)  × + × = 1 (i) P(A and B) = 0.4 × 0.8 = 0.32
7 6 7 6 7

2 (i) B 0.63 (ii) P(A | B) = P(A and B ) = 0.32 = 0.4


0.7 P(B ) 0.8
P(B and A ) 0.32
M (iii) P(B | A) = = = 0.8
P(A ) 0.4
0.9
0.3 or, since A and B are independent
B' 0.27
P(A | B) = P(A) = 0.4
B 0.09
0.9 P(B | A) = P(B) = 0.8
0.1
M' 2 (i) C D

0.1
B' 0.01

0.6 – p p 0.3
(ii) 0.63 + 0.09 = 0.72

3 (i) 0.8 Won


0.1
In
0.6
0.2 Lost 0.5 Won P(D) = 0.3 + p
In P( D ∩ C ) P
0.9 (ii)   (a)  P(D  C ) = = = 0.4 ⇒ p = 0.24
0.4 P(C ) 0.6
Out 0.5 Lost
1 P(C ∩ D ) p 1
0.1 Out Lost   (b)  P(C D) = = =
P( D ) p + 0.3 4
4 p = p + 0.3
(ii) P( Win) = 0.6 × 0.8 + 0.4 × 0.9 × 0.5 3p = 0.3
 = 0.66 p = 0.1

4 (i) 0.8 On-time   (c) C and D mutually exclusive ⇒ P(C ∩ D) = 0


p=0
Bike
0.4 0.2 Late   (d)  C and D independent
0.6 On-time   ⇒ P(C ∩ D) = P(C ) × P(D)
0.4
Bus    p = 0.6 × ( p + 0.3)
p = 0.6 p + 0.18
0.4 Late
On-time
0.4 p = 0.18
0.2 1
p = 0.45
Car
0 Late

14 P(late) = 0.4 × 0.2 + 0.4 × 0.4 = 0.24


8 (i) 0.95 Positive P(VP) = 0.00475
3 (i) P(M | B) = P(M and B )
P(B )
V
S1
0.63 0.005
=
0.72 0.05 Negative P(VN ) = 0.00025
7
= 0.875 or
  8 0.02 Positive P(V'P) = 0.0199
0.995
V'
(ii) If M and B are independent then
0.98 Negative P(V'N) = 0.9751
P(M and B) = P(M) × P(B)

but 0.63  0.9 × 0.72 (ii) P(incorrect)

S1 Statistics 1 Answers
so M and B are not independent. = 0.005 × 0.05 + 0.995 × 0.02

or = 0.02015
P(V and P )
Since P(M|B)  P(M) the events are not (iii) P(V | P) =
P(P )
independent. 0.00475
=
P(F and W ) 0.00475 + 0.0199
4 (i) P(F | W) =
P(W ) = 0.193 (3 s.f.)
0.6 × 0.8
=
0.66
= 0.727 (3 s.f.) 9 (i)
C M

(ii) If F and W are independent then

P(F and W) = P(F) × P(W) 10 24 38

but 0.48  0.6 × 0.66

so F and W are not independent. 28


P(B and L)
5 P(B | L) =
P(L)
0.4 × 0.4
(ii) (a) P(C′) = 0.66
=
0.24 24 12
(b)  P(M C ) = =
2 34 17
= 0.667 (3 s.f.) or
3 10 5
(c)  P(C M′) = =
6 (i) If A and B are mutually exclusive, 38 19

then P(B | A) = 0
10
Maths English Total
(ii) A and B are not independent, since
Boys 105 92 197
P(A and B)  P(A) × P(B) Girls 84 112 196

0  0.4 × 0.5 Total 189 204 393

(iii) P(A or B )′ = 0.1 197


(i) P(B) = = 0.501 (3 s.f.)
393
7 (i) P(B | A) = P(B) = 0.2 204
(ii) P(M′) = = 0.519 (3 s.f.)
393
(ii) P(A or B) = P(A) + P(B) - P(A and B)
105
(iii) P(B M) = = 0.556 (3 s.f.)
= 0.75 + 0.2 - 0.75 × 0.2 189
84
(iv) P(M B′ ) = = 0.429 (3 s.f.)
= 0.8 196

(iii) Since P(A and B)  0, A and B are not mutually


exclusive.

15
(v) If M and B are independent then (iv) P(One problem delayed)
S1 P(M and B) = P(M) × P(B)   P(One problem and delayed)
=
105 189 197 P(delayed)
but ≠ ×
393 393 393 0.329
=
so M and B are not independent. 0.388
= 0.848 (3 s.f.)
or
(v) P(Delayed no tech. problems)
Since P(B|M)  P(B) the events are
P(Delayed and no tech. problems)
not independent. =

P(no tech. problems)
S1 Statistics 1 Answers

11 3 0.8 × 0.15 + 0.8 × 0.85 × 0.1


Red =
7 0.8
4 Red = 0.235
7 (vi) 110 × 0.388 = 42.68
4 Black
7
  Approximately 43 flights.
2
Red
3 7
7 Black 14 60

5 Black
7 45

Ball drawn from A Ball drawn from B

Alina
30
4 3 3 2 18
(i) P(Red) = × + × =
7 7 7 7 49
15
P (Red A and Red B)
(
(ii) P Red A Red B =
P (Red B)
)
4 3 0 15 30 45 60
× Simone
= 7 7
18
49 Anywhere inside the shape shows the points
12 where the two friends will meet.
= 49
18 Calculating the areas:
49
12 2 A (meet)
= =
18 3
1
= 2× × 15 × 15 + 452 + 452 × 152 + 152
2
12 (i) P (CT ) = 0.05 + 0.15 + 0.28 + 0.1 = 0.58
= 1575
P ( S and CT )
(
P S CT = ) P (CT ) P(Meet) =
1575 7
= = 0.4375
602 16
0.05
=
0.58
15 P(W R ) = P(W ∩ R ) ⇒ 0.99 = 0.9
5 P( R ) P( R )
= ≈ 0.0862
58 10
⇒ P(R ) =
11
(ii) If the events were independent then
P(R ∩ W ) 0.9
P (V ∩ CT ) = P(V ) × P(CT ) P(R W ) = ⇒ 0.96 =
P(W ) P(W )
but 0.28 ≠ 0.35 × 0.58 15
⇒ P(W ) =
so the events are not independent. 16

10 15
13 (i) a = 0.8, b = 0.85, c = 0.9 P(W or R ) = + − 0.9 = 0.94659...
11 16
P(W or R )′ = 1 − 0.94659 = 0.0534 (3 s.f.)
(ii) P(not delayed) = 0.8 × 0.85 × 0.9 = 0.612

P(delayed) = 1 – 0.612 = 0.388 16 P(Win) = P(Home win) + P(Away win)


= 0.25 × 2p + 0.75 × p
(iii) P(Just one problem)
= 1.25 p
= (0.2 × 0.85 × 0.9) + (0.8 × 0.15 × 0.9) 1.25 p = 0.5 ⇒ p = 0.4
+ (0.8 × 0.85 × 0.1) 0.25 × 0.8
P(H W ) = = 0.4
= 0.329 0.5

16
Stretch and challenge (ii) P(win from 2 - 0)

1 P(at least 2 born same month) = P(W) + P(LW) + P(LLW)


S1
2
3 2 3 2 3
= 1 - P(none born in same month) =  +  ×  +  × 
 5  5  5  5  5
12 11 10 9 8 = 0.936
= 1− × × × ×
12 12 12 12 12
So Sahil should get $936 000
= 0.618 (3 s.f.)

2 P(2K  1K ) = P(2K ∩  1K )
P( 1K ) 6 Consider the case where P(A) = 0. Then
P(2K )
= P( A) × P(B ) × P(C ) = 0 and also
P( 1K )

S1 Statistics 1 Answers
4 × 3 P ( A ∩ B ∩ C ) = 0. Now consider that B and C are not
= 52 51
independent so that P(B ∩ C ) ≠ P(B ) × P(C ).
1− 48 × 47
52 51
So the three events are not independent but the
1
= ≈ 0.0303 equality holds.
33

7 Yes it is true.
3 (i) P(Cam wins)
P( A′ ∩ B ′ ) = 1 − P( A ∪ B )
= P(TTH) + P(TTTTTH) + P(TTTTTTTTH)
= 1 − (P( A) + P(B ) − P( A ∩ B ))
+…
3
 1  1  1
6 9 Since A and B are independent this becomes
=   +   +   + ...
 2  2  2 = 1 − (P( A) + P(B ) − P( A)P(B ))
3 3

 1  1 = 1 − P( A) − P(B ) + P( A)P(B))
GP, a =   , r =  
 2  2 = (1 − P( A)) (1 − P(B ))

()
3
1 = P( A′ ) × P(B ′ )
2 1
S∞ = =
1− ( ) 1
3
7 8 (i) P(Ravi wins first game)
2 1 1 1
= × 0.2 + × 0.5 + × 0.8
3 3 3
(ii) P(Ben wins) = 0.5

= P(TH) + P(TTTTH) + P(TTTTTTTH) + . . . P (W ∩ W1)


2
(ii) P (W2 W1 ) =
2 5 8 P (W1)
=   +   +   +  . . .
1 1 1
 2  2  2
2 3 1 P ( Win both good) + P ( Win both average ) 
G. P, a =   , r =  
1 1
3  + P ( Win both beginner ) 
 2  2 = 

()
2 0.5
1 1
S∞ =
2 4
= =
8
=
2
1
(
0.22 + 0.52 + 0.82 )
()
2
7 = 3
1 4×7 7
1− 0.5
2 8
= 0.62
P(Ben wins at least 1 game in 10)
= 1 − P(Ben wins two games) 9 P(Green 1st draw)
10 g
= 1−  
5 =
= 0.965 (3 s.f.) (g + r )
 7
P(Green 2nd draw)
1 8
4 0.7 × + 0.3 =
3 15 = P(RG) + P(GG)
5 (i) P(Sahil will win)
r g g g −1
= × + ×
= P(Win in 3 sets) + P(Win in 4 sets) + g + r g + r −1 g + r g + r −1
P(Win in 5 sets) rg g( g − 1)
= +
3 3 1 3 2 ( g + r ) ( g + r − 1) ( g + r ) ( g + r − 1)
3 3 2 3 2
=   + 3×   ×   + 6×   ×   g 2 + rg − g
 5  5  5  5  5 =
= 0.68256 ( g + r ) ( g + r − 1)
= 0.683 (3 s.f.) g( g + r − 1)
=
( g + r ) ( g + r − 1)
g
=
(g + r )
17
Exam focus 4 1
S1 1 A = {1, 3, 5} and B = {2, 3, 5}
(iii)
x+4 5
=

20 = x + 4 ⇒ x = 16
A B P( AW and BW )
(iv) P( AW BW ) =
P( BW )

3 3 x +1
×
1 2 5 x+4
5 =
3 x +1 2 x
× + ×
6 5 x+4 5 x+4
3( x + 1)
S1 Statistics 1 Answers

4
5( x + 4 )
=
3x + 3 2x
+
A B 5( x + 4 ) 5( x + 4 )

3( x + 1)
= 5( x + 4 )
1 1 1 5x + 3
6 3 6 5( x + 4 )
1
3( x + 1) 5( x + 4 )
3 = ×
5( x + 4 ) 5 x + 3

3x + 3
1 =
(i) P(A ∩ B) = 5x + 3
3
1 1 1 2 3
(ii) P(A ∪ B) = + + = (from diagram) Die 1
6 3 6 3
1 1 3 4
P(A ∪ B) = P(A) + P(B) - P(A ∩ B)
1 2 2 4 5
1 1 1 2
= + − = (using the formula) 1 2 2 4 5
Die 2

2 2 3 3
(iii) Since P( A ∩ B) ≠ P( A) × P(B) i.e. 3 4 4 6 7
1 1 1, 4 5 5 7 8
≠ ×
3 2 2
A and B are NOT independent. 8 1
(i) = (ii)  6 = 3
16 2 16 8
Since P( A ∩ B ) ≠ 0
(iii) To find if two events are independent, see if
A and B are NOT mutually exclusive. P(A ∩ B) = P(A) × P(B). In this case
1 1 1
2 (i) After choosing a blue ball from box A and P(A ∩ B) = , P(A) = , P(B) =
16 4 16
placing it in box B, there are now x + 4 balls in
total and x of them are white. Clearly 1 ≠ 1 × 1 so the events are
16 4 16
4 NOT independent.
(ii) B
x+4
B In fact, if A does not happen, B cannot happen
2
5 x so they are certainly not independent.
x+4 W
3 In this case P(A ∩ B)  0 so the events are NOT
B
x+4 mutually exclusive.
3
5 W
x+1 W
x+4

18
4 (i) Using a Venn diagram: (i) P(on time) = 0.7 + 0.3 × 0.2 = 0.76

P(Alarm on on time)
S1
Boy Prefer Chocopops (ii)
P( Alarm on and on time)
=
P( on time )
0.7 35
9 15 6 = = = 0.921 (3 s.f.)
0.76 38

6 (i) P(at least one six) = 1 - P(no sixes)


10
5
10
= 1−  
9 + 15 + 6 30 3  6
P(Boy or Chocopops) = = =
40 40 4 = 1 − 0.16150...

S1 Statistics 1 Answers
OR = 0.838 (3 s.f.)
P(B or C ) = P(B ) + P(C ) − P(B and C ) 5
n
(ii) 1 −    0.99
24 21 15  6
= + −
40 40 40 n
 5   0.01
30 3  6
= =
40 4
Using trial and error, n . 25
(ii) G: person chosen is a girl.
(iii) P(Russell wins)
C : person chosen prefers Chocopops.
= P(A Blue, R Red)
If independent,
+ P(A blue, R blue, A blue, R red)
P (G and C ) = P (G ) × P (C )

but
6

16 21
× + P(A blue, R blue, A blue, R blue, A blue, R red)
40 40 40
=  ×  +  × × ×  +  × × × × × 1
5 4 5 4 3 4 5 4 3 2 1
so the events are not independent.  9 8  9 8 7 6  9 8 7 6 5 
23
5 1 = ≈ 0.365 (3 s.f.)
Alarm on On time 63
0.7

0.8 Late
0.3
Alarm off

0.2 On time

19
S1 4 Discrete random variables (iii) Sample space:

Die 1
Exercise 4.1
1 (i) p = 0.62 D 1 2 3 4 5 6

(ii) p + 2p + 3p + 4p = 1 1 0 1 2 3 4 5

10p = 1 2 1 0 1 2 3 4

p = 1 = 0.1 3 2 1 0 1 2 3
10

Die 2
S1 Statistics 1 Answers

(iii) p2 + 2p + 2p2 = 1
4 3 2 1 0 1 2
3p2 + 2p − 1 = 0
5 4 3 2 1 0 1
(3p – 1)(p + 1) = 0
1
6 5 4 3 2 1 0
p= or −1
3
1
Since probabilities cannot be negative p = d 0 1 2 3 4 5
3
6 10 8 6 4 2
2 (i) We could use a probability tree: P(D = d)
36 36 36 36 36 36
H P(HH) = 0.25
0.5
12
H 3 (i) We want to find the value of r so that = 3.
r
0.5 So r = 4.
0.5 T P(HT) = 0.25
The probability X = 3 is then 4 = 2
H P(TH) = 0.25 10 5
0.5
0.5 (ii)
T r 1 2 3 4
0.5 T P(TT) = 0.25 12 12 12 12
x = 12 =6 =4 =3
1 2 3 4

h 0 1 2 1 2 1 3 2
P(X = x) =
P(H = h) 0.25 0.5 0.25 10 10 5 10 5

(ii) 0.5 B BBB 0 Girls So the final table with the values of X in
B numerical order would look like this:
0.5
0.5 G BBG 1 Girl
B x 3 4 6 12
0.5 B BGB 1 Girl
0.5 0.5 G 2 3 1 1
P(X = x)
0.5 G BGG 2 Girls 5 10 5 10
0.5 B GBB 1 Girl
4 No. 4s 1 2 3 4
0.5 B
0.5
0.5 G GBG 2 Girls 1 0 0 0 1
G
0.5 B GGB 2 Girls 2 0 0 0 1
0.5 G 3 0 0 0 1
0.5 G GGG 3 Girls
4 1 1 1 2
g 0 1 2 3
x 0 1 2
P(G = g) 0.125 0.375 0.375 0.125
9 6 1
P(X = x)
16 16 16

20
5 (i) When X = 24, 120 = 24 ⇒ r = 5 2 (i) E(X ) = −1 × 0.65 + 4 × 0.35 = 0.75
r
( )
Var(X ) = ( −1)2 × 0.65 + 42 × 0.35 − 0.752 S1
So P( X = 24) = 5 = 1
15 3 = 5.6875
(ii) r 1 2 3 4 5 (ii) P(X  µ) = P(X  0.75) = P(X = 4) = 0.35

x 120 60 40 30 24 3 E(X ) = 5 × p + 10 × (1 − p) = 8
1 2 1 4 1
P(X = x) 5 p + 10 − 10 p = 8
15 15 5 15 3
10 - 5p = 8
(iii) The modal value of X is 24.
-5p = -2

S1 Statistics 1 Answers
(iv) P(35 < X < 90) = P( X = 40 or 60) 2
p= or 0.4
2 1 1 5
= + =
15 5 3
4 a + 0.2 + b + 0.4 = 1

6 (i) If the correct order is ABCD, we can look at a + b = 0.4


each of the 4! = 24 different arrangements
E( X ) = −3 × a + 1× 0.2 + 2 × b + 4 × 0.4 = 2.1
to see how many letters go in the correct
−3a + 0.2 + 2b + 1.6 = 2.1
envelope
−3a + 2b = 0.3
ABCD 4 BACD 2 CABD 1 DABC 0
Solve simultaneously
ABDC 2 BADC 0 CADB 0 DACB 1
ACBD 2 BCAD 1 CBAD 1 DBAC 1   a + b = 0.4 … (1)

ACDB 1 BCDA 0 CBDA 1 DBCA 2 3a + 2b = 0.3


… (2)
ADBC 1 BDAC 0 CDAB 0 DCAB 0
 3a + 3b = 1.2 … (1) × 3
ADCB 2 BDCA 1 CDBA 0 DCBA 0
9 3 5b = 1.5 ⇒ b = 0.3
     
P ( X = 0) = =
24 8
a = 0.1
     
(ii) x 0 1 2 3 4
5 E(X ) = 2 × p + 3 × (1 − p)
3 3 5 1
P(X = x) 0 = 2p + 3 − 3p
8 8 24 24 = 3− p
(iii) The modal values are 0 and 1. Var(X ) = 22 × p + 32 × (1 − p) − (3 − p)2
= 4 p + 9 − 9 p − ( 9 − 6 p + p2 )
Exercise 4.2 = p − p2
1 2 1 p − p2 = 0.24
1 (i) E(X ) = 0 × + 5 × + 10 × = 7
10 5 2
p2 − p + 0.24 = 0
Var(X ) =  02 × + 52 × + 102 ×  − 72
1 2 1
 10 5 2 using the quadratic formula,
= 11
p = 0.4 or 0.6
SD(X ) = 11 = 3.32 (3 s.f.)
6
x p − 400 000 p
(ii) E(X ) = 3 × 0.2 + 8 × 0.5 + 12 × 0.3 = 8.2
P(X = x) 0.0012 0.9988
(
Var(X ) = 32 × 0.2 + 82 × 0.5 + 122 × 0.3 − 8.22 )
= 9.76 E(X) = 500
SD(X ) = 9.76 = 3.12 (3 s.f.)
(p − 400 000) × 0.0012 + p × 0.9988 = 500
(iii) E(X ) = −1 × 0.15 + 0 × 0.35 + 2 × 0.5 = 0.85 0.0012p − 480 + 0..9988p = 500
(
Var(X ) = ( −1)2 × 0.15 + 02 × 0.35 + 22 × 0.5 − 0.852 ) p − 480 = 500
= 1.4275 p = $980

SD(X ) = 1.4275 = 1.19 (3 s.f.)

21
7 (i) P(double) = 1 17 1 1
S1 6 (iv) E(Y ) = −10 ×
36
+ 0 × + 2× +
18 9
5
D' Lose 1 token
1 1 1
  5 × + 6 × + 8 × + 10 × +
1
6 18 36 18
8 18
5 D' 1 1 1 1
6   14 × + 15 × + 20 × + 26 ×
1 18 36 18 36
D Win 3 tokens
6 1
= − ≈ −0.03 points
36
1
6 D Win 5 tokens 9 Let X be the number of tokens gained

x −1 3 5 x −3 5 10
S1 Statistics 1 Answers

4 1 1
25 5 1 P(X = x)
P(X = x) 6 6 6
36 36 6
4 1 1 3 1
25 5 1 E( X ) = −3 × + 5 × + 10 × = =   token
(ii) E(X ) = −1 × + 3× +5× 6 6 6 6 2
36 36 6
1
=
5
of atoken Number of tokens for 100 rolls is 100 × = 50
2
9
If he plays the game 50 times, his expected 0.5
5 7 10 (i) B
profit is 50 ×  = 27 tokens
9 9 0.5 B
1
8 (i) 10 × 1 = 15 so gain is 5 points
B G
2 0.5 0.5

(ii) White B 0.5 G


0.5
1 2 3 4 5 6 0.5 G
Die
Die Score 0.5 1 1.5 2 2.5 3 0.5 B
0.5
1 2 1 2 3 4 5 6 G 0.5 B

2 4 2 4 6 8 10 12 0.5 G 0.5 B
3 6 3 6 9 12 15 18 0.5 G
Red

4 8 4 8 12 16 20 24 0.5 G
5 10 5 10 15 20 25 30
P(X = 1)
6 12 6 12 18 24 30 36
=P
 (1G after 2 children) + P(1G after 3 children)
Points + P(1G after 4 children)
0.5 1 1.5 2 2.5 3
gain 1 1 1 1 11
= + + + =
2 −10 −10 −10 −10 −10 −10 4 4 8 16 16
4 −10 −10 −10 −10 0 2 (ii) x 0 1 2 3 4
6 −10 −10 −10 2 5 8 1 11 1 1 1
P(X = x)
16 16 8 16 16
8 −10 −10 2 6 10 14
10 −10 0 5 10 15 20
11
(iii) E( X ) = = 1.375
12 −10 2 8 14 20 26 8
2
11  11 55
Since there are 4 entries with a gain of Var( X ) = − = = 0.859 (3 s.f.)
4  8 64
2 points,

4 1
P(Y = 2) = =
36 9
(iii)
y −10 0 2 5 6 8 10 14 15 20 26
17 1 1 1 1 1 1 1 1 1 1
P(Y = y)
36 18 9 18 36 18 18 18 36 18 36

22
Stretch and challenge 3 Obviously P(H = 1) = p
1 (i)
1
P(Double) = , P(sum  7) =
15
=
5
P(H = 2) = p(1 − p), P(H = 3) = p(1 − p)2
S1
6 36 12
P(H = h) = p(1 − p)h−1
5
< 7 Win 2 lollies
12 4 E( X ) = p + 3(1 − p) = 3 − 2p
5 D' Var( X ) = 12 × p + 32 (1 − p) − (3 − 2p)2
6
7 = p + 9 − 9 p − (9 − 12p + 4 p2 )
≥ 7 Lose 1 lolly
12
= 4 p − 4 p2
E( X ) = 2Var( X )
1
6 D Win 5 lollies 3 − 2p = 2( 4 p − 4 p2 )

S1 Statistics 1 Answers
3 − 2 p = 8 p − 8 p2
x −1 2 5
8 p − 10 p + 3 = 0
2

35 25 1
P(X = x)
72 72 6
( 4 p − 3) (2p − 1) = 0
1 3
p= or
35 25 1 75 1 2 4
E( X ) = −1 × + 2× +5× = =1 .
72 72 6 72 24
Exam focus
(ii) Many answers are possible
1 (i) E( X ) = ∑ xp = −2 × 0.4 + 1× 0.1+ 2 × 0.5 = 0.3
If the prize for a double on the first lollies ‘c’
(ii) Var( X ) = ∑ x 2 p − {E( X )}2
and the prize for  7 on the second roll is ‘b’ = (−2)2 × 0.4 + 12 × 0.1+ 22 × 0.5 − 0.32
and otherwise she gives her friend ‘a’ then we = 3.7 − 0.32
require = 3.61
35 25 12 Standard deviation = Var( X ) =   3.61 = 1.9
−a × +b× +c× =0
72 72 72
i.e − 35a + 25b + 12c = 0 (iii) Since (−2)2 = 22 = 4 we can combine the
probabilities for the first and last values of X.
2 E(N ) = 1
x2 1 4
Consider when n = 1, it is obvious that E(N ) = 1 .
P(X2 = x 2) 0.1 0.9
Consider when n = 2, there is a 50% chance that the
first person will choose their name so E(N ) = 1 . 2 (i) P(S = 5) = P(2 then 3) + P(3 then 2)

1 1 1 1 Selection Sum
Consider when n = 3. The probability distribution = × + ×
3 2 3 2 1, 2 3
will be 1
= 1, 3 4
3
n 0 1 3 2, 3 5
1 1 1 Another way to find the possible sums is to
P(N = n)
3 2 6 write out all the possibilities (since there are
1 1 1 only 3C 2 = 3 possible sums).
So E(N ) = 0 × + 1× + 3 × = 1
3 2 6
Since each sum is equally likely,
Similarly for n = 4, 1
P(S = 5) =
3
n 0 1 2 4 (ii) s 3 4 5
9 8 6 1 1 1 1
P(N = n) P(S = s)
24 24 24 24
3 3 3
9 8 6 1
E(N ) = 0 × + 1× + 2× +4× =1
24 24 24 24 1 1 1
(iii) E(S) = 3 × + 4 × + 5 × =4
3 3 3

1 1 1 50 2
Var(S) = 3 × + 4 2 × + 52 × − 4 2 = − 16 =
2
3 3 3 3 3

23
(iv) h 2 3 (iii) E(W) = 0 × 1 + 1× 3 + 2 × 3 = 1 1 = 1.2
S1 1 2
10 5 10 5
P(H = h) (iv) P(W  E(W)) = P(W  1.2)
3 3

1 2 8 2 = P(W = 0) + P(W = 1)
µ = E(H) = 2 × + 3× = = 2
3 3 3 3 1 3
= +
3 (i) 1
W
1
B
10 5
4 WWB 2 White
7
=
10
W
1
2 W WBW 2 White 8 5 1
5 3
3 4 (i) × =
B 20 16 8
S1 Statistics 1 Answers

4
2
B WBB 1 White P (GA ∩ GB )
3
1
W BWW 2 White
(ii) (
P GA GB = ) P (GB )
3
1
W 8 11
3 2 ×
5 2 = 20 16
B BWB 1 White
3 8 11 12 10
B 2 × + ×
3
W BBW 1 White 20 16 20 16
1 0.275
2 B =
1 0.65
B BBB 0 White
3 = 0.423 (3 s.f.)
3 1 1 1
P(W = 0) = × × = 8 11
5 2 3 10 (iii) P( X = 0) = P(GG ) = × = 0.275
20 16
(ii) P(W = 1)  = P(WBB) + P(BWB) + P(BBW) P( X = 1) = P(GB ) + P(BG )
= × ×  + × ×  + × × 
2 3 2 3 1 2 3 1 2 8 5 12 10
 5 4 3  5 2 3  5 2 3 = × + ×
20 16 20 16
1 1 1 3 = 0.5
= + + =
5 5 5 5 12 6
P( X = 2) = P(BB ) = × = 0.225
Once we know two of the probabilities, the 20 16
other one can be found easily since the sum of
x 0 1 2
the probabilities is 1.
P(X = x) 0.275 0.5 0.225
 1 3 3
P(W = 2) = 1−  +  =
 10 5  10

w 0 1 2

1 3 3
P(W = w)
10 5 10

24
5 Permutations and 10 (i) If we treat John and Rachel as a single ‘object’,
S1
there are now 5 objects to arrange which can be
combinations done in 5! ways. In each of these arrangements,
John and Rachel could swap around, so the final
Exercise 5.1
answer is:
1 (i) 6! 5 720
5! × 2 = 240
(ii) 5! 5 120
AB   C   D   E   F
(iii) 2 3 5! 5 240
(ii) To find the number of ways where they are not
(iv) 2! 3 5! 5 240 together, subtract the number of ways they are
together from the total number of ways the

S1 Statistics 1 Answers
(v) 6 3 5 3 4 or 6 P3 = 120
group of 6 could be arranged:
5!
2 (i) = 60
2! 6! 2 (5! 3 2) 5 480
2 4! 13!
(ii) × 60 = 24 or × 2 5 24 11 (i) = 25740
5 2! 8 ! × 3!
3!
(iii) × 3! = 18 6!
= 120
2! (ii)
3!
3 (i) 5 3 5 3 5 3 5 3 5 5 3125 11!
(iii) = 990
8!
(ii) 5 3 4 3 3 3 2 3 1 5 5! 5 5 P5 5 120 8!
12 (i) = 560
3! 3! 2!
(iii) 5 3 4 3 3 5 5 P3 5 60
(ii) Grouping the red blocks as one object, we now
(iv) 4 3 3 3 2 3 2 5 48 have 6 objects – the red blocks, three green
6!
blocks and two blue blocks = 60
(v) 3 3 3 3 2 3 1 3 2 5 36 3! 2!

4 (i) 6! 5 720 (iii) First we can arrange the 3 red and 2 blue
5!
blocks in = 10 ways. In each of those
(ii) 4! 3 3! 5 144 3! × 2!
arrangements we can place the green blocks in
(iii) 3! 3 4 3 3 3 2 5 144 6×5× 4
the ‘gaps’ in = 20 ways
3!
(iv) 3! 3 2! 3 2! 3 2! 5 48 R R R B B

5 (i) There are 8 letters in the word, with three E’s


and two S’s. So the number of arrangements is So the total number of ways is 10 × 20 = 200
8!
= 3360 (iv) We need to consider the different cases of
3! × 2!
the top and bottom blocks being green, red
(ii) There are 6 digits, with two 1’s and two 9’s, so or blue.
the number of arrangements is
6! If the top and bottom blocks are green (or red),
= 180
2! × 2! we have 6 blocks to arrange in the middle, with
6 (i) 9! 5 362 880 two identical blocks and three identical blocks
in the middle.
(ii) 3! 3 3! 3 2! 3 4! 5 1728
6!
No. ways with green top and bottom5 = 60
(iii) 5! 3 6 3 5 3 4 3 3 5 43 200 3! 2!
If the top and bottom blocks are red, we will
6!
7 (i) = 180 also have 60 arrangements.
2! × 2!
5!
(ii) = 60 If the top and bottom blocks are blue, we will
2!
have 6 blocks to arrange in the middle – three
(iii) 180 2 60 5 120
identical red and three identical green blocks.
9!
8 (i) = 90 720
2! 2! No. ways with blue blocks top and bottom
8!
(ii) = 20 160 =
6!
= 20
2!
3! 3!
6! 7 × 6 × 5
(iii) × = 37 800 Total number of arrangements
2! 2!
9 2 3 2 3 2 5 8 5 60 1 60 1 20 5 140

25
Another method involves finding the 3 9
C5 = 9 C 4 = 126
S1 probability that the arrangement will have
two blocks at the top and bottom the same 4 (i) 14
C 3 = 364
colour, then multiplying this answer by the total
number of arrangements. (ii) 364 2 no. ways with no green blocks

P(same top and bottom) 5 P(Green or Red or 5 364 2 11C 3 5 364 2 165 5 199
Blue) (iii) 2
C 2 × 12 C1 = 12
3 2 3 2 2 1 7
5 × + × + × = (iv) YGR or YGB or YRB or GRB
8 7 8 7 8 7 28
S1 Statistics 1 Answers

No. ways 5
7
× 560 = 140 = ( 4 C1 × 3 C1 × 5 C1 ) + ( 4 C1 × 3 C1 × 2 C1)
28

+ ( 4 C1 × 5 C1 × 2 C1 ) + ( 3 C1 × 5 C1 × 2 C1)
(v) If the three red blocks are together there
6! = 60 + 24 + 40 + 30
are now = 60 arrangements. Of those 60
3! 2!
5! = 154
arrangements, there are = 20 arrangements
3!
where the blue blocks are together, so the 5 (i) 3
C 0 × 8 C 4 × 2 C1 = 140
number of arrangements where the three red (ii) 3
C 2 × 1C1 × 9 C 2 = 108
blocks are together and the two blue blocks are
(iii) No ‘O’s or one ‘O’
not next to each other is 60 – 20 5 40.

13 (i) 64 = 1296
5 ( 5
C5 × 8 C 0 ) + ( 5 C 4 × 8 C1 ) = 41

6 (i) 7
C 4 = 35
(ii) 6 × 5 × 4 × 3 = 6 P4 = 360
(ii) 2 or 3 tropical fruits
(iii) Total number 2 number with no red
5 3 C 2 × 4 C 2 + 3 C3 × 4 C1 = 22
5 1296 − 54 = 671
7 (i) 3! 3 2! 3 5! 3 4! 5 34 560
(iv) There are 6 colours to choose from for the ends,
and also 6 choices for both middle spots, since (ii) 7! 3 8 3 7 3 6 3 5 5 8 467 200
repeats are allowed.
(iii) 2
C1 × 5 C1 × 4 C1 = 40
6 × 6 × 6 = 216
(iv) 2H 2F 1N or 2H 1F 2N or 1H 2F 2N or

Exercise 5.2 1H 3F 1N or 1H 1F 3N
1 (i) 9
C 3 = 84
( 2
C 2 × 5 C 2 × 4 C1 ) + ( 2 C 2 × 5 C1 × 4 C 2 ) +
(ii) C 3 = 4 (or 4 C 3 × 5 C 0 = 4 ) ( C1 × 5 C 2 × 4 C2 ) + ( 2 C1 × 5 C 3 × 4 C1 ) +
4 2

(iii) 5
C 2 × 4 C1 = 40 ( 2
C1 × 5 C1 × 4 C 3 )
= 40 + 30 + 120 + 80 + 40
(iv) 1 woman or 2 women or 3 women
= 310
5 C1 × C 2 + C 2 × C1 + C 3 × C 0
4 5 4 5 4 5

8 4 different colours: 5 C 4 × 4 ! = 120
5 40 1 30 1 4
3 different colours:
5 74
If the colours chosen were B, W, R
or
2B 1W 1R or 1B 2W 1R or 1B 1W 2R
84 – no. ways with no women
4!
each in = 12 ways, total 36 ways.
5 84 –  5 C 3 2!
Total for 3 different colours 5 36 × 5 C 3 = 360
5 84 – 10
2 different colours:
5 74
First choose the colours in 5 C 2 ways.
2 (i) 15
C11 = 1365
If the colours chosen were blue and green we could
(ii) 7
C 6 × 6 C 4 × 2 C1 = 210
have
(iii) 14
C10 = 1001 4!
BBBG in 4 ! = 4 , GGGB in =4
3! 3!
26 (iv) 2
C1 × C10 = 572
13
4! (v) The girls can be arranged in 5! ways and the
BBGG in
2! 2!
= 6 , total of 14.
4 boys go in the ‘gaps’ between the girls. S1
Total for 2 different colours 5 C 2 × 14 = 140 5
5! × 6 × 5 × 4 × 3 43 200 5
= = = 0.119
9! 362 880 42
Total number 5 120 1 360 1 140 5 620
5 (i) 8
C 3 = 56

Exercise 5.3 1
C1 × 7 C 2 21 3
(ii) 8
= = = 0.375
3
C3 56 8
C2 3 1
1 (i) = =
7
C 2 21 7 1 1 1 1
(iii) × × = ≈ 0.003 (3  s. f.)
8 7 6 336
(ii) 1 2 P(no Funk CDs)
3
C0 × 4 C2 6 5

S1 Statistics 1 Answers
5!
5 1− 7
= 1− =
21 7 6 (i) = 60
C2 2!
1
C1 × 6 C1 6 2 (ii)
4! 2
= = 0.4
(iii) = =
7
C2 21 7 60 5
(iii) P(E first) 1 P(E second)
2 3! × 2! × 2! × 2! = 48 = 1
6! 720 15 2 3 3 2
= × × 1× 1× 1+ × × 1× 1× 1
5 4 5 4
4! 2 5!
×2 × 3
3 (i) 2! 24 2 or 5 2! 2 =
= = = 5
5! 60 5 5! 5
2! 2! OR
OR 2× 3× 3× 2×1 3× 2× 3× 2×1
+
= 2! 2!
There are two numbers out of the 5 that are 60
2 3
odd so P(end in odd) 5 =
5 5

(ii) No. ways starting with 2: 7 (i) 5


C 3 × 9 C 3 = 840

2_ _ _ _ = 4 ! = 24 (ii) 840 40
14
= ≈ 0.280 (3  s.f.)
C 6 143
No. ways starting with 4:
6
C6 1
4! 8 (i) = ≈ 0.000 000 261 (3 s.f.)
4_ _ _ _ = = 12 40
C 6 3 838 380
2!

Total no. ways 5 36


6
C 3 × 34 C 3 119 680
(ii) 40 = ≈ 0.0312 (3  s.f.)
C6 3 838 380
36 3
P(start with an even no.) 5 =
60 5 (iii) P(at least one even number)
OR 5 1 2 P(no even numbers)
20
There are 5 numbers, 3 of them are even so C6
5 1− 40
3 C6
P(start with an even no.) 5
5 38 760
5 1 − 3 838 380 ≈ 0.990  (3  s.f.)
(iii) To be greater than 30 000, the number must
start with a 3 or 4 (2 choices) 3
C3 1
(iv) = ≈ 0.000129 (3  s.f.)
4! 37
C 3 7770

P( 30 000) 5 2! = 24 = 2
60 60 5 6 1
9 (i) = ≈ 0.000 772
65 1296
4
C0 × 5 C 4 5
4 (i) P(no boys) 5 = 5
C 4 × 6 × 5 150 25
9
C4 126 (ii) = = ≈ 0.0193 (3  s.f.)
65 7776 1296
(ii) P(at least 1 girl) 5 1 2 P(no girls) 2 × 5! 5
(iii) = ≈ 0.0309 (3  s.f.)
C × 4 C4 5
65 162
5 1− 09
C4 (iv) There are 3 possible ways to get two 6s in at
125 most two rolls.
5
126
2
C2 × 7 C2 6 5 getting a six
21 1
(iii) 9
= =
C4 126 6
6′  5 not getting a six
2
C1 × C 3 70 5
7
(iv) 9
= =
C4 126 9

27
(6 6) or (6 6′ then 6) or ( 6′ 6′ then 6 6) 3 Let A be the event that at least two people share the
S1 1 1 1 5 1 same birthdate.
=  ×  + 2× × × 
 6 6  6 6 6
A′ is the event that no one shares the same birthday.
5 5 1 1
   +  × × × 
 6 6 6 6 P( A) = 1 − P( A′ )
= 0.0934  (3  s.f.)
365 364 363 362 365 − k + 1
= 1−  × × × × ... ×
 365 365 365 365 365 
10 (i) P(full house)
365!
C1 × 4C 3 × 12C1 × 4C 2
13
3744 (365 − k )!
= 52 = ≈ 0.001 44 ((3 s.f.) = 1−
C5 2598 960 365k
S1 Statistics 1 Answers

365!
(ii) (two pairs) = 1−
365k (365 − k )!
C 2 × ( 4 C 2 ) × 11C1 × 4 C1
13 2

123552
= = ≈ 0.0475
Exam focus
52
C5 2598 960
(iii) P(four of a kind) 1 (i) We must consider that the two 5s are
C1 × C 4 × C1 × C1
13 4 12
6244
identical. The answer is
= 52 = ≈ 0.000 240
C5 2 598 960 5!
= 60
2!
(iv) P(royal flush)
4
C1 4 (ii) If the number ends with the 3, the number of
= 52
= ≈ 0.000 00154 3!
C5 2598 960 arrangements is 2 × × 1= 6
2!

If the number ends in one of the 5s, the


Stretch and challenge number of arrangements is 2 ×
3! × 2
= 12
2!
 n  n   n + 1 So the final answer is 6 1 12 5 18
1   +  =
 r   r + 1  r + 1 5
C1 × 3C1 × 4C1 3
2 (i) P(one of each) = 12 = ≈ 0.273 (3 s.f.)
LHS : C3 11
n! n! 5
C1 × 7 C 2 105 21
= + (ii) P(one apple) = = =
(n − r )! r ! (n − r − 1)!(r + 1)! 12
C3 220 44
n !(r + 1) + n !(n − r )
= 5
C0 × 7 C3 7
(n − r )!(r + 1)! (iii) P( X = 0) = =
12
n!r + n! + n!n − n!r C3 44
=
(n − r )!(r + 1)) ! 5
C 2 × 7 C1 7
P( X = 2) = =
n !(n + 1) 12
C3 22
=
(n − r )!(r + 1)!
5
C3 × 7 C0 1
(n + 1)! P( X = 3) = =
= 12
C3 22
((n + 1) − (r + 1)) !(r + 1)!
 n + 1 OR
=
 r + 1
P( X = 3) = 1 −  + + =
7 21 7 1
 44 44 22  22
2 For each of the 12 players there are 11 possible
opponents for the first round x 0 1 2 3
No. of permutations = 12 × 11 but since e.g (a, b)
7 21 7 1
and (b, a) are different permutations, but the same P(X 5 x)
44 44 22 22
“pair” then
12 × 11 3 (i) 9! 5 362 880
No. of district pairs = = 66
2
(ii) There are now 6 separate groups:
OR simply
B1 B2 B3 B4 G1 G2 G3 G4 G5
12 × 11
No. of ways of choosing 2 from 12 = 12C2 = = 66
1.2 These 6 groups can be arranged in 6! ways. In
each of these ways, the boys can be arranged
in 4! ways. The total number of arrangements
is: 6! 3 4! 5 17 280

28
(iii) G1 B1 G2 B2 G3 B3 G4 B4 G5 5 (i) 7
C 4 = 35
S1
(ii) There are now three places left on the team
5 × 4 × 4 × 3 × 3 × 2 × 2 × 1 × 1
and 6 people to choose from so C 3 = 20.
6

The 5 girls can be arranged in 5! ways, the 4


(iii) Choose the boys and girls separately and
boys can be arranged in 4! ways. The number
multiply to get the total number of ways
of arrangements is: 5! 3 4! 5 2880 4
C 2 × 3 C2 = 18

(iv) There is one choice for the first position, the
rest of the students can be arranged in (iv) More boys than girls means 3 boys or 4 boys in
the team. The word or here means we add the
8! 5 40 320 ways number of combinations for each case:

S1 Statistics 1 Answers
No. teams with 3 boys = 4 C 3 × 3 C1 = 12
Front

1 × 8 × 7 × 6 × 5 × 4 × 3 × 2 × 1

No. teams with 4 boys = 4 C 4 × 3 C 0 = 1
(v) It would be wrong to say the answer is the
difference between (i) and (ii) … this leaves Total number of teams 5 12 1 1 5 13
other cases where 2 boys are standing next to
6 (i) S SW PU
each other and the others are separated.
3 1 1 8
C 3 × 4 C1 × 6 C1 = 1344
The way to approach this problem is to slot
the boys in the gaps between the girls: 2 2 1 8
C 2 × 4 C 2 × 6 C1 = 1008
G1 G2 G3 G4 G5 2 1 2 8
C 2 × 4 C1 × 6 C 2 = 1680
1 2 2 8
C1 × 4 C 2 × 6 C 2 = 720

The girls can be arranged in 5! ways. There are Total 5 1344 1 1008 1 1680 1 720 5 4752
6 spots for the first boy to go, 5 spots for the
(ii) 4 objects to arrange, then cars can be
second, 4 for the third and 3 for the fourth
swapped.
boy. The total number of arrangements is:
4 ! × 2! × 2! = 96
5! 3 6 3 5 3 4 3 3 5 43 200
(iii) Cars at end 2!
4 (i) The first person has 8 seats to choose from,
then the second has 7 to choose from and so 3! to arrange cars in the centre
on. P7 = 40 320
8
6 possible ways to arrange cars in a block of 6
(ii) There are 2 choices for the driver’s seat, 6 × 3! × 2! = 72
leaving 6 other people to fill the 7 remaining
7 (i) C D K
seats. 2 × 7 P6 = 2 × 5040 = 10 080
3 2 2 6
C 3 × 4 C 2 × 3 C2 = 360
(iii) Find the number of ways where they are next
to each other and subtract from the total. 2 3 2 6
C 2 × 4 C 3 × 3 C2 = 180
There are two choices for the driver’s seat and 2 2 3 6
C 2 × 4 C 2 × 3 C3 = 90
four choices of two seats together. In each of
these four arrangements, the two children can Total 5 360 1 180 1 90 5 630
be interchanged. (ii) 3! × 3! × 2! × 2! = 144
No. ways together 5 2! × ( 4 × 2) × P4 = 1920 5
(iii) 4 ! × 5 × 4 × 3 = 1440
No. ways apart 5 10  0 80 – 1920 5 8160

(iv) There are 4 window seats, so the number of


ways the girls can be seated at a window is
4 × 3 = 12.

The driver seat is taken, which leaves 5 seats


left to choose from for the other 4 family
members in 5 P4 = 120 ways.

Answer is 12 × 120 = 1440

29
S1 6 Binomial distribution 3 X : no. people who support party M
 7
(i) P( X = 4) =   × 0.38 4 × 0.623
Exercise 6.1  4
1 (i) Not appropriate = 0.174 (3 s.f.)

(ii) Appropriate X ~ B(3, 0.5) (ii) P( X  2) = 1 − P( X = 0 or 1)


 7  7
(iii) Not appropriate = 1 −   × 0.380 × 0.627 +   × 0.381 × 0.626
 0  1

(iv) Not appropriate
= 0.814 (3 s.f.)
S1 Statistics 1 Answers

The probability changes at each trial 2


4 (i) P(Holden car) =
3
(v) Appropriate, e.g. X ~ B(20, 0.9)
(ii) •  Two outcomes – Holden or Ford
1
(vi) Appropriate X ~ B  5,  2
 6 •  Probability constant – of cars are Holden
3
(vii) Could be appropriate if P(1st) is constant in
•  Each car is independent
each race, e.g. X ~ B (10, 0.2)
•  Fixed number of trials (10)
(viii) Not appropriate
 10  2  4  1  6
(iii) P( X = 4) =  × ×
Breakage of each egg would not be  4   3   3

independent. = 0.0569 (3 s.f.)

2 (i) 0.6 (iv) P(H > 8) = P(H = 8 or 9 or 10)


 10 2
8
1
2
 10 2
9
1
1

(ii) 0.4 × 0.4 = 0.16 =   ×   ×   +   ×   ×  


 8 3 3  9 3 3
(iii) C ~ B(5, 0.4)  10 2
10
1
0
+   ×   ×  
 5  10 3 3
(iv) P(C = 3) =   × 0.43 × 0.62 = 0.2304
 3 = 0.1951 + 0.0867 + 0.0173
(v) P(C  2) = P(C = 0) + P(C = 1) + P(C = 2)
= 0.299 (3 s.f.)
 5  5
=   × 0.40 × 0.65 +   × 0.41 × 0.64
 0  1 5 (i) •  There are two outcomes – blue or white

 5 • 
The probability is constant – the first ball is
+   × 0.42 × 0.63
 2 replaced

= 0.077 76 + 0.2592 + 0.3456 •  Each draw is independent

= 0.682 56 = 0.683 (3 s.f.) •  There are a fixed number of trials – 2 draws

(vi) P(C > 1) = 1 – P(C = 0) (ii) n = 2, p = 3 = 0.6


5
 5 So X ~ B(2, 0.6)
= 1 −   × 0.40 × 0.65
 0
= 0.922 (3 s.f.) (iii) P(X = 1) = 2 C1 × 0.61 × (1− 0.6)2−1 = 0.48

(iv) P(X > 1) = P(X = 1) + P(X = 2)


 n n− 3  n n− 4
(vii)  3  × 0.4 × 0.6 =  4 × 0.4 × 0.6
3 4

= 1 − P(X = 0)
n! n!
× 0.43 × 0.6n−3 = × 0.4 4 × 0.6n−44
3 ! ( n − 3) ! 4 !( n − 4 ) ! = 1 − 2 C0 × 0.60 × (1− 0.6)2−0
4 ! 0.6n−3 0.4 4 ( n − 3)! = 1 − 0.16
× = ×
3! 0.6n− 4 0.43 ( n − 4 )!
4 × 0.6 = 0.4 × (n − 3) = 0.84
2.4 = 0.4n − 1.2
3.6 = 0.4n
n=9

30
6 P(at least one correct) = 1 − P(none correct) 10 X : no. of thick crust pizzas in next n orders

= 1−
3 5
= X ~ B(n, 0.32)
S1
8 8
X : no. times he gets at least one correct in 5 tries P(X > 1) > 0.99

5
X ~ B  5,  1 − P(X = 0) > 0.99
 8
P(X = 0) < 0.01
P( X  1) = 1 − P( X = 0 or 1)
  5  5 0 3 5  5 5 1 3 4   n
= 1 −       +       
0 n

 0  0.32 0.68  0.01


   8   1  8   8 
  0 8 
= 1 − 0.0692 0.68n  0.01

S1 Statistics 1 Answers
= 0.931 (3  s.f.)
n  12
 10  10
7   × p4 × (1 − p)6 = 2 ×   × p6 × (1 − p)4 11 X : no. of puzzles solved each week
 4  6
(1 − p)6 p6 X ~ B(7, 0.85)
210 × = 420 × 4
(1 − p) 4
p
(i) P( X > 5) = P( X = 6 or 7)
(1 − p)2 = 2p2
 7  7
=   ( 0.85) ( 0.15) +   ( 0.85) ( 0.15)
6 1 7 0
1 − 2 p + p2 = 2 p2
 6  7
0 = p2 + 2 p − 1
= 0.717 (3  s.f.)
Using the quadratic formula, p = 0.414 (3 s.f.) (ii) (0.717...)3 = 0.368  (3 s.f.)
8 (i) X ~ B(8, 0.2) 12 X : no. times the train is late on any day
 8
(ii) P(X = 4) =   × 0.24 × 0.8 4 X ~ B(4, 0.12)
 4
= 0.0459 (3 s.f.) P( X > 1) = P( X = 2 or 3 or 4)

= 1 − P( X = 0 or 1)
(iii) P(Pass) = P(X = 5, 6, 7, 8)
 4  4 3
 8  8 = 1 −   ( 0.12) ( 0.88 ) +   ( 0.12) ( 0.88 ) 
0 4 1
=   × 0.25 × 0.83 +   × 0.26 × 0.82
 5  6  
0  
1 
= 1 − 0.9268...
 8  8
+   × 0.27 × 0.81 +   × 0.28 × 0.80 = 0.0732
 7  8
Y : n
 o. days the train is late more than once in the
= 0.0104 (3 s.f.)
next week
9 (i) X ~ B(7, 0.75)
Y ~ B(7, 0.0732)
 7
(ii) P(X = 7) =   × 0.757 × 0.250
 7  7
P(Y = 5) =   ( 0.0732) (1 − 0.0732)
5 2

= 0.133 (3 s.f.)  5

(iii) P(X > 3) = 1 − P(X = 0, 1, 2) = 0.000 0379 (3  s.f.)



 7
= 1 −   × 0.750 × 0.257 13 (i) X : depth of randomly chosen drum
 0

 7 x 8 10
+   × 0.751 × 0.256
1 P(X = x) 0.75 0.25
 7
    +   × 0.752 × 0.255 E( X ) = 8 × 0.75 + 10 × 0.25 = 8.5
 2
Var( X ) = 82 × 0.75 + 102 × 0.25 − 8.52 = 0.75
= 1 − 0.012 87…
(ii) Either 10” then 10” or 8” then 8”
= 0.987 (3 s.f.)
P(both the same)

= 0.25  ×   0.25  +   0.75  ×   0.75  =   0.625

31
(iii) For a depth of 34” we need exactly three 8” Stretch and challenge
S1 drums and one 10” drum 1
1 X ~ B  3,  
2
Y : number of 8” drums chosen out of 4
1
3
P( X = 0) =   = 0.125
Y ~ B(4, 0.75)  2
 3 1 1 1 2
P(Y = 3) = 4 C 3  ×   0.753  ×   0.251 = 0.422 (3 s.f.) P( X = 1) =       = 0.375
 1  2   2 
 3 1 2 1 1
Exercise 6.2 P( X = 2) =       = 0.375
 2  2   2 
1 C ~ B(5, 0.4)
S1 Statistics 1 Answers

P(sum = 2) = P(0, 2) + P(1, 1) + P(2, 0)


(i) E(C) = 5 × 0.4 = 2 = 0.125 × 0.375 + 0.375 × 0.375

+ 0.375 × 0.125
(ii) Var(C) = 5 × 0.4 × (1 − 0.4) = 1.2
= 0.234 (3 s.f.)
 
2 H ~ B 10, 2 
 3 2 X : no. patients that require further surgery from a
2 2 sample of n patients.
(i) E(H) = 10 × = 6
3 3
2  2 X ~ B ( n,  0.02)
(ii) Var(H) = 10 × × 1− 
3  3 P ( X = 0 ) ≈ 0.7
2
= 2  n
9
 0  0.02 0.98 = 0.7
0 n

2
SD(H) = 2 = 1.49 (3 s.f.)
9 0.98n = 0.7
n = 17.7
3 (i) n = 3 (3 balls drawn) p = 4 = 0.4 so sample size of 18
10
(ii) E(X) = 3 × 0.4 = 1.2 3 (i) P(at least one accident in 7 days)
(iii) Var(X) = 3 × 0.4 × (1 − 0.4) = 0.72 = 1 − P(no accidents in 7 days)
(iv) SD(X) = 0.72 = 0.849 (3 s.f.) = 1 − 0.887
4 np = 4 = 0.591 (3 s.f.)
np(1 − p) = 3.92
(ii) X : no. weeks where there is at least
∴ 4(1 − p) = 3.92 one accident on the freeway
3.92
(1 − p) = X ~ B (10,  0.591...)
4
p = 0.02 = 2% P ( X  3)

= 1 − P ( X = 0, 1,  2)
n = 200
 10  10 
  (0.591) (0.409) +   (0.591) (0.409) 
0 10 1 9

5 X : no. broken eggs in a carton  0   1 


= 1−  
  10 
X ~ B(12, p)  +   (0.591) (0.409)
2 8

  2 
E( X ) = np = 0.48 ⇒ 12p = 0.48 ⇒ p = 0.04 = 1 − 0.0143
= 0.986 (3 s.f.)
P(1  X  3)
4 E( X ) = p
 12  12
=   (0.04)1(0.96)11 +   (0.04)2 (0.96)10 Var( X ) = p − p 2 = 0.16
 1  2
 12 p 2 − p + 0.16 = 0
+   (0.04)3 (0.96)9           p = 0.2 orr 0.8
 3

= 0.386 (3 s.f.)

32
5 X : no. of people who fail to arrive 3 (i) Possible multiples of 4:

X ~ B ( 224,  0.04 ) 8, 12, 16, 20, 24 out of 20 possible days.


S1
P(not enough seats) Since each is equally likely,

= P ( X  3) P(multiple of 4) =
5 1
= = 0.25
20 4
 224  224 X ~ B ( 8,  0.25)
= (0.04)0 (0.96)224 +  (0.04)1(0.96)223
 0   1 

 224  224 (ii) P(2  X  4)


+ (0.04)2 (0.96)222 +  (0.04)3 (0.96)221
 2   3   8  8  8
=   0.2520.756 +   0.2530.755 +   0.2540.754
= 0.0200 (3  s.f.)  2  3  4

S1 Statistics 1 Answers
The assumption of independence may not be = 0.606 (3 s.f.)
satisfied as there are likely to be pairs or groups of
(iii) X ~ B ( n,  0.25)
people who are late for the same reason.
 n
P ( X = 0 )  0.02 ⇒   0.2500.75n  0.02
 0
Exam focus
⇒ 0.75n  0.02
1 1 1 1 7
1 (i) E( X ) = 0 × + 1× + 2 × + 3 × = ⇒ n  13.6
2 4 8 8 8
Var ( X ) The least possible value of n is 14.
2
1 2 1 1 1 7
= 02 × + 1 × + 22 × + 32 × −   2 1
2 4 8 8  8 4 (i) P(Win on any spin) = 10 = 5 = 0.2
71
= ≈ 1.11 (3  s.f.)
64 X  : number of times she spins a number
greater than 8 in 6 spins
1
(ii) X : no. of 1s from 10 throws, X ~ B  10,  X ~ B ( 6, 0.2)
 4
P ( X  3) P ( X > 1) = 1− P ( X = 0,1)

= P ( X = 0,1,  2,  3) 6   6 
= 1−   0.200.86 +   0.210.85 
 10 1 0
3  10 1
10
3
1 9
0  1  
=         +       
 0   4  4  1   4  4 = 1− 0.65536
 10 1 2
3  10 1
8 3
3
7
= 0.345 (3 s .f. )
+     +    
 2   4  4  3   4  4
k
= 0.776 (3  s.f.) (ii) P (1 spin  k ) =
10
1
(iii) X : no. of 2s from 10 throws, X ~ B  10,   k
X ~ B  n,  
 8  10 
 10 1 3 7 7
P ( X = 3) =       E( X ) = 30 ⇒
nk
= 30
 3   8  8 10
= 0.0920 (3 s.f.) nk  k
Var( X ) = 12 ⇒ 1 −  = 12
10  10 
k
2 np = 18 and np(1 − p) = 12 ⇒ 30  1 −  = 12
 10 
18(1 − p) = 12 k
⇒  1−  =
12
 10  30
12
1 − p = k 3
18 ⇒ =
10 5
1 − p = 2 ⇒k =6
3
1 nk
p= Substituting into = 30,
3 10
6n
1 = 30 ⇒ n = 50
n × = 18 so n = 54 10
3

33
5 (i) X : no. of flawed glasses in group of 10 (iii) In 50 boxes there are 500 glasses.
S1
X ~ B (10, 0.08 )
Z : no. of flawed glasses in 50 boxes.
 10
P ( X = 1) =   0.0810.929 = 0.378 (3 s.f.) Z ~ B(500,0.08)
 1

(ii) Y : no. boxes with at least one flawed glass  500
P( Z = 20) =  0.08200.92480 = 0.000128
 20 
 10
P ( X = 0 ) =   0.0800.9210 = 0.434 (3 s.f.)
 0
P ( X  1) = 1 − P ( X = 0 ) = 0.566 (3 s.f.)
S1 Statistics 1 Answers

Y ~ B(50,  0.566)
 50 
P (Y = 20 ) =   0.56620 (1 − 0.566)30
 20
= 0.007 15 (3 s.f.)

34
7 The normal distribution (iii) P(X  21.8)
S1
 21.8 − 30 
Exercise 7.1 = P Z  
 25 
1 (i) P(Z  1.92) = P ( Z  −1.64 )
= 0.9495
= 0.9726
(iv) P(24.7 < X < 40.25)
(ii) P(0 < Z < 0.765)  24.7 − 30 40.25 − 30 
= P Z 
 25 25 
= 0.2779
= P ( −1.06  Z  2.05)
(iii) P(Z  1.058) = 0.4798 + 0.3554

S1 Statistics 1 Answers
= 0.8352
= 1 − 0.855
2 (i) X : Waiting time to order
= 0.145 4 − 3.5
z= = 0.625
0.8
(iv) P(Z < −0.257)
P( X  4) = P( Z  0.625)
= 1 − 0.6014 = 1 − 0.7340
= 0.266
= 0.3986
2 − 3.5 
(ii) P( X  2) =  P  Z 
(v) P(−1.26  Z  2.417)  0.8 
=  P( Z  −1.875)
= 0.3962 + 0.4921 =  1 – 0..9697
= 0.8883 =  0.0303
(iii) P(2.8  X  5.2)
(vi) P(Z > −1.172)
2.8 − 3.5 5.2 − 3.5 
= P Z
= 0.8794  0.8 0.8 
= P( −0.875  Z  2.125)
(vii) P(0.068  Z < 1.925)
= (0.9832 - 0.5) + (0.8092 - 0.5)
= 0.9729 − 0.5271
= 0.7924
= 0.4458
3 (i) P(X > 395)
(viii) P(−1.818  Z  −0.844)
395 − 400 
= P Z 
= 0.9654 − 0.8006  3.3 
= P ( Z  −1.515)
= 0.1648
= 0.9351
(ix) P(−0.522  Z < 1.263) (ii) P(X > 402)
402 − 400 
= 0.3968 + 0.1992 = P Z 
 3.3 
= 0.5960 = P ( Z  0.606)
= 1 − 0.7276
Exercise 7.2 = 0.2724

1 (i) P(X > 38) (iii) P(404  X  408)


404 − 400 408 − 400 

 38 − 30  = P Z
= P Z    3.3 3.3 
 25 
= P (1.212  Z  2.424 )
= P ( Z  1.6)
= 0.9923 − 0.8873
= 1 − 0.9452
= 0.105
= 0.0548
(ii) P(X  23)

 23 − 30 
= P Z  
 25 
= P ( Z  −1.4 )
= 1 − 0.9192
= 0.0808

35
4 (i) (a) P(H  167) (ii) X : no. times she prepares the meal in under
S1

= P Z 
167 − 172  15 minutes in the next week
 
47 
= P ( Z  −0.729) X ~ B(7, 0.057)
= 1 − 0.7669
P( X  2) = P ( X = 0 or 1)
= 0.2
2331
 7  7
=   0.0570 (1 − 0.057) +   0.0571 (1 − 0.057)
7 6
(b) P(H  183)
 0 1
 183 − 172 
= P Z   = 0.944
 47 
= P ( Z  1.605) Exercise 7.3
S1 Statistics 1 Answers

= 1 − 0.9457 1 (i) a = 0.81


= 0.0543
(ii) b = -0.539
(c) P(155  H  163)
 155 − 172 163 − 172  (iii) c = 2.054 or 2.055
= P Z 
 47 47 
(iv) d = -1.439
= P ( −2.480  Z  −1.3
313)
= 0.9934 − 0.9054 (v) e = 2.576 (accept 2.574 < e < 2.579)
= 0.088
2 (i) P( Z  k ′ ) = 0.635 ⇒ k ′ = 0.345
(ii) P(H  180)
0.345 =
k − 12


180 − 172  2.5
= P Z  
 47  k = 12.8625 = 12.9 (3 s.f.)
= P ( Z  1.167)
(ii) P( Z  k ′ ) = 0.218 ⇒ k ′ = −0.779
= 0.8784 k − 12
−0.779 =
No. students less than 180 cm 2.5
k = 10.0525 = 10.1 (3 s.f.)
≈ 86 × 0.8784 = 75.5
3 P( Z  k ′ ) = 0.03 ⇒ k ′ = −1.881
so 75 or 76 students
k − 22
−1.881 =
5 (i) P(B  60) 0.6
60 − 56  k = 20.9 s (3 s.f.)
= P Z 
 13  4 P( Z  k ′ ) = 0.04 ⇒ k ′ = 1.751
= P ( Z  0.308 )
1.751 =
k − 45
= 0.6209
4.8
(ii) P(B > 75) k = 53.4 m (3 s.f.)
75 − 56 
= P Z  5 (i) P( Z  k ′ ) = 0.005 ⇒ k ′ = 2.576
 13 
2.576 =
k − 56
= P ( Z  1.462) 13
= 1 − 0.9282 k = 89.5% (3 s.f.)
= 0.0718
(ii) P( Z  k ′ ) = 0.25 ⇒ k ′ = −0.674
(iii) P(20 < B < 50) k − 56
−0.674 =
20 − 56 50 − 56  13
= P Z
 13 13  k = 47.2% (3 s.f.)
= P ( −2.769  Z  −0.462) 6 (i) P( Z  k ′ ) = 0.98 ⇒ k ′ = −2.054
= 0.9972 − 0.6779
−2.054 =
k − 400
= 0.3193
3.3
6 (i) P(T  15) k = 393 ml (3 s.f.)
 15 − 20  (ii) P( Z  k ′ ) = 0.008 ⇒ k ′ = 2.41
= P Z  
 10  k − 400
2.41 =
= P ( Z  −1.581) 3.3
= 1 − 0.9430 k = 408 ml (3 s.f.)
= 0.057

36
Exercise 7.4 8 M: mark in exam

1 P( Z  k ′ ) = 0.3 ⇒ k ′ = −0.524 (i) P(M  46)


S1
12 − µ 46 − 56 
    − 0.524 = = P Z 
5  13 
µ = 14.62 = 14.6 (3 s.f.) = P ( Z  −0.769)
2 P( Z  k ′ ) = 0.9 ⇒ k ′ = −1.282 = 1 − 0.7791
6−8 = 0.2209 ≈ 22.1%
−1.282 =
σ (ii) 100% − 22.1% = 77.91%
σ = 1.56 (3 s.f.)
77.91%
= 38.955% so we want the top 38.955%
3 P( Z  k ′ ) = 0.03 ⇒ k ′ = −1.881 2

S1 Statistics 1 Answers
9.5 − µ P( Z  k ′ ) = 0.389 55 ⇒ k ′ = 0.28
−1.881 =
15 m − 56
0.28 =
µ = 16.8  (3 s.f.) 13
    m = 59.64% ≈ 60%
4 P(Z  z) = 0.843 ⇒ z = 1.007
30 − µ 9 X : height of a randomly chosen Year 12 student.
1.007 = 1
20 X ~ N  µ, µ
 15 
1.007 × 20 = 30 − µ
(i) First find the z value so that P( Z  z ) = 0.04  or
µ = 30 − 1.007 × 20 P( Z  z ) = 0.96 .
µ = 25.5 (3 s.f.) z = 1.751
5 T : time for trip 182 − µ
1.751 =
1 1
P(T  30) = µ
5 15
1
P( Z  k ′ ) = 0.2 ⇒ k ′ = 0.842 1.751 × µ = 182 − µ
15
30 − 25 0.11673µ = 182 − µ
0.842 =
σ µ +   0.11673µ = 182
σ = 5.94 (3 s.f.)
1.11673µ = 182
6 P( Z  k ′ ) = 0.15 ⇒ k ′ = 1.036 µ = 162.975
P( Z  k ′′ ) = 0.01 ⇒ k ′′ = −2.326 µ = 163
3 cm(3  s.f.)

1.5 − µ (ii) First find the probability that any one student
1.036 = ⇒ 1.036σ = 1.5 − µ
σ is over 180 cm tall.
1− µ
−2.326 = ⇒ −2.326σ = 1− µ 1 1
σ σ= µ= × 163 = 10.865 = 10.9 (3  s.f.)
subtract the two equations 15 15
180 − 162.97 
3.362σ = 0.5 P( X  180) = P  Z 
 10.865 
           σ = 0.149 m (3 s.f.)
= P ( Z  1.567)
          µ = 1.35 m (3 s.f.)
= 1 − 0.9414
7 (i) P( Z > k ′ ) = 0.005 ⇒ k ′ = 2.576 = 0.0586
P( Z < k ′′ ) = 0.4 ⇒ k ′′ = −0.253 The problem now switches to a binomial

120 − µ
2.576 = ⇒ 2.576σ = 120 − µ distribution problem, where
σ
90 − µ Y  : number of students over 180 cm tall in a
−0.253 = ⇒ −0.253σ = 90 − µ
σ
group of 5. Y ~ B(5, 0.0586)
subtract the two equations
P(Y  1)
2.829σ = 30
         σ = 10.6 min (3  s.f.) = 1 − P(Y = 0 or 1)
          µ = 92.7 min (3  s.f.)  5   5 
= 1 −   0.058600.94145 +   0.058610.9414 4 
  0   1  
(ii) P (T  105)
= 1 − ( 0.9695)
105 − 92.7... 
= P Z  = 0.0305
 10.6... 
= P ( Z  1.162)
= 0.8774
No. students
≈ 18 000 × 0.8774 ≈ 15 793 students

37
10 S ~ N(µ, σ 2)
12 4σ 2 = 3µ ⇒ µ = 4σ
2

S1 P(S  100) = 0.24


3
P( Z  k ) = 0.0309 ⇒ k = −1.867
 1
P(S  90) = 0.04 µ − µ
 1   2 
P W  µ =P Z
 2   σ 
 
0.24 1 1 4σ 2
0.04 µ − µ − µ −µ −
2 = 2 = = 3 = −4σ
90 µ 100 σ σ 2σ 2σ 6
−4σ
P(Z  z1) = 0.24 → z1 = 0.707 = −1.867 ⇒ σ = 2.80  (3  s.f.)
S1 Statistics 1 Answers

6
P(Z  z 2) = 0.04 → z 2 = −1.751 4σ 2 4 × 2.802
µ= = = 10.5 (3  s.f.)
3 3
100 − µ
0.707 = → 0.707σ = 100 − µ Exercise 7.5
σ
90 − µ
−1.751 = → −1.751σ = 90 − µ 1 (i) A: no. shoes of brand A sold A ~ B(6, 0.3)
σ
subtract 2.458σ = 10 B: no. shoes of brand B sold B ~ B(6, 0.7)
σ = 4.07 km/h  6
(a)  P(B = 3) =   0.7 0.3 = 0.185 (3 s.f.)
3 3

 3
substitute 0.707 × 4.07 = 100 − µ (b)  P( A > 3) = P( A = 4, 5, 6)
µ = 97.1 km/h (3  s.f.)   =  6  0.340.72 +  6 0.350.71 +  6 0.360.70
 4  5  6
11 (i) P(Z  k) = 0.02 ⇒ k = −2.054   = 0.0705 (3  s.f.)

10 − µ
−2.054 = (ii) X : no. shoes of brand B sold in a week
1
µ
8
X ~ B(48, 0.7)
1
−2.054 × µ = 10 − µ
8 µ = 48 × 0.7 = 33.6

−0.25675µ = 10 − µ σ 2 = 48 × 0.7 × 0.3 = 10.08
0.74325µ = 10 X ≈ N(33.6,10.08)
µ = 13.454 P( X  30)

µ = 13.5 (3  s.f.)  29.5 − 33.6 
= P Z  
(ii) P(X + 1  µ) ⇒ P(X  µ - 1 )  10.08 
= P ( Z  −1.291)
= P(X  12.5 )
= 0.9017
 
12.5 − 13.5  2 F : no. flat batteries from 10 sampled

P( X  12.5) = P  Z 
1 
 × 13.5  F ~ B(10, 0.125)
8
= P(Z  −0.5946)
(i) P(F  2) = P(F = 0,1)
= 1 − 0.7241
= 0.276 (3  s.f.)  10  10
=   (0.125)0 (0.875)10 +   (0.125)1(0.875)9
 0  1
  = 0.639 (3  s.f.)
16 − 13.5 
(iii) P( X  16) = P  Z 
1 
 × 13.5 
8
= P ( Z  1.514 )
= 1 − 0.935
= 0.065

38
(ii) X : no. flat batteries in a box of 200 (ii) P( X  2µ ) = P( X  2 × 12.4...) = P( X  24.8...)

X ~ B(200, 0.125)     S1
 24.8... − 12.4...   2µ − µ 
=P Z  or P Z 
 1   1 
µ = 200 × 0.125 = 25  × 12.4...   µ 
2 2
σ 2 = 200 × 0.125 × 0.875 = 21.875
= P ( Z  2)
X ≈ N(25, 21.875)
= 0.9772
P( X  20)
(iii) Y : no. observations less than 10
 19.5 − 25 
= P Z  
 21.875  Y ~ B(120, 0.35)
= P (Z  −1.176)
Since np  5 and n(1 − p)  5 use normal
= 1− 0.8802

S1 Statistics 1 Answers
approximation.
= 0.1198
3 A: no. Brand A mp3 players sold in one month µ = 120 × 0.35 = 42
σ = 120 × 0.35 × (1 − 0.35) = 27.3
2
A ~ B  60, 
 3 Y ≈ N( 42, 27.3)
2  39.5 − 42 
µ = 60 × = 40 P(Y  40) = P  Z  
3  27.3 
2 1 1 = P ( Z  − 0.478 )
σ 2 = 60 × × = 13
3 3 3
= 1 − 0.6837
1
A ≈ N  40, 13  = 0.316 (3 s.f.)
 3
P(run out) = P(A  45) 6 X : number of drivers who pass at the first attempt
 
 44.5 − 40 
X ~ B(1200, 0.82)
= P Z  
 1  np  5 and n(1 − p)  5 so normal approximation is
13
 3 
appropriate.
= P ( Z  1.232)
= 1 − 0.8911 µ = 1200 × 0.82 = 984
= 0.1089
σ 2 = 1200 × 0.82 × (1 − 0.82) = 177.12

4 X : no. students who use the internet for more than X ≈ N(984, 177.12)
24 hours per week from the 950 students
P(X  950) becomes P(X  949.5) with a continuity
16
p= = 0.32 correction.
50
X ~ B(950, 0.32)  949.5 − 984 
P(X  949.5) = P  Z  
 177.12 
µ = 950 × 0.32 = 304
= P ( Z  −2.592)
σ 2 = 950 × 0.32 × (1− 0.32) = 206.72
= 1 – 0.9952
X ≈ N(304, 206.72)
= 0.0048
P(275  X  318)
 274.5 − 304 Assuming that the probability that any person passes
318.5 − 304 
= P Z   on the first attempt is 0.82, there is only a very small
 206.72 206.72 
= P (−2.052  Z  1.009) chance (less than 0.5%) that less than 950 people will
= 0.4799 + 0.3434 pass out of 1200.
= 0.8233 The testing procedures are about right at this centre.
5 (i) P( Z  k ) = 0.35 ⇒ k = −0.385
Even though the percentage who passed is 79% –
10 − µ
− 0.385 = not much difference from 82%, the probability of
1
µ
2 0.0048 shows that there is only an extremely small
1 chance that this would happen.
− 0.385 × µ = 10 − µ
2
−0.1925µ = 10 − µ
0.8075µ = 10
µ = 12.4

39
Stretch and challenge Exam focus
S1 1 X : diameter of a randomly chosen bolt  2

C ~ N  µ ,   µ  
2
1 (i)
 3  
X ~ N (18,  0.22 )
 
P(rejected) 0 − µ
P (C  0 ) = P  Z 
= P ( X  17.68 ) + P ( X  18.32)  2 
 µ 
3 
17.68 − 18  18.32 − 18 
= P Z  + P Z  3
 0.2   0.2  = P Z  − 
 2
= P ( Z  −1.6) + P ( Z  1.6)
= 0.933
= 2(1 − 0.9452)
S1 Statistics 1 Answers

(ii) H ~ N (175, σ 2 )
= 0.1096 (0.110)
182
18.5 − 18  P ( H  195) = = 0.0303
P(X  18.5) = P  Z  6000
 0.2 
P( Z  k ′ ) = 0.0303 ⇒ k ′ = 1.8
875
= P( Z  2.5) 195 − 175
1.875 =
= 1 − 0.9938 σ
= 0.0062 1.875σ = 20
P ( X  18.5) σ = 10.7 (3 s.f.)
P ( X  18.5 rejected) =
P (rejected)
 2

2 D ~ N  µ ,  µ  
1
0.0062
=  4  
0.110
= 0.0566 P ( Z  k ′ ) = 0.045 ⇒ k ′ = −1.695
2 X : IQ score, X ~ N (100, 15 2
) 10 − µ
−1.695 =
1
µ
P (100  X  105) 4
 105 − 100  −0.42375µ = 10 − µ
= P 0  Z  
 15  0.57625µ = 10
= P (0  Z  0.333) µ = 17.4 (3 s.f.)
= 0.1304
3 W : weight of randomly chosen apple
P (105  X  k ) = 0.12
W ~ N (160,  62 )
⇒ P (100  X < k ) = 0.1304 + 0.12 = 0.2504
150 − 160 

P 0  Z 
k − 100  = 0.2504 (i) P(W  150) = P  Z 
 15   6 
 k − 100  = 0.7504 = P ( Z  −1.667)
⇒ P Z 
 15  = 1 − 0.9522
k − 100 = 0.676 ⇒ k = 110.14 = 0.0478
15
(ii) Proportion of medium and large apples
                              k = 110
3 X : no. of correct guesses = 1 − 0.0478 = 0.9522

Assuming the person is guessing, Medium : Large

X ~ B (60 000,  0.2) 3  : 1

Since np  5 and n(1 − p)  5 use normal 0.714  : 0.238


approximation.
P (W  w ) = 0.238
µ = 60 000 × 0.2 = 12000

P( Z  k ′ ) = 0.238 ⇒ k ′ = 0.713
σ = 60 000 × 0.2 × (1− 0.2) = 9600
w − 160
X ≈ N (12000,  9600) 0.713 =
6
w = 164.3g
 12283.5 − 12000 
P( X  12284) = P  Z  
 9600
= P ( Z  2.893)
= 1 − 0.9981
= 0.0019 ( 2 s.f.)
 
Since the probability of picking the correct card that many
times is very low, we can conclude there is evidence of ESP.

40
4 X : no. babies born before their due date (ii) F : no. of forwards on time

X ~ B(160,  0.25)
F ~ B (8,  0.762) S1
P(F  7)
Since np  5 and n(1 − p)  5 use normal = P(F = 7 or 8)
approximation.
 8  8
=   0.76270.2381 +   0.76280.2380
µ = 160 × 0.25 = 40  
7  8

σ = 160 × 0.25 × (1 − 0.25) = 30 = 0.398  (3  s.f.)
X ≈ N ( 40, 30 ) (iii) B: no. of backs on time
 50.5 − 40  B ~ B (25,  0.762)
P( X  50) = P  Z  
 30 

S1 Statistics 1 Answers
  = P ( Z  1.917) Since np  5 and n(1 − p)  5 use normal
= 1 − 0.9723 approximation.
  = 0.0277 (3 s.f.) µ = 25 × 0.762 = 19.05

σ = 25 × 0.762 × (1 − 0.762) = 4.53...
5 T : time that players arrive at training relative to start B ≈ N (19.05,  4.53...)
time
 17.5 − 19.05 
P(B  18) = P  Z 
(i) T ~ N (−5, 72 )  4.53... 

0 − ( −5)  = P ( Z  −0.728 )
P(T  0) = P  Z 
 7  = 1 − 0.7666
= P ( Z  0.714 ) = 0.233 (3 s.f.)
= 0.762

41
S1 S1 Past examination 4 (i) a = 40

(ii)
questions 250
Length of cars

240
230
Chapter 1 Exploring data & Chapter 2 220
Representing and interpreting data 210
200
190
1 (i) Median = 78 kg 180
170
LQ = 72 kg, UQ = 88 kg 160

frequency density
S1 Past examination questions

150
(ii) Weights 140
130
120
110
Country Q 100
90
80
70
60
Country P 50
40
30
20
50 60 70 80 90 100 110
10
weight (kg) 0
2.8 2.9 3 3.1 3.2 3.3 3.4
(iii) •  People are heavier in P than Q length (m)

•  Weights are more spread out in Q than P


745
5 (i) x= = 41.38 = 41.4 (3 s.f.)
18
2
Number 0 2
33 951
s= − 41.42 = 13.2 (3 s.f.)
Frequency 23 17 18
17

∑fm = 0 × 23 + 2 × 17 = 0.85 ∑x 17
x= (ii) x 17 = 1
= 41 so ∑ x = 697
∑f 40 17 1

Since the sum of the ages of the 18 people


s2 =
∑fm 2

− x 2 = 0 × 23 + 2 × 17 − 0.852 = 0.9775
2 2

was 745, the person who left must be


∑f 40

745 – 697 = 48
3 Area 0– 500– 1000– 2000– 3000–4000 33 951 − 482
s= − 412 = 13.4
Frequency 3 7 6 3 1 17
Frequency
0.006 0.014 0.006 0.003 0.001 6 (i) 16
density

(ii) 8
Floor area of factories

(iii)
0.015 Matches 1 2 3 4 5
0.014
0.013 Frequency 16 8 4 2 2
0.012
frequency density

0.011
0.010 (iv)
0.009
Matches, x 1 2 3 4 5
0.008
0.007
Frequency, f 16 8 4 2 2 32
0.006
0.005
xf 16 16 12 8 10 62
0.004
0.003
x2f 16 32 36 32 50 166
0.002
0.001
62
0 Mean = = 1.9375 = 1.94 (3 s.f.)
500 1000 1500 2000 2500 3000 3500 4000 32
area (m2)
166
Variance = − 1.93752
32
= 1.43 (3 s.f.)

42
7 (i) Chapter 3 Probability
S1
10 4 4 9
11 5 7 1 (i) 113, 131, 311, 122, 212, 221
12 0 4 5
Total no. of outcomes = 6 × 6 × 6 = 216
13 2 4
6 1
P(Total of 5) = =
14 2 5 216 36
15 8 (ii) Ways of getting a total of 7:
16 0 2
115, 151, 511
10|4 represents 104
124, 142, 214, 241, 412, 421
(ii) LQ = 115

S1 Past examination questions


133, 313, 331
Median = 125
223, 232, 322
UQ = 145 15 5
P(Total of 7) = =
216 72
(iii)
2 Rachel
0.7

Rachel
0.7
0.3
Anna

90 100 110 120 130 140 150 160 170 Rachel


0.4
Rachel
pulse rate (bpm)
0.3 Anna
0.6
8 (i) Let y = x − 60
0.6
y=
∑ y = 245 = 3.5 Anna
n 70 Rachel
0.7
x = 3.5 + 60 = 63.5 km/h
Rachel
(ii) ∑ ( x − 60) = ∑ x − ∑ 60 = ∑ x − 70 × 60 0.4 0.4
0.3
∑ x − 4200 = 245 ⇒ ∑ x = 4445 Anna
Anna
Rachel
∑ ( x − 50) =∑ x − ∑ 50 0.4
= 4445 − 70 × 50 0.6 Anna
= 945
(iii) Let z = x − 50 0.6
Anna
s x = s y = sz = 10.6
(i) P(wins 1st | loses 2nd)
945 P(wins 1st ∩ loses 2nd )
z= = 13.5 =
70 P(loses 2nd )

= 0.6 × 0.3
sz =
∑z 2

−z 2 0.6 × 0.3 + 0.4 × 0.6


n
∑z 2 = 0.18 = 3 = 0.429 (3 s.f.)
10.6 = − 13.52 0.42 7
70
∑ z = 20 622.7
2 (ii) P(win 2, lose 1)

∑ ( x − 50) = 20 622.7 = 20 600 (3 s.f.)


2
= 0.6 × 0.7 × 0.3 + 0.6 × 0.3 × 0.4 +
0.4 × 0.4 × 0.7
= 0.31

43
3 Colour-blind 6 (i) 24 = 24 × 1, 12 × 2, 8 × 3, 6 × 4
S1 0.05
Q: (12, 2) or (2, 12), (8, 3) or (3, 8), (6, 4) or (4, 6)
Male 6 1
P(Q ) = =
144 24
0.54
0.95 Not colour-blind 4 1
(ii) P(one die > 8) = =
12 3
1 1 1
P(both die > 8) = × =
3 3 9
Colour-blind
OR
0.02
S1 Past examination questions

0.46 List all 16 posibilities:


Female
(9, 9) (9, 10) (9, 11) (9, 12) (10, 9) etc…
0.98 Not colour-blind (iii) Since P(Q ∩ R ) = 0, Q and R are exclusive.
P(Male  Colour-blind)
(iv) If independent, then
= P(Male ∩ Colour-blind)
P(Colour-blind) P(Q ∩ R ) = P(Q ) × P(R )
0.54 × 0.05 1 1
= but 0 ≠ ×
0.54 × 0.05 + 0.46 × 0.02 24 9
so Q and R are not independent.
= 0.027 = 0.746 (3 s.f.)
0.0362
OR
4 On-time
0.96
If the events were independent then
1
A P (Q   R ) = P(Q ) = . But P (Q   R ) = 0 so the
24
0.5 0.04 Late events are not independent.
On-time
0.94 7 (i) > 65
0.05
0.3
B
0.25
Cell 30–65
0.06 Late 0.68
0.7
On-time < 30
0.2 0.83

C > 65
0.64
0.32
0.17 Late 0.1
No cell 30–65
(i) P(late)
0.26
< 30
= 0.5 × 0.04 + 0.3 × 0.06 + 0.2 × 0.17
P (C ∩ 30 − 65)
(ii) P (C 30 − 65) =
= 0.072 P ( 30 − 65)
P(B and late)
(ii) P(B | late) = 0.68 × 0.25
P(late) =
0.68 × 0.25 + 0.32 × 0.1
0.3 × 0.06 0.018 = 0.842 (3 s.f.)
= = = 0.25
0.072 0.072

0.25
5 P(1) = P(2) = P(3) = P(4) = P(6) = = 0..05
5
P(1, 5, even) = 0.05 × 0.75 × (3 × 0.05)
9
= 0.005 625 =
1600

44
8 (i) No 2 (i) X : no. of new pens in a sample of two
Designer
labels
designer
labels Total 2 New S1
9
High-heeled
2 4 6 New
shoes 3
Low-heeled 10
1 3 4 7
shoes Old
9
Sports
5 5 10
shoes
3 New
Total 8 12 20 9
7
10 Old
1
(ii) = 0.05
20
6

S1 Past examination questions


1 9 Old
(iii) = 0.5
2
P(one new pen)
2
P ( H ∩ D ) 20 1  3 × 7  +  7 × 3  = 42 = 7
(iv) P (H D) = = = = 0.25 =
 10 9   10 9  90 15
P (D) 8 4
20 7 6 42 7
(ii) P(no new pens) = × = =
1 8 2 10 9 90 15
(v) P( S ) = , P(D) = =
2 20 5
7 7
5 1 P(two new pens) = 1 −  +  = 1
P ( S ∩ D) = =  15 15  15
20 4
If independent, then OR = 3 ×2= 6 = 1
10 9 90 15
P( S ∩ D) = P( S ) × P(D)
x 0 1 2
1 1 2
≠ ×
4 2 5 7 7 1
P(X = x)
so the events are not independent. 15 15 15

(vi) X : no. of days she wears designer shoes


7 7 1 9 3
(iii) E( X ) = 0 × + 1× +2× = = = 0.6
X ~ B(7,  0.4) 15 15 15 15 5

P( X  4) 3 (i) X : smaller of 2 scores or score if the same


= 1− P( X = 5,  6,  7) X 1 2 3 4 5 6
7 5 2 7 6 1
  0.4 0.6 +   0.4 0.6  1 1 1 1 1 1 1
5  6 
= 1−   2 1 2 2 2 2 2
7 7 0
+   0.4 0.6  3 1 2 3 3 3 3
 7 
= 1− 0.096 4 1 2 3 4 4 4
= 0.904 5 1 2 3 4 5 5
6 1 2 3 4 5 6
Chapter 4 Discrete random variables
1 (i)
a 1 4 9 16
x 1 2 3 4 5 6
1 1 1 1
P(A = a)
2 6 6 6 P(X = x)
11 9 7 5 3 1
36 36 36 36 36 36

(ii) E(A) = 1 × 1 + 4 × 1 + 9 × 1 + 16 × 1 = 5 1
2 6 6 6 3 (ii) E(X)
2
1 1 1 1 1
Var(A) = 12 × + 42 × + 92 × + 162 × −  5  = 1×
11
+ 2×
9
+ 3×
7
+4×
5
+5×
3
+6×
1
2 6 6 6  3 36 36 36 36 36 36
2 91
= 59 1 −  5 1  =
36
= 2.53 (3 s.f.)
3  3
8
= 30 = 30.9 (3 s.f.)
9

45
4 (i) X : the number of dice that fall on a green 9!
S1 X ~ B(5, 0.25)
3 (i) 2! × 2!
= 90 720

5! × 4 !
 5 (ii) = 720
P(X = 4) =   0.2540.751 = 0.0146 (3 s.f.) 2! × 2!
 4
4 (i) 9! = 362 880
 5
(ii) P(X = 0) =   0.2500.755 = 0.237
 0 (ii) 6! × 7 × 6 × 5 or 6! × 7 P3 = 151 200

 5 (iii) 1 or 2 or 3 woman
P(X = 1) =   0.2510.754 = 0.396
 1
= ( 3
C1 × 6 C 2 ) + ( 3 C 2 × 6 C1 ) + ( 3 C 3 )
S1 Past examination questions

 5
P(X = 2) =   0.2520.753 = 0.264
 2 = 45 + 18 + 1

 5 = 64
P(X = 3) =   0.2530.752 = 0.0879
 3

OR Total no. ways − no. ways with all men
 5
P(X = 5) =   0.2550.750 = 0.000 977
 5 = C3 − C3
9 6

= 84 − 20
x 0 1 2 3 4 5

P(X = x) 0.237 0.396 0.264 0.0879 0.0146 0.001 = 64

5 (i) 6!
= 120
3!
5 (i) x 0 1 2 (ii) 4!
5_ _ _ _ 7 = = 12
2!
1 4 2 4!
P(X = x) 7_ _ _ _5 = = 12
7 7 7 2!
7_ _ _ _ 7 = 4 ! = 24
1 4 2 8
(ii) E( X ) = 0 × + 1× + 2 × = Total no. ways = 12 + 12 + 24
7 7 7 7
2 = 48
1 2 4 2 8
Var( X ) = 02 × + 1 × + 22 × −  
7 7 7  7
6 (i) 9 ! = 362 880
= 0.408 (3 s.f.)
(ii) No. ways with pink and green together
P (G ∩ A′ )
(iii) P (G A′ ) =
P ( A′ ) = 8 !× 2 = 80 640
2 1
× No. ways apart
= 5 4
2 1 3 9
× + ×
5 4 5 10 = 362 880 - 80 640
5
= = 0.156 (3 s.f.) = 282 240
32
(iii) 9
P3 = 504 or 9 C 3 × 3! = 504
Chapter 5 Permutations and
(iv) 1
C1 × 8 C 2 × 3! = 168
­combinations
1 (i) 8
C 2 = 28 (v) No. ways with pink next to green

(ii) 8
C1 + 8 C 2 + 8 C 3 + 8 C 4 = 162 = PG _ or GP _ or _ PG or _ GP

(iii) 1624 = 688 747 536 = 7 + 7 + 7 + 7 (or 7 × 2! × 2 )

2 (i) (a) 18 C6 = 18 564 = 28

(b) 17 C5 = 6188 No. ways apart = 504 − 28 = 476

(ii) (a) 8! = 40 320

(b) 4! × 5! = 2880

46
10 ! (ii) X ~ B(n, 0.2)
7 (a) 5! × 4 !
= 1260
P(X > 1) > 0.85
S1
(b) (i) 8
P4 or C 4 × 4 ! = 1680
8

1 − P(X = 0) > 0.85


(ii) 2 C2 × 6 C2 × 4 ! = 360
P(X = 0) < 0.15
(c) A B C
 n 0 n
 0 0.2 0.8  0.15
7 1 1 9
C 7 × 2 C1 × 3 = 216
0.8n  0.15
5 3 1
9
C5 × 4 C 3 × 1C1 × 3! = 3024
n > 9 (guess & check)
3 3 3 9
C 3 × 6 C 3 × 3 C 3 = 1680

S1 Past examination questions


4 X : no. of New Year’s Days on a Saturday
Total = 216 + 3024 + 1680 = 4920
1
X ~ B  15, 
 7
P(X > 3) = 1 − P(X = 0, 1, 2)
Chapter 6 The binomial distribution  15 1
0
6
15
= 1 –   ×   ×  
1 X : no. of plants per box which produce  7  7
 0 
yellow flowers
 15  1  1  6  14  15  1  2  6  13 
+   ×  7  ×  7  +   ×  7  ×  7  
X ~ B(n, p)  1  2 
= 1 − (0.0990 + 0.2476 + 0.2889)
(i) E(X) = np = 11
= 0.365 (3 s.f.)
Var(X) = np(1 − p) = 4.95
5 X : no. throws that result in a 5
by substitution:
X ~ B(10, 0.75)
 11(1 − p) = 4.95
P(X > 8) = P(X = 8, 9, 10)
  (1 − p) = 0.45
 10  10
=   × 0.758 × 0.252 +   × 0.759 × 0.251
     p = 0.55  8  9

∴ n × 0.55 = 11  10
+   × 0.7510 × 0.250
 10
n = 20
= 0.526 (3 s.f.)
(ii) X ~ B(20, 0.55)
6 (i) X : no. of fireworks that fail to work
 20
P(X = 12) =   0.55120.458 = 0.162 (3 s.f.)
 12  X ~ B ( 20, 0.05)
2 X : total no. of eggs laid P ( X  1)

X ~ B(30, 0.7) = 1 − P ( X = 0 or 1)

 30   20 
P(X = 24) =   0.7240.36 = 0.0829 (3 s.f.) = 1 − 0.9520 +   (0.0
05)1(0.95)19 
 24   1 
3 X : no. of damaged tapes = 0.264 (3 s.f.)

X ~ B(15, 0.2) (ii) P: profit for company


(i) P(X < 2) Profit if no refund

= P(X = 0) + P(X = 1) + P(X = 2) = 450 × 10 − 20 × 24 = $4020


 15  15  15
=   0.200.815 +   0.210.814 +   0.220.813 Expenditure = 20 × 24 = $480
 0  1  2
= 0.398 (3 s.f.) p 4020 −480

P(P = p) 0.736 0.264



E(P ) = 4020 × 0.736 + −480 × 0.264
= $2832

47
 120 − 112 
Chapter 7 The normal distribution P(X  120) = P  Z >
17.2 
S1 1 (i) Since P(X  3.6) = 0.5, µ = 3.6 = P ( Z > 0.465)

= 1 − 0.6790

0.6554 = 0.321

x (ii) With new fertilizer, X ~ N(115, σ 2 )


1 2 3 4 5 6 7
P(X  103) = 0.80
P(X  2.8) = 0.6554
103 − 115
−0.842 =
S1 Past examination questions


z value = −0.4 (from table) σ
2.8 − 3.6 −0.842σ = −12
−0.4 =
σ
σ = 14.3 (3 s.f.)
−0.4σ = 2.8 − 3.6
4 X : no. eggs laid by 30 hens in one day
σ  = 2
X ~ B(30, 0.7)
(ii) Y : no. of observations  2.8 out of four
Since np  5 and nq  5 use normal approximation.
Y ~ B(4, 0.6554)
X ≈ N(21, 6.3)
P(Y > 2) = 1 − [P(Y = 0) + P(Y = 1)]
 4
= 1 −   (0.6554)0 (0.3446)4
 0 
 4 
+   (0.6554)1(0.3446)3 
 1 
= 1 − [0.0141 + 0.107]
10 20 30 40
= 0.879 (3 s.f.)
 19.5 − 21
P(X  19.5) = P Z  
2 X : no of plants in a box which produce pink flowers  6.3 
= P (Z  −0.598)
X ~ B(100, 0.3)
= 1 − 0.7251
Since np  5 and nq  5 use normal
approximation with = 0.2749

µ = 100 × 0.3 = 30 5 X : weights of melons

σ = 100 × 0.3 × 0.7 = 21 = 4.58 X ~ N(450, 1202)

(i)

20 25 30 35 40 45

 
P(X  34.5) = P  Z < 34.5 − 30 
 21 
100 200 300 400 500 600 700 800
= P ( Z < 0.982)
P(X  350) = P  Z < 350 − 450 
= 0.8370  120 
= P ( Z < −0.833)
3 (i) X : height of sunflowers
= 1 − 0.7975
X ~ N(112, 17.22)
= 0.2025

50 70 90 110 130 150 170

48
(ii) The upper area needs to be divided in two. 8 X : time spent visiting dentist
We want the z-value so
(
X ~ N 8.2, σ 2 )
S1
0.7975
P(Z  z) = = 0.398 75
2 (i) P ( X  10 ) = 0.79
z = 0.256 (from tables) 10 − 8.2 
P Z  = 0.79
 σ 
x − 450
0.256 = 10 − 8.2
120 = 0.807
σ
x = 480.72 g = 481 g (3 s.f.)
σ = 2.23 (3 s.f.)

6 X ~ B(1600, 0.2)

np  5 and n(1 − p)  5 so use normal approximation.

S1 Past examination questions


µ = 1600 × 0.2 = 320 0.79

σ = 1600 × 0.2 × 0.8 = 16


3 4 5 6 7 8 9 10 11 12 13 14
so X ≈ N(320, 162 )
(ii) P ( X  7.2) + P ( X  9.2)
9.2 − 8.2 
= 2× P Z 
 2.23 
= 2 × P ( Z  0.448 )
= 2 × (1 − 0.6729)
= 0.654  (3  s.f.)
280 300 320 340 360 380

(iii) P( X  10) = 0.21


P(X > 290) = P(X  289.5) continuity correction
289.5 − 320  Y  : no. of people whose visits last longer than
  = P  Z 
 16  10 minutes
  = P( Z  −1.906)
Y ~ B ( 6, 0.21)
  = 0.9717 P (Y  2)
= 1 − P (Y = 0,1, 2)
7 X ~ N (µ , σ 2 )
 6  6
(i) P ( X  2µ ) = 1 −   (0.21)0 (0.79)6 +   (0.21)1(0.79)5
  0  1
   6 
2µ − µ  +   (0.21)2 (0.79)4 
= P  Z   2 
3µ 
 
5  = 1 − 0.888
5 = 0.112 (3  s.f.)
= P Z  
 3
= 0.952 (3 s.f..) (iv) 
T : no. people with visits lasting less than

8.2 min from sample of 35 people.
1
(ii) P  X  µ T ~ B ( 35, 0.5)
 3 

 1 µ = 35 × 0.5 = 17.5
µ − µ
= P  Z  3 
σ 2 = 35 × 0.5 × (1 − 0.5) = 8.75
σ 
 

 2 Since np  5 and n(1 − p)  5, normal


− µ
= P  Z  3  approximation is appropriate.

σ 

X ≈ N(17.5, 8.75)
2
− µ P(T  16)
3 = 1.047
σ  15.5 − 17.5 
− 2 µ = 3.141σ = P Z  
 8.75 
µ = −1.57σ = P ( Z  −0.676)

= 1 − 0.7505
= 0.250 (3 s.f.)

49
S2 S2 Statistics (c) 
Since 0.0113  0.05, there is sufficient
evidence at the 5% significance level to

2 Answers 3 (i)  
conclude that the manager is correct.

(a)  P(X   2) = 0.0192

8 Hypothesis testing using P(X   10) = 0.0192


P(X    2 or X   10) = 0.0384
the binomial distribution 4% or 5% significance level

Exercise 8.1  (b)  P(X  2 or X  10)


= P(X  2) + P(X  10)
S2 Statistics 2 Answers

1 (ii) X: the number of voters that support Party A


= 0.0192 + 0.0192 = 0.0384 = 3.84%
X ~ B(25,   0.35)
H0 : p = 0.35, H1: p  0.35 (ii)  P(X  9) = 0.0537 + 0.0161 + 0.0029 + 0.0002
= 0.0729 = 7.29%
(iii) X: the number of apples that have developed 7.29%  5% so accept H0
bruising after 1 week
X ~ B(400, 0.02)
4 (i) H0: p = 0.25, H1: p  0.25
H0 : p = 0.02, H1: p  0.02
(ii) X ~ B(20, 0.25)
(iv) 
X : the number of people who make a purchase
P(X  x ) ≈ 0.05
(out of all the people visiting the store)
P(X  1) = 0.0243 = 2.43%
X ~ B(100, 0.92)
P(X  2) = 0.0913 = 9.13%
H 0: p = 0.92, H1: p  0.92
The critical region is X  1
2 (i) (a) 0.5 × 10 = 5 (iii) X = 3 is outside the critical region
(b)
H ~ B(10, 0.5) so do nott reject the null hypothesis.
There is insuffficient evidence to conclude that
P(H  8) = 0.0547 the progrramme has been effective.
(c)
0.0547 = 5.47%  5% 5 (i) X ~ B(20, 0.1)
Do not reject H0 H0 : p = 0.1, H1: p  0.1
P(X  5) = 1 − P(X  5)
Not enough evidence to say that the coin is   = 0.0432 = 4.32%
biased. 4.32%  10%, reject H0
There is evidence that Samantha has
(ii) (a) E(X) = 0.35 × 25 = 8.75
a special ability.
(b)
P(X  3) = 0.0097 = 0.97%
(ii) P(X  x)  0.1
(c)
H0 is not true at the 10% significance level. P(X  4) = 0.133
So rejection region is X  5
There is evidence to say that the support is
less than 35%.
6 X ~ B  30, 
1
(iii)
(a) 0.02 × 400 = 8 6
1 1
H0 : p = , H1: p 
(b)
P(X  3) 6 6
= P(X  = 0) + P(X  = 1) + P(X  = 2) + P(X  = 3) P(X  2) = 0.103 = 10.3%
= 0.000 3093 + 0.002525 + 0.010 28 + 0..02784 10.3%  5%, do not reject H0
= 0.0410 = 4.10% There is no evidence that the dice is biased.

(c)
4.10%  4%: do not reject H0

(iv)
(a) E(X) = 100 × 0.92 = 92

(b)
P(X  98)
= P(X = 98) + P(X = 99) + P(X = 100)
= 0.008 95 + 0.002080 + 0.000 239
= 0.0113

50
7 X ~ B(9, 0.65) 2 (i) P(X  8) = 5.47%
H0 : p = 0.65, H1: p ≠0.65 P(X  9) = 1.07% S2
P(X  2) = 0.0112 Critical region is X  9
P(X  3) = 0.0536 P(Type I error) = 0.0107 (3 s.f.)
So X  2
P(X  8) = 0.121 (ii) X ~ B(10,  0.6)
P(X  9) = 0.0207 P(Type II error) = P(X  9   p = 0.6)
X  9 gives closest value to 2.5%       = 1 − P(X  9)    p = 0.6
Claim rejected for X = 9 and X  2      = 1 − 0.0464
     = 0.954  (3 s.f.)
8 (i) X ~ B(10, 0.28)

S2 Statistics 2 Answers
H0 : p = 0.28, H1: p  0.28 (iii) The probability that Pieta concludes that the coin
P(X  5) = 0.118 = 11.8% is no
ot biased when it actually is biased.
11.8%  5%, do not reject H0
No evidence that her shooting has improved. 3 (i) X ~ B(14, 0.35)
H0 : p = 0.35, H1: p  0.35
(ii) P(X  5) = 11.8% P(X  3) = 0.221 = 22.1%
P(X  6) = 0.342 = 3.42% 22.1%  5%, do not reject H0
X  6 gives closest value to 5% There is insufficient evidence to con
nclude
Rejection region is X  6 that the support has decrreased.

(ii) X ~ B(14, 0.35)
1
9 X ~ B(10, ) P(X  1) = 0.0205
3
P(X  2) = 0.0839
1 1
H0 : p = , H1: p  Critical region is X  1
3 3
P(X  6) = 0.076 = 7.6% P(Type I error) = 0.0205 (3 s.f.)
P(X  7) = 0.0197 = 1.97%
(iii) X ~ B(14, 0.2)
X 7
P(Type II error) = P(X  1   p = 0.2)
Aris must provide correct evidence 7 times.
= 1 − P(X  1   p = 0.2)
= 1 − 0.198
10 (i) H0 : p = 0.15, H1: p  0.15
= 0.802 (3 s.f.)
(ii) P(X  1) = 0.176 = 17.6%
17.6%  5%, do not reject H0 4
4 (i) X ~ B(7,  )
(iii) If n  18, P(X  0)  5% 7
4 4
or for P(X  0)  0.05   H00 : p = ,  H11: p 
7 7
n  19 (using logarithms) P(X  6) = 0.124
Not sufficient evidence to reject H0 , P(X  7) = 0.0199
even if no patients turn up. Critical region is X  7
P(Type I error)  =  0.0199
This is the probability the council conclude
Exercise 8.2 that th he level of contamination is rising,
1 when re eally it is not rising.
Decision
H0 accepted H0 rejected (ii) Type II error: probabbility that the council
conclude that the levvel of contamination
Reality

H0 is true OK Type I error


is not rising, when rea
ally it is rising.
H0 is false Type II error OK Type II error is more imp portant to avoid
for public safety.

51
1 Exam focus
S2 5 (i) H1: p    
6
1 (i) H0 : p = 0.1, H1: p  0.1
1
(ii) N ≈ N  40, 33  (ii)
 3 X : no. of faulty tiles, X ~ B(12, 0.1)
P(N  n)  0.05 P(X = 0) = 0.912 = 0.2824
(n − 0.5) − 40  12
= 1.645 P(X = 1) =   0.110.911 = 0.3766
1 1
33
3
 12
⇒ n = 49.99 P(X = 2) =   0.12 0.910 = 0.2301
2
i.e. N  50
 12
S2 Statistics 2 Answers

 1 P(X = 3) =   0.13 0.99 = 0.0852


(iii) P(Type II error) = P  N  49   p =  3
 4
P(X  2) = 0.341 ( 0.05)
 49.5  −  60  P(X  3) = 0.111 ( 0.05)
     = P  Z  
 45 
X ~ B(n, p) P(X  4) = 0.0256 ( 0.05)
       = P(Z  −1.565)
H0 : p = 0.4, H1: p  0.4 Critical region: X  4
     = 0.0588  (3 s.f.)
P(X = 0) = 0.6n  0.05
(iii) 3 does not lie in critical region, do not reject H0
nlog0.6  log0.05
Not sufficient evidence to show that cheaper
Stretch    and ⇒ n challenge
5.86
way increases the proportion of faulty tiles.
So n  6
1 X ~ B(n, p)
Also need n such that 2 (i) H0 is rejected when H0 is true.
H0 : p = 0.4, H1: p  0.4
P (X = 0) + P (X = 1)  0.05 The die is not biased but Ishani concludes
P(X = 0) = 0.6n  0.05
 n n−1 that it is biased.
0.6n + nlog0.6 0.410.6  0.05
 log0.05
 1  X : no. of fours in six rolls, X ~ B(6, 0.25)
     ⇒ n  5.86
n9 P(Type I error) = P(X = 5 or 6)
So n  6
6n9
Also need n such that  6  6
=   (0.25)5 (0.75)1 +   (0.25)6 (0.75)0
P (X = 0) + P (X = 1)  0.05  5  6
2 X ~ B(10, p)
 n p(11 − pn)−= = 0.00464 (3 s.f.)
Var(
0. + ) = n0.4
6n X 0.6 1  p(1 − p)
100.05
1 
10p(1 − p)  1.6 (ii) P(Type II error)
n9 1 4
6p −np+ 90.16  0 ⇒ 5  p  5
2
= P(X  4    p = 0.5)

1 = 0.891 (3 s.f.)
so  p = makes 5 alternatives for each answer.
5
X ~ B(10, p)
Var( 1 p(1 − p) = 10p(1 − p)
pX=) = nwould also work.
3 X : no. of goals scored from18 shots
4 H0: p = 0.74, H1: p > 0.74
10p(1 − p)  1.6
1 4 Under H0 , X ~ B(18, 0.74)
p2 − p + 0.16  0 ⇒  p 
5 5 P( X  16) = 0.116
1 0.116 > 0.10, do not reject H0
so  p = makes 5 alternatives for each answer.
5
Not sufficient evidence to show that he has improved
d.
1
p = would also work.
4

52
9 The Poisson distribution 7 (i) mean = variance = 0.15
S2
(ii) X ~ Po(0.15)
Exercise 9.1
(All answers given to 3 s.f.) P(X  1) = 1- P(X = 0, 1) = 0.0102

e −1.8 1.82 (iii) Y ~ B(n, 0.0102)


1 (i) P(X = 2) = = 0.268
2!
e −1.8 1.85 P(Y  1) = 1- P(Y = 0) = 0.95
(ii) P(X =  5) = = 0.0260
5!
P(Y = 0) = 0.9898 n = 0.05
(iii) P(X  1) = 1 − P(X = 0) = 0.835
n = 292 or 293
(iv) P(X = 2) + P(X = 3) + P(X = 4) = 0.501

S2 Statistics 2 Answers
8 (i) W : number of accidents on a weekday
2 (i) X ~ Po(3)
W ~ Po(0.8), P(W = 3) = 0.0383
P(X = 0) = e -3 = 0.0498
S: number of accidents on Saturday or Sunday
(ii) Y ~ Po(1.5)
S ~ Po(1.4), P(S = 3) = 0.113
P(Y = 2) = 0.251
5 2
0.0383 ×     +   0.113 ×   = 0.0596
(iii) Z ~ Po(6) 7 7

P(Z  2) = 1 - P(Z = 0, 1) = 0.983 (ii) P(W = 0) = 0.449


P(S = 0) = 0.247
3 (i) X ~ Po(3.6)
X : number of accidents on any given day
P(X = 6) = 0.0826 5 2
P(X = 0) = 0.449 ×   + 0.247 ×   = 0.391
7 7
(ii) Y ~ Po(0.3)
P ( weekday ∩ (X = 0))
P ( weekday | X = 0 ) =
P(Y = 2) = 0.0333 P(X = 0)
5
× 0.449
(iii) Z ~ Po(1.8) P(W = 0) 7
= = = 0.820
P(X = 0) 0.391
P(Z  1) = 0.835
9 (i) X ~ Po(0.125)
4 (i) ∑ f = 120
∑ fx = 312 P(X  2) = 0.000 296

312 (ii) P(X  1) = 0.118


= 2.6
120
Y ~B(n, 0.118)
(ii) The variance, s 2, is 3.51. Since the mean and
variance are not equal, the Poisson model is P(Y  1) ≈ 0.99
inappropriate. P(Y = 0) ≈ 0.01
5 (i) Y ~ Po(8) 0.882n ≈ 0.01

P(Y = 10) = 0.0993 log0.01


n≈ = 36.7
log0.882
˙
(ii) Z ~ Po(1.33)
n = 37
P(X  2) = 0.385
50
10 (i) = 2.78
6 P(X  1) = 0.32 18
1 − P(X = 0) = 0.32 (ii) X ~ Po(2.8)

P(X = 0) = 0.68 P(X  4) = 0.692


−λ
e λ 0
(iii) Y ~ Po(5.6)
= 0.68
0!
P(Y = 3) = 0.108
e − λ = 0.68
λ = −ln0.68 = 0.386

53
(iv) σ 2 = 2.8, σ = 1.67 (iii) L: number of accidents in 10 days
S2 a = 2.8 − 3 × 1.67
M: number of accidents in northbound
direction in 10 da
ays
= -2.22 N: number of accidents in southbound
direction in 10 days
Since a  0, a = 0
M ~  Po(2.5), N ~  Po( 4), L ~ Po(6.5)
b = 2.8 + 3 × 1.67 P ( M = 2 ∩ L = 6)
P(M = 2 | L = 6) =
P(L = 6)
= 7.82
P (M = 2 ∩ N = 4)
=
(v) P(X = 0) = 0.0608 P(L = 6)
S2 Statistics 2 Answers

e −2.5 2.52 e −4 4 4
 × 
Z ~ B(12, 0.0608) = 2! 4!
e −6.5 6.56
P(Z = 3) = 0.0281 6!
= 0.318

Exercise 9.2 4 X : number of faults in wool carpet in the house


(All answers given to 3 s.f.) Y : number of faults in nylon carpet in the house

1 (i) X + Y ~ Po(8) X ~ Po(0.64)

(ii) X - Y not Poisson Y ~ Po(1.2)

(iii) 2X + 3Y not Poisson (i) P(X  1)  0.473

(iv) 4X + 4Y not Poisson (ii) P(Y  2)  0.879

2 (i) X + Y ~ Po(5) (iii) Z : number of faults in carpet


e −5 51
P(X + Y = 1) = = 0.0337 Z ~ Po(1.84)
1!
(ii) T ~ Po(5) P(Z  1) = 0.841

(iii) P(T  2) = 1 − P (T = 0 or 1) (iv) P(Z = 0) = 0.15


= 1− e −5
(1 + 5) e − λ = 0.15
= 0.960 λ = 1.90
3 (i) A A
X: number of accidents in 4 days in northbou
und 0.02 ×     +   0.06  × = 1.90
2 2
direction
A = 47.5 m2
Y : number of acidents in 5 days in southbound
direction
5 (i) Cars cannot arrive simultaneously.
X ~ Po(1)
Cars arrive at a constant rate.
Y ~ Po(2)
The arrival of one car does not affect the
7 7 7 7 7 14
λ Z = λ X + λY = (1) + (2) = + = 4.55 arrival of another car.
4 5 4 5 4 5
Z ~ Po(4.55)
(ii) X : number of cars arriving at the petrol station
P(Z = 0) = e −4.55 = 0.0106 in a 2-minute interval
(ii) K : number of accidents in 2 weeks
X ~ Po(7.6)
K ~ Po(9.1)
P(K  3) = 1 − P(K = 0, 1,  2)  7.62 7.63 
P(X  4) = 1 − e −7.6 1 + 7.6 +  +   = 0.945
 9.12   2! 3! 
= 1 − e –9.1  1  +   9.1  +  
 2! 
= 0.994

54
(iii) Z : total number of arrivals at the petrol station 2 (i) Faults in capacitators are independent.
in a 3-minute interval The probability that the capacitator is faulty is S2
the same for each capacitator.
Z ~ Po(13.8)
Capacitor is either faulty or not.
P(Z  2) = 0.000 015 0 There are a fixed number of capacitors.

(iv) Y : number of trucks arriving at the petrol (ii) n large, p small, np  5
station in a 1-minute interval
λ = 0.4
Y ~ Po(0.8)
(iii) P(X  3) = 1 − P(X = 0, 1, 2)
P(X  1) = 0.978  _  0.42  
= 1 − e 0.4  1 +   0.4  +  

S2 Statistics 2 Answers
  2!  
P(Y = 1) = 0.359
= 0.007 93
P(X  1 and Y = 1) = 0.978 × 0.359 = 0.351
3 (i) X : number of ‘out of shape’ cans in a crate
6 (i) Faults cannot occur simultaneously.
X ~ B(24, 0.001)
Faults occur at a constant rate.
Faults occur independently and randomly. Since np  5, use Poisson approximation.

(ii) X : number of faults occuring in 1 day X ≈ Po(0.024)


X ~ Po(0.15)
P(X  1) = 1- P(X = 0) = 1 - e - 0.024 = 0.0237
(a) 
P(X = 0) = 0.861
(ii) Y : number of boxes that have at least 1
(b) 
P(X  2) = 0.0102 'outt of shape' can
Y ~ B(30, 0.0237)
(iii) Y : number of faults occuring in 30 days
P(Y  2)
Y ~ Po(4.5) = 1 − P(Y = 0, 1, 2)
 30  30
P(Y  3) = 0.342 = 1 −   0.02370 0.97630 +   0.023710.97629
  0   1
(iv) I: number of faults detected by track in 10 days  30 
+     0.02372 0.97628 
J: number of faults detected by train in 10 days  2 
= 1 − 0.9577...
I ~ Po(1.5) = 0.0423

J ~ Po(0.5)
(iii) C: number of 'out of shape' cans in 30 boxess
K: total number of faults occuring in 10 days C ~ B(720, 0.001)
Since np = 0.72 (  5) use Poisson approximation.
K ~ Po(2)
C ≈ Po(0.72)
P(K = 5) = 0.0361 P(C  2) = 1 − P(C = 0, 1, 2)
  0.722  
= 1 − e −0.72  1 + 0.72 +
Exercise 9.3   2!  

1 (i) Yes, λ = 4 = 1 − 0.96338...


= 0.0366
(ii) Yes, λ = 2.72
4 (i) X ~ B(1000, 0.0038)
(iii) Cannot use approximation. np . 5
(ii) Y ≈ Po(3.8)
  3.82 3.83  
P(Y  4) = 1 − e3.8  1 + 3.8 + +
  2! 3!  
= 0.527

55
5 (i) X ~ B(94, 0.05) (v) Bank: assumptions likely to be valid.
S2 (ii) n large, p small, np  5
Emergency Room: calls not likely to be
independent.
(iii) Y : number of no shows per day in August
3 (i) X ~ Po(0.37)
Y ≈ Po(4.7)
e4.7 4.74 (a) P(X = 2) = 0.0473 (3 s.f.)
(a) P(Y = 4) = = 0.185
4!
(b) P(X  2) = 1- (P(X = 0) + P(X = 1) + P(X = 2))
(b) P(Y  4) = 1 − P (Y = 0, 1, 2, 3)
  4.72 4.73   = 0.0064
= 1 − e −4.7  1 + 4.7  +    +
 2! 3!  
S2 Statistics 2 Answers


(ii) Y : number of days on which there were
= 0.690
more than 2 bacteria
(iv) Z : number of days that there are enough rooms
Y ~ B(30, 0.0064)
Z ~ B(31, 0.690)
P(Y  1) = 0.984 (3 s.f.)
P(Z = 31) = 0.000 010 2
(iii) Z : number of bacteria in 50 ml
6 (i) X ~ B(200,   0.01)
Y ≈ Po(2) Z ~ Po(3.7)
P(X  3) = 0.857 P(Z  8) = 0.0137 (3 s.f.)
(ii) P(X  1)  0.9
(iv) I : number of bacteria in 1000 ml
⇒ P(X = 0)  0.1
0.99n  0.1 I ~ Po(74)
nlog0.99  log0.1
I ≈ N(74, 74)
n  229.1...
230 tickets P(I  90)

= 1 - P(I  90.5)
Exercise 9.4
= 1 - φ(1.918)
1 L : theL :number of faults
the number in 200
of faults in days
200 days
L ~ Po(30) ) = 0.0276 (3 s.f.)
L ~ Po(30) )
λ  15,
SinceSince λ use
15,normal approximation.
use normal approximation. (v) P(Questionable)
L ~ NL(30, 30)
~ N(30, 30)
 49.5  30 30  = P(X  2 and Z  8 and I  90)
49.5
P(L P(L = 50)
50) P  Z= 
 P  Z 30 30  
= 0.0064 × 0.0137 × 0.0276
= P(Z=P(3.560)
Z  3.560)
=0 =0 = 0.000 002 42 (3 s.f.)

4 (i) Meteors are seen randomly.


X : numbe
2 (i) r of calls
X : numbe in 5 minutes
r of calls in 5 minutes
Meteors occur independently.
X ~ Po(3.2)
X ~ Po(3.2)
Meteors cannot occur at the same time.
P(X =P(
1)X==01) = 0(3
.130 s.f.)
.130 (3 s.f.)
The rate at which meteors are seen is constant.
(ii)
Y : number of calls
Y : number in 1 minute
of calls in 1 minute
Y ~ Po(0.64)
Y ~ Po(0.64) (ii) X : number of meteors seen in 1 minute
P(Y =P(1)Y==0.337
1) = 0.337
X ~ Po(1.3)
(iii) 5
= 0.004
5
= 0.004
0.3370.337 38 (338 (3 s.f.)
s.f.) e −1.3 1.31
(a) P( X = 1) = = 0.354
1!
(iv) I : number
I : number of calls
of calls per hour
per hour
I ~ Po(38.4 ) (b) P( X  4)
I ~ Po(38.4 )
SinceSince
λ  15λ  15 = 1 − P( X  3)
I ≈ N(38.4,
I ≈ N(38.4, 38.4)38.4)   1.32 1.33  
= 1 − e −1.3  1 + 1.3 + +
P(I  P(
45)I  45)   2! 3!  
 P 45.5 45.5
 38.43 8.4  = 0.0431 (3 s.f.)
= P  Z=  Z   
 38.4 38.4
= P(Z =P(1.146)
Z  1.146)
= 0.874= 0.874 (3 s.f.)
(3 s.f.)

56
X ~ Po(32), X ≈ N(32, 32)
(iii) 
Y : number of meteors in 10 minutes
4 (i) X ~ Po(32), X ≈ N(32, 32)
Y ~ Po(13)
H0 : λ = 32, H1: λ ≠32
H0 : λ = 32, H1: λ ≠32 S2
 35.5  32 
−13 10 P(X  36) = P  Z  35.5  32 
P(Y = 10) = e 13 = 0.0859 (3 s.f.) P(X  36) = P  Z  32 
10 !  32 
= P(Z  0.619)
(iv) W: number of meteors in one hour, W ~ Po(78) = P(Z  0.619)
= 0.268
= 0.268
Since λ   15, use normal approximation. 26.8%  0.5%, do not reject H0
26.8%  0.5%, do not reject H0
No evidence to suggest that the number
W ≈ N ( 78, 78 ) No evidence to suggest that the number
of faults has chan nged.
P (W  100 ) of faults has chan nged.
 a  0.5  32 
 99.5 − 78  P(X  a)  0.005 ⇒ P  Z  a  0.5  32   0.005
= P Z   (ii) 
P(X  a)  0.005 ⇒ P  Z  32   0.005
 78  

S2 Statistics 2 Answers
32
a 0.5 − 32
= P ( Z  2.434 ) 32 = 2.576 ⇒ a  0.5 = 46.6
a 0.5 32− = 2.576 ⇒ a  0.5 = 46.6
= 1 − 0.9926 32 ⇒ a = 47.1
= 0.0074 ⇒ a = 47.1
H0 rejected for X  48 and X  16 (by symmetry)
H0 rejected for X  48 and X  16 (by symmetry)
(v) M : number of meteors in t minutes
 47.5  32 
M ~ Po(1.3t) (iii) P ( X  48 ) = P  Z  47.5  32 
P ( X  48 ) = P  Z 
 32 
 32 
P(M  1)  0.99 = P ( Z  2.74 )
= P ( Z  2.74 )
= 0.0031
= 0.0031
1 − e −1.3t  0.99 By symmetry, P ( X  16) = 0.0031
By symmetry, P ( X  16) = 0.0031
0.01  e −1.3t Actual significance level is 2 × 0.0031
Actual significance level is 2 × 0.0031
ln 0.01  −1.3t = 0.0062 = 0.62%
= 0.0062 = 0.62%
t  3.54
(iv) 0.0062
Smallest integer value of t is 4. 0.0062
(v) X ~ Po(48), X ≈ N(48, 48)
X ~ Po(48), X ≈ N(48, 48)
P(Type II error) = P (17  X  47 λ = 48 )
Exercise 9.5 P(Type II error) = P (17  X  47 λ = 48 )
 16.5  48 47.5  48 
= P  16.5  48  Z  47.5  48 
1 H  
0: λ = 8, H1: λ  8 = P  48  Z  48 
H0: λ = 8, H1: λ  8  48 48 
P(
H0X : λ=P(2)
8,
X
=H0.0138
: λ=0.0138
12) 8 = P (4.55  Z  0.072)
0.0138 = P (4.55  Z  0.072)
P(X 0.01382) =0.05,
 0.0138 reject H0
 0.05, reject H0 = 1  P(Z  0.072)
The changes have decreased the number = 1  P(Z  0.072)
0.0138 The 0.05,
changes reject
haveH0 decreased the number = 0.471
of accidents.
The changes have decreased the numb er = 0.471
of accidents.
of0 :accidents.
H λ = 2.4, H1: λ  2.4
2 (i) H0 : λ = 2.4, H1: λ  2.4
Stretch and challenge
H0X: λ=4)2.4,
P( = 0.H 1: λ  2.4
221 1 (i) X : no. earthquakes per year, X ~ Po( λ )
(ii) P(X  4) = 0.221
P(
P(X X P( 4)X==0.0959
5) .221
05) = 0.0959 1 2
5 or more
P(X 55)or= more hurricanes
0.0959hurricanes P( X  1) = ⇒ P( X = 0) =
3 3
5X or~ Pomore
(3.6)hurricanes −λ 2
⇒e =
(iii) X ~ Po(3.6) 3
P(Type
X ~ Po(II3.6) error) = P(X  5 λ = 3.6)
P(Type II error) = P(X  5 λ = 3.6) 2
P(Type II error) = P( X  5 λ = 3.6) 3.62 3.62 3 3.63 4  ⇒ λ = − ln = 0.405
= e −3.6  1 +−3.63.6
 + +3.6 +3.6  3.64  3
= e  1 + 3.6 32! 3!3+ 4!4+
.62+ 3.6  Mean = λ = 0.405
−3.6   3.6
= 0.706
= e  1(3+ 3 s. .6
f .)+ + 2! + 3!  4! 
 2! 3! 4! 
= 0.706 (3 s.f.) Standard deviation = λ = 0.637
= 0.706
X : number of taxis in 5 hours (3 s.f .)
− λt
(ii) P( X = x ) = e ( λt )
x
X : number of taxis in 5 hours
X :~ Po(21),
3 X  number
X ~ Po(21),
X  ≈taxis
of N(21, 21)
X  ≈inN(21,
5 hours 21) x!
H : λ = 21,
X 0 ~ Po(21), H :  λ  21
1 ≈ N(21, 21)
X  λ = 0.405..., t = 4
H0 : λ = 21, H1: λ  21
H0X: λ=14)  14.5 − 21
P( 21,= H P1: Zλ 
 21
 14.5 − 21 ⇒ P( X = x ) =
e −0.405× 4 (0.405 × 4) x
P(X  14)  = P Z 
 14.521 − 2121  x!
P(X  14) = P  Z  
= P (Z  −1.418 )  e −1.622 (1.622) x
= P ( Z 21 −1.418 ) =
=
=P ( Z=0.078
0.078 −1.418 ) x!
0.078  0.=05, do not reject H0 P( X  2) = P ( X = 0, 1,  2)
0.078
0.078  0.05, do not reject H0
Not
0.078 enough
 0.05,sufficient
do not reject evidence H0 to prove that  1.6222 
= e −1.622  1 + 1.622 +
the
Not enough sufficient evidence
suspicions are correct.
to prove that  2! 
Not enough sufficient
the suspicions areevidence
correct. to prove that
the suspicions are correct.
  = 0.778 (3 s.f.)

57
−0.405× 4
(0.405 × 4)1 n!
(iii) P( X = 1) = e
S2 = 0.320
1!
(iii) Number of triplets =
3!(n − 3)!
P(Exactly one earthquake over next 3 years) =
n(n − 1)(n − 2)
6
= 0.3203 = 0.0329 1
P (all 3 have the same birthday) =
−0.405t 0 3652
e (0.405t )
(iv) P( X = 0) = = e−0.405t  
0! B ~ B  n(n − 1)(n − 2) ,  1 2 
 6 365 
(v) P( X = 0) for t  16 n(n − 1)(n − 2)
E(B ) =
= e −16 λ + e −17 λ + e −18 λ + e −19 λ + ... 799 350
( ) + (e ) + (e ) + (e )
16 − λ 17 − λ 18 − λ 19
= e− λ + ...
S2 Statistics 2 Answers

16 17 18 19
 
= 
2
+ 
2
+ 
2
+ 
2
+ ... (iv) B ≈ Po  n(n − 1)(n − 2) 
 3  3  3  3  799350 
16
2 (approximation valid if n  159)
GP with a =   ,  r =
2
 3 3 B ≈ Po(1.21)

()
16
2 P (B  1) = 1 − P (B = 0) = 1 − e −1.21 = 0.703 (3 s.f.)
3
S∞ = = 0.004 57 (3 s..f.)
2
1− 3 X : no. of doctors who call in sick per weekend
3

(vi) Since P(T  t ) = P( X = 0) X ~ Po(1.6)


P(T  t ) = e− λt P(X 5 2) 5 0.258
F(t ) = P(T  t ) = 1− e− λt
f(t ) = F′(t ) = −e− λt × −λ = λe− λt P(X  1) 5 0.798
P(X = 2)
P(X = 2  |   X  1) =
n  n! P(X  1)
2 (i) Number of pairs of people =   = 0.258
2  2!(n − 2)!                            = = 0.323 (3 s.f.)
0.798
     =
n(n − 1)
2
N ~ B 
 n(n 1)

3 

Exam focus
 2 365 
1 (i) Errors occur independently and randomly.
 3n(n − 1) 
approx N ~ Po   Errors occur at a constant rate.
 730 
The approximation is valid when np  5 Errors occur one at a time.
3n(n − 1) (ii) X : number of errors per page
 5 ⇒ 3n(n − 1)  3650
730
3n 2 − 3n − 3650  0 X ~ Po(0.85)
−34  n  35
P(X  2) = 0.2093
Since n  0, n  35
(iii) Y : number of errors in 10 pages
(ii) P(N  1) = 1 − P(N = 0)
− 3n( n−1) Y ~ Po(8.5)
= 1− e 730

−3n( n−1) P(Y = 10) = 0.110


1− e 730
 0.5
−3n( n−1) (iv) k = 4
e 730
 0.5 I : number of errors in 30 pages
3n(n − 1) I ~ Po(25.5)), I ≈ N(25.5, 25.5)
−  ln 0.5
730
 30.5 − 25.5 
3n(n − 1) P(I  30) = P  Z  
−  − ln 2  25.5 
730
= P(Z  0.9901...)
3n(n − 1) = 730 ln 2
= 0.839 (3 s.f.)
3n2 − 3n − 730 ln 2  0
n = 13.5 or 12.5
n = 13 or 14

58
2 (i) (a) X : number of butterflies in 1 minute
−4.25 n
(b) e 4.25  e 4.25
−4.25 n+1

X ~ Po(2.1)

n! (n +1)! S2
(n + 1)! 4.25n +1

P(X  2) = 0.620 (3 s.f.) n! 4.25n
n + 1  4.25
(b) Y : number of butterflies in 5 minutes n  3.25
Y  Po(10.5) Smallest value is n = 4
 10.58 10.59 10.510  4 (i) Y : number of calls in 5 minutes
P(8  Y  10) = e10.5   +    +  
 8! 9! 10! 
= 0.342 (3 s.f.) Y ~ Po(9)

(ii) I: number of butterflies in 1 hour P(Y  3) = 0.979 (3 s.f.)

S2 Statistics 2 Answers
I ~ Po(126),, I ≈ N(126,126)
(ii) Z : number of calls in 48 hours
 129.5 126 
P(I  130) = P  Z   Z ~ Po(5184), Z ≈ N(5184, 5184)
 126 
   = P(Z  0.3118...)  4999.5 − 5184 
P(Z  5000) = P Z  
  = 0.378 (3 s.f.)  5184 
= P(Z  −2.5625)
(iii) Events do not occur singly.
= 0.005 20
(iv) S: number of single butterflies, S ~ Po(1.7)
T : number of butterflies in pairs, T ~ Po(0.2)
5 (i) Z = X +Y
Z ~ Po(11)
P(no more than 3 total)
P(X = 2 and Y = 8   X + Y = 10 )
= P(S  3 and T = 0) OR P((S = 0 or 1) and T = 1))

P(X = 2 and Y = 8)
= 0.9068 × 0.8187 + 0.4932 × 0.1637 =
P(X + Y = 10)
= 0.823
0.147 × 0.130
= = 0.161 (3 s.f.)
0.119
3 (i) X : number of faulty laptops
3
(ii) P( X = r ) = P( X = r + 1)
X ~ B(850, 0.005) 4
e −4 4r 3 e −4 4r +1
X ~ Po(4.25) =
r! 4 (r + 1)!
p small, n large, np  5 (r + 1)! 3 4r +1
= × r
r! 4 4
(ii) (a) P(X  2) = 1 − P(X = 0, 1, 2) 4(r + 1) = 3 × 4
  4.252   4r + 4 = 12
= 1 − e −4.25  1 + 4.25 +
  2!   r=2
= 0.796 (3 s.f.)

59
S2 10 Continuous random (ii) y

variables 4k

Exercise 10.1
1
1 (i) No, area under the line is ×2×2=2
2
(ii) Yes k

(iii) No, p( x )  0 for x  1


5 x
S2 Statistics 2 Answers

(iv) No, area under lines .1 and/or P(x = 1) has


1
2 values A= • 3 • 3k + 3k = 1
2
15
2 (i) 
x can take on any value in the interval    k =1
2
0  x  10 2
  k=
15
(ii) 0  x  10

(iii) P(0  x  5) = 0.5 (iii) f(x)


2
(iv) f(x)
1
0.1

0 1 x

∫ kx (x − 1)dx = 1
1
A=
0

k ∫ ( x − x )dx = 1
1
2

0
0 2 4 6 8 10 12 1
 3 2
t (min)
k  x  −  x  = 1
3 2 0
 
0.1

0  x  10 k  1   −   1 = 1
(v) f( x ) =   3 2
 0 otherwise 1
− k =1

6
k = −6
3 (i) f(x)
4k
(iv) y

10k

0 1 2 3 4 x

1
A= • 4 • 4k = 1
2 2k
    8k = 1
1
   k =
8 –1 0 3 x

1
A= • 4 • 8k + 4 • 2k = 1
2
24k = 1
1
k=
24

60
(v) f(x) (ii) y

1
1 S2
0.5

0
0 0.5 1 1.5 2 x

(iii) P(1  X  1.5)


1.5


3
0 1 2 3 4 5 6 x = x (2 − x )dx
1 4
1.5
3 2 x 
3


5
A= kx 2dx = 1 = x −  
4  3 1

S2 Statistics 2 Answers
2
5
 3 3  1.5   2 1  
3 3
k x  = 1 =   1.52 −    −  1  −    
 3 2 4  3   3
 3 3 11
k  5  −  2  = 1 =
32
≈ 0.344  (3  s.f.)
3 3
39k = 1

4
5 (i) k dx = 1
k= 1 1 x2
39
k∫
4
x −2dx = 1
(vi) f(x) 1
4
 x −1 
2 k  =1
 −1 1
4
1
k −  = 1
 x 1
1
 1 1 
k − −  −   = 1
 4  1 
3
1 x k  =1
 4

∫ kx (1− x )dx = 1
1
A= 2 4
k=

0
3
k ∫ ( x − x )dx = 1
1
2 3

0 (ii) y

 3  4 1 1.5
k  x  −  x  = 1
3 4 0 1
 3 4
k 1   −   1  = 1 0.5
3 4
0
1
k =1 0 0.5 1 1.5 2 2.5 3 3.5 4 x
12
k = 12

4
4 1
(iii) P( X  2) =
3 2 x2
∫ kx (2 − x )dx = 1
2
4 (i) A= 4
0 4  1
= −
3  x  2
k ∫ (2x − x )dx = 1
2
2

0 4  1  1 
= −  −  − 
 x3 
2
3  4  2 
k x2 − =1
 3 0 =
1
3
 23 
k 22 −  = 1
 3
4
k =1
3
3
k=
4

61
∫ (ii)
k
1 P( x  a)
S2 6 (i)
1 x
dx = 1
= P( − a  x  a)
= 2 • P(0  x  a) by symmetry

k −1
x 2 dx = 1 1 1 1
1 When x = a, y = (2 − a ) =   −   a
k 4 2 4
 1
x2  1 1 1
 1  =1  2  −  4 a + 2 
 2  = 2× •a
1  2 
k  
2  x 1 = 1
 1 
2 k −2 1=1 = 1  −   a  • a
 4 
S2 Statistics 2 Answers

2 k =3 1 2
= a  −  a
4
k=3
2 1
9 (i) A = 4k + • 2• k =1
k=9 2
4 5k = 1


1.5
1 1
(ii) P( X  1.5) = dx k = = 0.2
1 x    5
1
= 2 x 1
1.5

1
 m(line) = 5 = − = −0.1 ⇒ a = −0.1
= 2 1.5 − 2 1 2 10

= 0.449 (3  s.f.) y = −0.1x + c


= −0.1 • 6 + c ⇒ b = 0.6
0
7 f(x)
(ii) P(0  x  4) = 0.8
P( 4  x  t ) = 0.15 ⇒ P(t  x  6) = 0.05

y = −0.1t + 0.6
1
0 1 2 3 4 x A= (6 − t )( −0.1t + 0.6) = 0.05
2
1
1 1 1 ( −0.6 t  + 3.6 + 0.1t 2 − 0.6t ) = 0.05
A= × 1 + (c − 1) • = 1 2
2 2 2 0.1t 2 − 1.2t + 3.6 = 0.1
1 1
+ (c − 1) = 1 0.1t 2 − 1.2t + 3.5 = 0
2 4
1 1 t = 5  or 7
(c − 1) =
4 2 Since  t < 6, t = 5
c=3
1 Exercise 10.2

1 1
m(line) = 2 = − → a = −
∫ x3
k
3−1 4 4 1 (i) dx = 1
0 4
1
y =− x +b
4 k
x 4 
1 3   =1
0 = − •3+b ⇒b =  16  0
4 4
k4 =1
16
8 y
k 4 = 16
k = 2
2k

–2 –a 0 a 2 x

1 1
(i) A= • 4 • 2k = 1 ⇒ k =
2 4

62
(ii) Median, m 4 f(x)
1.5 S2
∫ x3
m
dx = 0.5
0 4
m
 x4  1
 16  = 0.5
 0
m4 0.5
= 0.5
16
m4 = 8 0
0 0.5 1 1.5 2 x
m = 1.68  (3 s.f.)

∫ k (x − x )dx = 1 ∫ kx dx = 1
1 2
2 (i) 5 (i) 2

S2 Statistics 2 Answers
0 1
2
 3
k x  = 1
1
 2 6
k  x  −  x  = 1  3 1
2 6 0
 3 3
 
k  1   −   1   = 1 k  2  −  1  = 1
2 6 3 3
 
1
k =1 k 7 = 1
3  3
k =3
k=3
7

m
(ii) 3( x − x 5 )dx = 0.5

0
(ii) E( X ) = x • p( x )dx
m
x 2 x6
3    −   = 0.5 2
3 2
2 6 0
m 2 m 6  1

= x
1

7
x dx

3    −    = 3  x4 
2

 2 6  2 =
7  4  1
m 2  −  m 6 = 1
2 6 6 3  24 14 
=  −   
3m 2   −  m 6 = 1 7  4 4

m 6 − 3m 2 + 1 =  0 3  15 
=
7  4 
45
1 = = 1.61 (3  s..f.)
3 (i) 4k = 1 ⇒ k = = 0.25 28
4
(ii) Median = 3 (iii) Var( X ) = ∫ x p( x ) − µ
2 2

(iii) Mean = 3
= ∫ x
2
2
3  
x dx −  45 
2

2

 Check: 7
1  28 


2
3  x 5   45 
5 2
E( x ) = x • 0.25 =   − 
1 7  5 1  28 
5
 x 2
3  25 15   45 
2
= 0.25   =  −  − −   
 2 1 7  5 5   28 
 52 12  2
= 0.25    −    3  31  45 
2 2 =    −  
7  5   28 
= 0.25 [12] = 0.0742
=3
5 (i)  Area = ( p2 − p) • k = 1
(iv) Var( X ) = ∫ x p( x ) − µ
2 2
k=
1
p2 − p
= ∫ ( x 0.25) dx − 3
5
2 2

1 p2 − p
3 5
(ii) Median =
x  2
= 0.25   − 32
 3 1
 53 1 
= 0.25  −  − 32
 3 3
124  2
= 0.25  −3
 3 
4
=
3

63

3
6 (i) f(x) P(0  x  3) =
6
x (5 − x )dx
S2 (iii)
0 125
3
6  5x 2 x3
=  −  
125  2 3 0
6  5 × 32 33 
0 x =   −  
125  2 3
(ii) y =
81
= 0.648
125

P(Exactly one failure)


S2 Statistics 2 Answers

= 2 C1 × 0.6481 × 0.3521
= 0.456 (3 s.f.)


5

8 (i) Ax 2 dx = 1
25

 −3 
0 1 2 3 4 5 6 7 8 9 10 11 x x 2 
A =1
(iii) y 3
− 
 2  25

 2 
A −  =1
 3 x 3  25
 2  2  2
A − − −     = 1; as r → ∞ , − →0
 3 r 3
 3 253   3 r3
2 
A 0 +  = 1
 375 
0 1 2 3 4 5 6 x 375
A= = 187.5
2
∫ kx (5 − x )dx = 1
5
7 (i)   

m 5
0 375 − 2
(ii) x dx = 0.5
∫ k (5x − x )dx = 1 25 2
5
2
m
0 375  2 
5 − 3 
= 0.5
 5x 2 x3 2  3 x  25
   k   −   = 1
 2 3 0  2  2  1
 3 −   −   −   =
 3 m 3  3 253  375
2
k  5 • 5   −   5   = 1
 2 3 2 2 1
− + =
125 3 m 3 375 375
    k =1
6 2 1
− =−
6 3 m3 375
    k = = 0.048
125
3 m 3 = 750
m 3 = 250

1
6
(ii) P(0  x  1) = x (5 − x )dx 2
0 125 m = (250) 3
1 m = 39.7
6  5x 2 x3
=   −   = $39700
125  2 3 0
6  5 1 ∞ 5


=   −   x • 375x

2
125  2 3  (iii) E( X )  = dx
13 25 2
= = 0.104 ∞


125 375 −
3
= x 2
dx
p(x)
2 25

0.3 375  − 
1
=  −2 x 2 
2   25

0.2 375  2 
= −
2  x  25
0.1
375  2  2  2
= −   −   −  as r → ∞ , − →0
2  r  25   r
0 1 2 3 4 5 x
=
375 
0+
2 
2  25 
P(2  failures) = 0.1042 = 0.0108  (3  s.f.)
= 75
64
= $75000
f(x)
(iv)
∫ ke
1
(iv) P (0  X  1) =
S2
−kx
dx
0
1
0.06 = −e −kx 
0

= −e−k − (−e0 )
= 1 − e −k
1 ek − 1
= 1− = k
ek e


x
(v) P( X  x ) = ke−kx dx
25 median mean x 0
x
= −e−kx 

S2 Statistics 2 Answers
Median is the ‘middle’ value so not influenced 0

by high salaries. = −e−kx − (−e0 )


= −e−kx + 1
High salaries push the mean up significantly.
= 1− e−kx
The company would use the mean to attract
potential employees. 3
3
x2  x3 
2 (i) ∫0 18
dx =  
 54  0
Stretch and challenge 33
    =
k 54
1 (i)         f(x ) =  0 for x  0
ekx     = 0.5

P (3  x  4) = 0.5
∫ ke − kx ∞
=  − e − kx  0
0 ⇒ k = 0.5
                = − e − kr − − e0 ;( ) as r → ∞ ,  e − kr → 0
3
 x2  4

∫ x  18  dx  + ∫ x 2 dx
1
                = 0 − 1( −1) (ii) E( X )  = •
0 3
               ==1 3 4
1
18 ∫ 2∫
1
=  x dx  +   3
x dx

10
(ii) Median = 10 → ke−kx dx = 1 0 3

0 2 1 x 
4
1x 
2 3 4

=      +    
10 1 18  4  0 2  4 3
 − e − kx  =
2
( )
0
1  81 1
=    +   42 − 32
18  4 
(
− e −10 k − − e0 = ) 1
2
4
7
81 7
1 =    +     =   28  or 2.875
− e −10 k + 1 = 72 4
2 i.e 287.5 kg
1
− e −10 k = −
2
81 7
1 (iii) Profit = 50 × + 40 ×
e −10 k = 72 4
2
= $126.25
1
−10k = ln  
 2

k=
1
2
ln ()
−10
k = 0.0693 (3  s.f.)

(iii) P ( X  20 ) =
∫ 0.0693e
20
−0.0693 x
dx

=  − e −0.0693 x  20

( )
= − e −0.0693r − − e −0.0693× 20 as r → ∞ ,  e −0.0693r → 0
−0.0693 × 20
=0+e
= 0.25

65
x
 1 πcos πx  dx Exam focus
S2 3 (i) P(X  x) =
∫ 2 
∫ k (1− x ) dx = 1
1 1

2
1 (i)
x
1 1 0
= π sin πx  1
2  π −1  2
2 k x − x  = 1
 2 0
1 1 
= sin πx − sin  π × −    
2   2  k 1− 1 = 1
 2
1
= [ sin πx + 1] 1
2 k =1
2
(ii) By symmetry E(X) = 0 k =2
S2 Statistics 2 Answers

1 or
1
Var( X ) = 1 x  π cos π x dx − 02

2
2
1
− 2  A= ×  1 × k = 1
2 2
1
2 k=2
1
= π
2 ∫ −
1
2
x 2 cos πx dx
f(x)
1
2
1  2 1 1 

2
= π  x ×   sin πx − 2 x ×   sin π x dx 
2  π −
1 π 
 2
1
1
1  x2 2 1 1 

2
= π  sin πx   −    x × − cos π x − − coss πx  
2 π π π −
1 π  
 2
0 0.5 1 x
1
2
1  x2 2 x 1 1  1
sin πx   −    − cos πx + × sin πx  
= π
2  π π π π π −1
2
(ii) E( X ) =
∫ x ×  2(1 − x ) dx
0
1
 x2 x3 
 1 2

1  2
= π 
π ()
2 1
sin   −    −
π 1 π 
cos + 2 sin  
 = 2   −   
 2
1 1
3 0

2  π 2 π  2π 2 π 2  = 2    −   
   2 3
1
=
 1
( ) 
2
3
 −2 −π 2 1 −π 1 −π   

− sin   −    + cos + 2 sin    2
π  2π 2  
1
 π 1
∫x × 2 (1 − x ) dx −  
2 2 π Var( X ) = 2
    3
 0
1 2
 x3 x4  1
1  1  1
= π    −    2  −  −
2 1 1 
  −    − 2   
2 = 2    −      −   
2  4π π  π   4π π  π    3 4  0  3
1 1 1
1  1 2 1 2 = 2   −    −
= π  −   3 +   −   3   3 4  9
2  4π π 4π π 
1 1
1 1 4 = 2×  −  
= π    −   3  12 9
2  2π π  1
=
1 2 18
=  −   2
4 π 1
1−

∫ 2 (1 − x ) dx
2
1 2 (iii)
SD( X ) = − 0
4 π2
1
1−
 x2  2
= 2  x  −   
(iii) P ( −σ  X  σ ) = 2 × P ( 0  X  σ )  2 0
 1− 1  
1 2 2
−  
1

4 π2
= 2× π cos π x dx  
  
2 1 2
0
= 2 1  −   − 
 − 
1

2
   2 2 
1
= π  sin πx   
4 π2
 
π 0
  1− 2 1 
  1 2   +
= sin π  − 1  2 2
  4 π 2   = 2 1  −     −   
 2 2
= 0.632   

1 1 1 1
= 2 1  −    −   +   −   
 2 2 2 4
1
= 2  
4
1
=
2
66
(iii) P ( X  E( X )) = P ( X  2.53)

a
1
2 (i)
25 4 t
dt = 0.8
2.53
2 S2
1

a
t dt = 0.8
−1
2
=
∫ 2 5
xdx
4 25 2.53
2  x2 
1 =
5  2  2
a
2 t  25 = 0.8
8
4
2  ( 2.53) 22 
2
1
2 a − 2 25  = 0.8 =   −   
4 5  2 2 
2 a − 10 = 3.2
38
2 a = 13.2 = 0.484  using E( X ) = ;  0.480  E( X ) = 2.53
15
a = 6.6
a = 43.56 min (iv) Since P ( X  E( X ))  0.5, the median is greater

S2 Statistics 2 Answers
than the mean.
49


1 ∞
(ii) E(T ) = k
25

49 1
4 t
dt 4 (i) ∫ x dx = 1
2
4


1 ∞
= t dt

2 −4
4 25
k x dx = 1
2
49
1 2 t  ∞
3
=  x −3 
4  3  25 k  =1
 −3  2
1  2 49 2 25 
3 3 ∞
1
=
4  3
 −  
3 
k  − 3  = 1
 3x  2
109
= = 36.3 min ( or 36 min  20 s )  1
k  − 3 −  −
1  1
  = 1;   a s r → ∞ , − 3 → 0
3
 3r  3 × 23   3r
3
1
k 0 +
49
 2 1   109 
Var(T ) =
∫ 25
t ×  dt − 
4 t 3   24 
 = 1

49 3 2 k = 24
1 109 
t 2 dt − 
=
∫  3 

4 24 3
P (2  x  3) =
25
(ii) dx
1 2 t  5
109 
49 2 x4 2
= −
4  5  25  3 
3
 1 
= 24 − 3 
2
 3x  2
1  2 495 2 255   109 
=   −    −  1  1 
4 5 5   3  = 24 −   −   − 
 3 × 33  3 × 23 
= 48.1 min  (3  s.f.)
2
 19 
= 24 
 648 

k
2
3 (i) x dx = 1 19
2 5 = = 0.704
k 27
x 2 
 5  =1 ∞
24
2
k  −  2 = 1
 2
2
(iii) E( X ) =
∫ 2

x4
dx

5 5
k2 = 9
= 24
∫ 2
x −3dx

5 5  x −2 
= 24  
k2 = 9  −2  2
k =3 (ignore k = -3) 1

= 24  − 2 
3  2x  2
2 
(ii) E( X ) =
∫ x  5 x  dx
2
 1
= 24  − 2 −  −
1 
as r → ∞ , −
1
→0
3  2r  2 × 22   2r 2
2  x3 
= 1
5  3  2 = 24 0 + 
 8
2  33 23  =3
=  −   
5  3 3
38
= = 2.53 (3  s.f.)
15

67
S2 11 Linear combinations of (ii)
27
E( X ) = 0 ×
64
+ 1×
27
64
+ 2×
9
64
+ 3×
1
64
random variables = 0.75

Exercise 11.1 (iii) Var( X ) = 02 ×


27 2 27
+1 × + 22 ×
9
+ 32 ×
1
− 0.752
64 64 64 64
1 (i) E( X ) = 0 × 0.1 + 1 × 0.3 + 2 × 0.2 + 4 × 0.4 9
= = 0.5625
= 2.3 16
(ii) Var( X ) = 02 × 0.1 + 12 × 0.3 + 22 × 0.2 (iv) E(2 X + 1) = 2 × 0.75 + 1 = 2.5
+ 42 × 0.4 − 2.32
9
= 7.5 − 2.32 (v) Var( 4 X − 3) = 42 × =9
S2 Statistics 2 Answers

16
= 2.21
(iii) E(2 X ) = 2 × E( X ) = 2 × 2.3 = 4.6
Exercise 11.2
(iv) Var(2 X ) = 22 × Var( X ) 1 (i) E( X ) = 0 × 0.2 + 1 × 0.3 + 2 × 0.5 = 1.3
= 4 × 2.21 E(Y ) = 0 × 0.4 + 1 × 0.5 + 3 × 0.1 = 0.8
= 8.84
Var( X ) = 02 × 0.2 + 12 × 0.3 + 22 × 0.5 − 1.32 = 0.61
2 (i) E( X − 1) = E( X ) − 1 = 5 − 1 = 4 Var(Y ) = 02 × 0.4 + 12 × 0.5 + 32 × 0.1 − 0.82 = 0.76

(ii) Var( X + 2) = Var( X ) = 22 = 4 (ii)


x+y 00.4 10.5 30.1
(iii) E(2 − X ) = 2 − E( X ) = 2 − 5 = −3 0.20 0(0.08) 1(0.1) 3(0.02)

(iv) sd(2 − X ) = sd( X ) = 2 0.31 1(0.12) 2(0.15) 4(0.03)


0.52 2(0.2) 3(0.25) 5(0.05)
(v) E(2 X + 3) = 2 × E( X ) + 3 = 2 × 5 + 3 = 13

(vi) Var(3 X − 2) = 32 × Var( X ) = 9 × 22 = 36 x-y 0 1 3

0 0(0.08) −1(0.1) −3(0.02)


(vii) Var( X ) = E( X 2 ) − µ 2 ⇒ 4 = E( X 2 ) − 5
  ⇒ E( X ) = 9
2
1 1(0.12)    0(0.15) −2(0.03)
2 2(0.2)   1(0.25) −1(0.05)
(viii) E ( X − 2)2  = E( X 2 − 4 X + 4)
= E( X 2 ) − 4E( X ) + 4 x+y 0 1 2 3 4 5
= 9− 4×5+ 4 Prob 0.08 0.22 0.35 0.27 0.03 0.05
= −7
x-y −3 −2 −1 0 1 2
(ix) Var( 4 − 5 X ) = ( −5)2 Var( X )
Prob 0.02 0.03 0.15 0.23 0.37 0.2
= 25 × 22
= 100
(iii) E( X + Y ) = 0 × 0.08 + 1× 0.22 + 2 × 0.35
3 
X : price of TV before GST   + 3 × 0.27 + 4 × 0.03 + 5 × 0.05
= 2.1
E( X ) = 650 Var( X ) = 40
E( X − Y ) = −3 × 0.02 + ( −2) × 0.03 + ( −1) × 0.15
E(1.15X + 20) = 1.15 × 650 + 20
    + 0 × 0.23
3 + 1 × 0.37 + 2 × 0.2
= $767.50 = 0.5
Var(1.15X + 20) = 1.152 × 40
= $52.90 Var( X + Y ) = 02 × 0.08 + 12 × 0.22 + 22 × 0.35
   + 32 × 0.27 + 42 × 0.03 + 52 × 0.05 − 2.12
3
3 27 = 1.37
4 P( X = 0) =   =
 4 64
3 Var( X − Y ) = ( −3)2 × 0.02 + ( −2)2 × 0.03 + ( −1)2 × 0.15
1 1
P( X = 3) =   =
 4 64     + 02 × 0.23 + 12 × 0.37 + 22 × 0.2 − 0.52
 3 1 3
1
27
2 = 1.37
P( X = 1) =       =
 1  4   4  64
2
(iv) E( X + Y ) = E( X ) + E(Y )
 3 1 3 9
P( X = 2) =       = E( X − Y ) = E( X ) − E(Y )
 2  4   4  64
Var( X + Y ) = Var( X ) + Var(Y )
(i) Var( X − Y ) = Var( X ) + Var(Y )
x 0 1 2 3
21 27 9 1
P(X = x)
64 64 64 64
68
2 (i) E(P + Q ) = E(P ) + E(Q ) 1 2 3 4
= 8+3
Var( X1 + X 2 ) = 22 ×
16
+ 32 ×
16
+ 42 ×
16
+ 52 ×
16 S2
= 11 3 2 1
   + 62 × + 72 × + 82 × − 52
16 16 16
(ii) Var(P + Q ) = Var(P ) + Var(Q ) = 2.5
= 5 + 10 (iii)
= 15 X1 – X 2 1 2 3 4

(iii) E(2P + 5Q) = 2 × 8 + 5 × 3 1 0 1 2 3


= 31
2 1 0 1 2
(iv) Var(P − Q) = 5 + 10
3 2 1 0 1

S2 Statistics 2 Answers
= 15
4 3 2 1 0
(v) Var(3P + 2Q ) = 32 × 5 + 22 × 10
= 85
X1 – X 2 0 1 2 3
(vi) Var(P − 3Q) = Var(P ) + ( −3)2 × Var(Q)
4 6 4 2
= 5 + 9 × 10 Prob. 16 16 16 16
= 95

3 (i) E( X 1 + X 2 ) = 250 + 250 = 500 g 4 6 4 2


E( X1 − X 2 ) = 0 × + 1× +2× +3×
16 16 16 16
Var( X 1 + X 2 ) = 52 + 52 = 50g2 ⇒ sd = 50 = 7.07 g
20
= = 1.25
16
(ii) E(Y1 + Y2 + Y3 + Y4 ) = 125 × 4 = 500 g
Var(Y1 + Y2 + Y3 + Y4 ) = 4 × 52 = 100 g 4 6 4 2
Var( X 1 − X 2 ) = 02 × + 12 × + 22 × + 32 × − 1.252
16 16 16 16
sd = 100 = 10 g
15
= = 0.9375 (0.938)
16
4 (i)
x 1 2 3 4 (iv)
2 X1 2 4 6 8
P(X = x) 0.25 0.25 0.25 0.25
1 1 1 1
E( X ) = 1 × 0.25 + 2 × 0.25 + 3 × 0.25 + 4 × 0.25 Prob
4 4 4 4
= 2.5
1 1 1 1
Var( X ) = 12 × 0.25 + 22 × 0.25 + 32 × 0.25 E(2 X1 ) = 2 × + 4× +6× +8×
4 4 4 4
   + 42 × 0.25 − 2.52 =5
= 7.5 − 2.52
1 1 1 1
= 1.25 Var(2X 1 ) = 22 × + 42 × + 62 × + 82 × − (52 )
4 4 4 4
=5
(ii)
+ 1 2 3 4
(v) (a) T (b)
F (c)
T
1 2 3 4 5
(d) F (e)
T (f)
F
2 3 4 5 6
3 4 5 6 7 5 (i) T =W +B
E(T ) = 12 + 25 = 37 min
4 5 6 7 8
Var(T ) = 1.22 + 1.62 = 4
T ~ N(37, 4)
x1 + x 2 2 3 4 5 6 7 8
40 − 37 
1 2 3 4 3 2 1 (ii) P(T > 40) = P  Z >
Prob.  2 
16 16 16 16 16 16 16
= P( Z > 1.5)
1 2 3 4 = 1 − 0.9332
E( X1 + X 2 ) = 2 × +3× +4× +5×
16 16 16 16 = 0.0668
3 2 1
 + 6 × +7× +8×
16 16 16
=5

69
(iii) Z(0.98) = 2.054 0.098 (ii) B − H ~ N(12.4 − h, 0.0017)
S2 2.054 =
x − 37 P(B − H  0)  0.999
2 0.02
0 − (12.4 − h)
3.090 =
x = 41.108 min 37 x 0.0017

h = 12.53 cm

∴ 98% of all trips take 41.108 min or less 0.999

Time she should leave is 8:30 am − 41.108 min


= 8:30 am − 42 min
= 7: 48 am
? 0
S2 Statistics 2 Answers

6 (i) P ( F − S  6) F − S ~ N(7, 19.62 )


9 N ~ N( 45, 8 )  L ~ N(55, 10 )
2 2

E(C ) = 4 × 45 + 3 × 55
= P(F − S  6) + P(F − S  −6)
= 345
6− 7 −6 − 7 
= PZ  + PZ  Var(C ) = 42 × 82 + 32 × 102
 19.6   9.6 
19 345 390
= P ( Z  −0.051) + P ( Z  −0.663) = 1924
= 0.5203 + (1  −   0.7464) C ~ N(345,1924)
= 0.774  (3  s.f.) P(C  390)

 390 − 345 
= P Z  
 1924 
= P ( Z  1.026)
= 1 − 0.8475
–6 67
= 0.1525 (0.153)
(ii) The standard deviation would be less.
i.e approx. 15.3% of applicants will get an interview
Pairing fathers and sons would mean less
variation.
Exercise 11.3
7 W ~ N(750, 2.62 ) 1 (i) X + Y + Z ~ N(35, 14)
(i) W1 + W2 ~ N(1500,  2 × 2.6 ) 2
(ii) X − Y − Z ~ N( −11, 14)
P(W1 + W2  1495 g)
 (iii) E(2 X + 3Y + Z ) = 2 × 12 + 3 × 8 + 15
1495 − 1500 
=PZ   = 63
 13.52 
= P ( Z  −1.360 ) Var(2 X + 3Y + Z ) = 22 × 4 + 32 × 1 + 9
= 1 − 0.9131 = 34
= 0.0869 2 X + 3Y + Z ~ N(63, 34)

(iv) E( X − 2Y + 5 Z ) = 12 − 2 × 8 + 5 × 15
(ii) W1 − W2 ~ N(0, 13.52)
= 71
= P(W1 − W2  5) + P(W2 − W1  5)
Var( X − 2Y + 5 Z ) = 4 + 22 × 1 + 52 × 9
= 2 × P(W1 − W2 > 5)
= 233
 5−0 
= 2 × P Z   X − 2Y + 5 Z ~ N(71,  233)
 13.52 
= 2 × (1 − 0.9131) 2 T = A + B + C + D
= 0.174  (3  s.f.) E(T ) = 62 + 70.4 + 57.6 + 52.8
= 242.8 s (= 4 min 2.8 s)
–5 5 Var(T ) = 0.212 + 0.122 + 0.182 + 0.352
= 0.2134
8 H ~ N(12.5, 0.042 )  B ~ N(12.4, 0.012 )
T ~ N(242.8,  0.2134)
(i) P(B − H  0)   B − H ~ N( −0.1,  0.0017)
 242 − 242.8 
 0 − ( −0.1)  P(T  242) = P  Z  
= P Z    0.2134 
 0.0017 
= P( Z  2.425) = P( Z  −1.732)
= 0.992 (3
3  s.f.) = 1 − 0.9584

– 0.1 0 = 0.0416
242 242.8

70
3 (i) W : Weight of truck + 10 containers (ii) P = (E1 + E2 > W1 + W2 + W3 + W4 + W5 )
E(W ) = 2450 + 10 × 42 = P ( E1 + E2 − (W1 + ... + W5 ) > 0 ) S2
= 2870 kg E(E1 + E2 − (W1 + ... + W5 )) = 2 × 1080 − 5 × 450
Var(W ) = Var(T + C1 + C 2 +  + C10 ) = −90
= 85 + 10 × 3
2 2 Var(E1 + E2 − (W1 + ... + W5 )) = 2 × 452 + 5 × 702
= 7315 kg2 = 28550

W ~ N(2870, 7315)

 3000 − 2870 
(ii) P(W  3000) = P  Z  
 7315 
–90 0
= P( Z  1.520)

S2 Statistics 2 Answers
= 1 − 0.9357 P ( E1 + E2 − (W1 + ... + W5 ) > 0 )

= 0.0643  0 − ( −90) 
= P Z  
= 6.43%  28550 
= P( Z  0.533)
(iii) W ~ N(2450 + 10 x , 7315)
= 1 − 0.7029
 x : average weight of one container
= 0.297 (3  s.f.)
Z (0.995) = 2.576
3000 − (2450 + 10 x )
2.576 =
7315 0.5% Stretch and challenge
220.32 = 550 − 10 x
1 M ~ N( µ , σ )   F ~ N( µ − 15, σ )
10 x = 329.68
P(M1 + M2  F1 + F2 + F3 ) = 0.25
x = 33.0 kg (3  s.f.) 2450+10x 3000
P(M1 + M2 − (F1 + F2 + F3 )  0) = 0.25

E(T ) = 2 × 0.1 + 3 × 0.05 + 0.15 E(M1 + M2 − F1 − F2 − F3 ) = µ + µ − ( µ − 15)
4 (i)
= 0.5 mm   − ( µ − 15) − ( µ − 15)
= 45 − µ
Var(T ) = Var( A1 + A2 + B1 + B2 + B3 + C1 )
Var(M1 + M2 − F1 − F2 − F3 ) = 5σ 2
= 2 × 0.022 + 3 × 0.01
12 + 0.032
 0 − ( 45 − µ ) 
= 0.002 mm2 ∴ P Z   = 0.25
 5σ
T ~ N(0.5,  0.002)
µ − 45
= 0.674
 0.6 − 0.5  µ = 8σ 2 5σ
(ii) P(T  0.6) = P  Z   µ − 45 = 1.5071σ
 0.002 
= P( Z  2.236) 8σ 2 − 45 = 1.5071σ
= 1 − 0.9873 8σ 2 − 1.5071σ − 45 = 0
= 0.0127 σ = 2.47 ((3  s.f.)
= 1.27% µ = 48.7 (3  s.f.)

5 
C : amount of coffee dispensed in one serve 0.25
2
C ~ N(210,  3 )

C1 + C 2 + C 3 + C 4 ~ N(840,  62 ) (45 – µ) 0

840 850
2 (i) T : total weight
T ~ N(750, 500)
P(C1 + C 2 + C 3 + C 4  850) P(T > 800)

850 − 840 
= P( Z  ) 800 − 750 
6 = P Z >  750 800
 500 
= P( Z  1.667)
= P( Z > 2.236)
= 1 − 0.9522
= 1 − 0.9873
= 0.0478
= 0.0
0127
6 T = W1 + W2 + W3 + W4 + W5 + E1 + E2

(i) E(T ) = 5 × 450 + 2 × 1080


= 4410 MB
Var(T ) = 5 × 702 + 2 × 452
= 28550 MB2
T ~ N( 4410, 28550)
71
(ii) M: amount (in kg) macadamias in 2 scoops P(10 400  T  10 600)
S2 M ~ N(0.3,  0.0002)  10 400 − 10 430 10 600 − 10 430 
= P Z 
 7504 7504
C : amount (in kg) of cashews in 3 scoops
= P( −0.346  Z  1.962)
C ~ N(0.45,  0.0003) = (0.6353 − 0.5) + (0.9751 − 0.5)
P(12M  9C ) = 0.610  (3  s.f.)

= P(12M − 9C  0) E(12M − 9C ) (ii)      P( A  50P ) = P( A − 50P  0)
 0 − ( −0.45)  = 12 × 0.3 − 9 × 0.45        E( A − 50P ) = 4350 − 50 × 76
= P Z  
 0.0531  = − 0.45                          = 550 kg
S2 Statistics 2 Answers

= P( Z  1.953) Var(12M − 9C )   Var(( A − 50P ) = 322 + 502 × 92


= 0.975 (3  s.f.) = 12 × 0.0002 + 9 × 0.0003
2 2
                         = 203524 kg2
= 0.0531 (3  s.f.)  0 − 550 
P( A − 50P  0) = P  Z  
 203 524 
Exam focus                          = P( Z  −1.219)
1 P ~ N(14, 1.8 )2
W ~ N(7.8,  0.6 ) 2                          = 0.889 (3  s.f.)

F = 0.8P + 0.2W
E(F ) = 0.8 × 14 + 0.2 × 7.8
= 12.76
Var(F ) = 0.82 × 1.82 + 0.22 × 0.62 0 550

= 2.088  (2.09)
4 (i) Var( S ) = Var( X1 + X 2 + X 3 + X 4 )
SD(F ) = 2.088
= 4 × 1.4
= 1.44  (3  s.f.)
= 5.6

2 CD  ~ N(15,  0.22 )  Case C ~ N(80, 1.42 ) (ii) Var(T ) = Var( 4 X + 3)


T = CD1 + CD2 + ... + CD12 + C1 + ... + C12 + B1 + ... + B12 = 42 × Var( X )

E(T ) = 12 × 15 + 12 × 80 + 320 = 42 × 1.4
= 1460 g = 22.4
Var(T ) = 12 × 0.22 + 12 × 1.42 (iii) E( S − T ) = E ( X1 + X 2 + X 3 + X 4 − ( 4 X + 3))
= 24 g2 = 3.5 + 3.5 + 3.5 + 3.5 − ( 4 × 3.5 + 3)

= −3
 1470 − 1460 
P(T  1470) = P  Z   Var( S − T ) = Var( X1 + X 2 + X 3 + X 4 ) + Var( 4 X + 3)
 24

= 4 × 1.4 + 42 × 1.4
= P( Z  2.041) = 28

= 1  −   0.9793
= 0.0207

3 (i) P : weight of passenger P  ~ N(76,  92 )


A : weight of plane empty  A ~ N( 4350,  322 )
T : total weight of plane + 80 passengers
T = A + P1 + P2 + ... + P80
E(T ) = 4350 + 80 × 76
= 10 430 kg
Var(T ) = 322 + 80 × 92
= 7504 kg2

10 430

72
12 Sampling (c) 
Many ways, including cluster sampling
S2
cities first, then suburbs, then randomly
Exercise 12.1 calling people (follows-up by post if
1 • List the students 1–120. needed).

• Generate 3-digit random numbers on your (iii) (a) All cars in the city.
calculator. (b) 
Only one part of the city, on only one
• Choose 3 numbers ignoring any outside the morning, is being considered.
range or repeats. (c) 
Go to licence register and choose every
2 (i) Because each number 1–8 is not equally likely. 10th car on the list until they have enough

S2 Statistics 2 Answers
cars.
(ii) – Throw one die.
(iv) (a) All students at the school.

If even pick from 1–4, if odd pick from 5–8.
(b) 
Only morning, only students who catch

Throw again and choose. the bus, not a random selection.
1
(c) 
Randomly choose 2 or 3 students in each
2
Even 3 class of the school to give the survey to.
4
1 (5) (v) (a) Every voter in the town.
Odd 2 (6)
3 (7) (b) 
Only people available then can be
4 (8)
involved, only those home at the time

Repeat process for 2nd choice. (excludes people at work).


Ignore repeats on 2nd choice. (c) 
Choose every 100th name on the list, post
survey and follow up with calls.
(iii) Each person has an equally likely chance of
being chosen.
Stretch and challenge
3 (i) (a) Shoppers at the store. 1 
Student’s own research.

(b) Only on one day, bias in choosing people.


Exam focus
(c) 
Give the questionnaire to the 1st, 5th,
1 (i) 
Anyone in the class with surnames starting
10th person to enter the shop every hour
with less frequent letters (Q, Z, etc) will have
for a whole week.
less chance of being chosen.
(ii) (a) All people in the country.
(ii) Number all students, e.g. 1–30.
(b) 
Many! Only viewers, only people

Use the calculator to produce random 2-digit
available to watch at 7 pm, only a certain
numbers. Ignoring numbers outside the range
demographic watching, only those with
and any repeats, choose 4 numbers.
strong opinions will call, etc.

73
S2 13 Hypothesis testing and
confidence intervals using
the normal distribution
Exercise 13.1
1 (i) Sample Mean Variance Standard deviation Median Minimum Maximum
1+ 3
1, 3 =2 1 1 2 1 3
2
S2 Statistics 2 Answers

1, 4 2.5 2.25 1.5 2.5 1 4


1, 8 4.5 12.25 3.5 4.5 1 8
3, 4 3.5 0.25 0.5 3.5 3 4
3, 8 5.5 6.25 2.5 5.5 3 8
4, 8 6 4 2 6 4 8
Mean of samples 4 4.33 1.83 4 2.17 5.83
Population mean 4 6.5 2.55 3.5 1 8

(ii) The mean 2 x =


∑ = 996 = 124.5 min
x
n 8
 2

(iii) When the population is symmetric (like the s = 1  128 440   −   (996)  = 634 min2
2
8 − 1 8 
normal distribution) the median will be an

or
unbiased estimator.
8  128 440 
s2 =   −  124.52  = 634 min2
(iv)
2
× 6.5 = 4.3 = E(V ) 8 − 1  8 
3
∑x  3492
2
3 x = = 145.5 s
s2 = n   −   x 
2
(v) 24
n − 1 n 
1  (3492)2 
s2 =  508 397  −   = 13.5 s2
 ∑ x2  n ∑x 24 
2

= n  24 − 1 
  −    
n − 1 n  n − 1 n  1  (102)2 
4  y = x − 200 s y2 =  1143.5  −  
12 
Let
12 − 1 
(∑ x )
2

=
∑x 2

 −   ∑ y = 102 , ∑ y 2
= 1143.5
n −1 n(n − 1) = 25.1 m2
102
 (∑ x )  y= = 8.5 m s y = 25.1
2

= 1  ∑ x 2  −   12
n − 1

n  x = y + 200 = 208.5 m = 5.01 m
s y = s x ⇒ s x = 5.01 m
Exercise 13.2
1 (i)
Numbers drawn 1, 2 1, 3 1, 4 1, 5 2, 3 2, 4 2, 5 3, 4 3, 5 4, 5
Mean 1.5 2 2.5 3 2.5 3 3.5 3.5 4 4.5
Probability 0.1 0.1 0.1 0.1 0.1 0.1 0.1 0.1 0.1 0.1

Mean 1.5 2 2.5 3 3.5 4 4.5


Prob. 0.1 0.1 0.2 0.2 0.2 0.1 0.1

y
0.25
probability

0.2
0.15
0.1
0.05

0.5 1 1.5 2 2.5 3 3.5 4 4.5


mean

(ii) Numbers drawn 1, 2, 3 1, 2, 4 1, 2, 5 1, 3, 4 1, 3, 5 1, 4, 5 2, 3, 4 2, 3, 5 2, 4, 5 3, 4, 5


Mean 2 2.33 2.67 2.67 3 3.33 3 3.33 3.67 4
Probability 0.1 0.1 0.1 0.1 0.1 0.1 0.1 0.1 0.1 0.1

Mean 2 2.33 2.67 3 3.33 3.67 4


74 Prob. 0.1 0.1 0.2 0.2 0.2 0.1 0.1
 21 − 22 
0.2 ( )
P X < 21 = P  Z <
 2.6

30 
S2
probability

0.1 = P( Z < −2.107)
= 1  −  0.9824
= 0.0176
2 4
mean
(ii) No, since the sample size was large enough
1  +   2  +   3  +   4   +  5 (.30) by the CLT the sample means are
(iii) x = =3
5
approximately normally distributed.
12   +   22   +   32   +   42   +  52
s2 =   −   32 = 2
5 16 
6 X  ~ N( 4, 16) X  ~ N  4, 

S2 Statistics 2 Answers
 n
(iv) 1.5, 2, 2.5, 2.5, 3, 3, 3.5, 3.5,  4,  4.5

x=
30
=3
(i) ( )
E X =µ=4
10
Var ( X ) =
16
97.5 (ii) = 0.16 ⇒ n = 100
s2 = − 32 = 0.75 n
10
(iii) (i) Would be the same.

The mean is the same as the population mean.
(ii) Would also be the same.

The variance is less than the population
variance. 7 
X: weight of a packet of chips.
 1.52 
(i) X  ~ N  150, 
(v) 2, 2.33, 2.67, 2.67, 3, 3, 3.33, 3.33, 3.67, 4  15 
30  
x= =3
10
93.3 1
( 
)
P X < 149 = P  Z <
149 − 150 
1.5 
s2 =   −   32 =  15 
10 3
= P( Z < −2.582)

The mean is the same as the population mean. = 1  −  0.9951
= 0.0049

The variance is less than the population
variance (ii) No, the sample size was not enough to
2 X  ~ N(50, 10 ) 2 guarantee that X is approximately normal.

 102  8 X ~ B(20,  p)   E( X ) = 20 p
X  ~ N  50, 
 125 
  Var( X ) = 20 p(1 − p)
(
P X > 52 = P  Z >
 10 )
52 − 50 

( )
E X = µ = 20 p = 4.8 ⇒ p = 0.24
 125 
Var ( X ) =
2
σ 20 × 0.24 × (1 − 0.24 )
= = 0.304
= P ( Z > 2.236) n 12

= 1  −   0.9873
= 0.0127
9 (i) ( )
E X = 3.5

35
3 Var X = ( )
σ2
= 32 ( )
Var X = 12 = 7 = 0.0583
50 120
45
σ = 1440
2
 
σ = 37.9  3.7 − 3.5 
(ii) ( )
P X > 3.7 = P  Z >
7 

4 
4 X  ~ Po( 4) X ≈ N  4,   120 
 70 
  = P( Z > 0.828)
 3.5 − 4 
(
P X < 3.5 = P  Z < ) 4 
 = 0.204

 70  (iii) T : total score of 50 rolls

= P( Z < −2.092) E(T ) = 50 × 3.5 = 175
= 1  −   0.9818 35
Var(T ) = 50 × = 145.83
= 0.0182 12
 
5 (i) X: length of snapper in the lake P(T < 150) = P  Z < 149.5 − 175 
 145.83 

 2.62 
X ≈ N  22,  = P( Z < −2.112)
 30 
= 0.0173
75
Exercise 13.3
S2 ∫
10

10  (i) kxdx = 1
0
1 (i) H0 : µ = 58 , H1: µ > 58
10
 2
k x  = 1  82 
 2 0 (ii) Under H0 ,  X  ~ N  58,  
 80 
 2 
k 10 − 0  = 1
 2 
50k = 1

k= 1
50
5%

OR
S2 Statistics 2 Answers

55 56 57 58 59 a 60 61
1 1 Score on test
× 10 × 10k = 1 ⇒ k =
2 50 a − 58
= 1.645
f(x) 8
80
10k 8
a = 1.645 × + 58 = 59.5
80

(iii) 64% is in the rejection region, so reject H0 and


10 x
conclude that here is evidence that the mean

∫  
10
(ii) E( X ) = x ×  1 x  dx has increased.
0  50 
1 x 
3 10 2 H0 : µ = 50 , H1: µ < 50
=  
50  3  0
(i) 0.02
1 10  3
=  − 0
50  3  (ii) Type II error
20
= 3 
X : light bulb intensity
3
10
1  20
2 (i) H0 : µ = 1720 , H1: µ < 1720
Var( X ) =
∫ 0
x2 ×  x dx −  
 50   3
10 2  90 
2
1  x4  20 (ii) X ≈ N  1720, 
= −   30 
50  4 0  3 
2
1  10 4  20
= − 0 −  
50  4   3
50
0
=
9
1703 1720
 50 
(iii) X ≈ N  20 ,  9   
 3 80 
( )
P X < 1703 = P  Z <

1703 − 1720 
90 
   30 
7.2 − 20 3 

(
P X > 7.2 = P  Z > ) 5
 = P( Z < −1.035)
  = 0.150
 72 

= P( Z > 2.024) Since 0.150 > 0.05, accept H0

= 0.0215

There is insufficient evidence to conclude that
11  X : no. nests per 10 m2 the intensities are lower.
X  ~ Po(0.4) 4 X  ~ N(27500,  40002 )
0.4 
X ≈ N  0.4, 
 50  (i) H0 : µ = 27500, H1: µ > 27 500

 
 0.25 − 0.4   4000 
2

( )
P X < 0.25 = P  Z <
0.4 
 (ii) X  ~ N  27500,
 15 

 50 

= P( Z < −1.677)
= 0.0468

27 500 28 630

76

(
P X > 30 450 ) S2
 
30 450 − 27500 
= P Z 
 4000 
 15  0.395 0.436

= P( Z  2.856)  
 0.395 − 0.436 
= 0.0
0022 ( )
P X  0.395 = P  Z 
0.0343 


 120 
Since 0.00 22  0.05, reject H0
= P( Z  −2.426)
There is evidence at the 5% level that the mean = 0.0077

S2 Statistics 2 Answers
lifetime has increased.

Since 0.00762  0.005, accept H0.
5 X  ~ N(355, 522 )

There is insufficient evidence at the 1% level
 522 
X  ~ N  355, 
 25  to conclude that the new drug has changed
the level of the hormone.

(ii) No, because the sample size is large so by


the CLT the sample mean distribution is
approximately normal.
344 355
8 X
 : weight of cereal
H0 : µ = 355, H1: µ ≠ 355

  X  ~ N( µ , 152 )

(
P X < 344 = P  Z <
 )344 − 355 

 152 
X  ~ N  µ , 

52  16 
25 
= P( Z < −1.058)
(i)      H0:  µ = 375, H1: µ   375
= 0.145
z (0.975) = 1.96
 Since 0.145 > 0.025, accept H0 x − 375 2.5%
1.96 =
15
6 
X: level of protein 16
 22  x = 382.35 375
(i) X  ~ N  12.5,   
 25 
(ii) (
P(Type II error ) = P X < 382.35 | µ = 385 )

(
P X > 13.2 ) 2%
 
  382.35 − 385 
13.2 − 12.5  = P Z 

=P Z >  15 
 2  2.5 13.2  16 
 25  = P( Z  −0.707)
= P( Z > 1.75) 382.35 385
= 0.240
= 0.0401
= 4.01% (iii) z(0.025) = −1.96
382.35 − µ
−1.96 =
(ii) (
P( Type II error ) = P X < 13.2 | µ = 13.5 ) 15
16 0.025
  µ = 389.7
13.2 − 13.5 
= P Z  µ  389.7 µ
 2  382.35
 25 
= P( Z  −0.75) (iv) P(Type I error) = 0.025
= 0.227 (3
3  s.f.) 
P(at least 1 Type I error ) = 1  −  P (no errors)

7 (i) X : level of hormone = 1  −   0.9752


H0: µ = 0.436, H1 : µ ≠ 0.436 = 0.0494  (3  s.f.)

1  ( 47.4)2  9 
X : number of patients who suffer an allergic reaction
s2 =  22.8   −  
120 − 1  120 
(i) X  ~ B(90,  0.045)
= 0.0343
X ≈ Po( 4.05)
0.0343 
Under H0 ,  X ≈ N  0.436, 
 e −4.05 × 4.054
 120  P( X = 4) =
4!
47.4
x= = 0.395                = 0.195 (3  s.f.)
120 77
(ii) H0:  p = 0.045,  H1:  p  0.045 k − 18
S2 X  ~ B( 400,  0.045) ≈ N(18, 17.19) 18
= 1.645

P( X  25) = P( X  24.5) k = 24.979...



Rejection region is X  26 (applying the
continuity correction)
 
(iii) P( X  26)   = P  Z  25.5 − 18 
 18 
18 25

 24.5 − 18  = P(Z  1.768)


= P Z  
 17.19  = 0.0385
S2 Statistics 2 Answers

= P( Z  1.568) = 3.85%
= 0.0584
(iv) λ = 0.8 × 30 = 24

Since 0.0584  0.05, accept H0 X  ~ Po(24)    X ≈ N(24,  24)

 P(Type II error)
X  ~ B 270,  
1
10  (i)
 2
= P( X < 26 | λ = 24)
X ≈ N(135,  67.5)
 25.5 − 24 
= P Z < 
 24 
1% = P( Z < 0.306)
= 0.620 24

135 k
This means that if λ = 24, the probability is

k − 135 0.620 that we will conclude that sales have
= 2.326
67.5
increased when in fact they have not.
           k = 154.11...
∴ X  155
Exercise 13.4
(ii) P(Type I error) = P( X  155) 1 (i) x = 63.8
154.5 − 135 
= P Z  99% CI:
 67.5 
= P( Z  2.373) 34
63.8 ± 2.576 ×
= 0..0088 10

(iii) P(Type II error) 63.8 ± 4.75

= P( X  154  p = 0.58) [59.1, 68.6]



X  ~ B(270,  0.58) 34
(ii) 2.576 ×  0.4
X ≈ N(156.6,  65.772) n
2.576 × 34  0.4 n
37.551...  n
8813...  n
∴ n  8814
154 156.6

 154.5 − 156.6  930.4


= P Z   2 (i) x= = 11.63
 65.772  80
= P( Z  −0.259) 1  (930.4)2 
s2 =  11 024.88 − = 2.59
= 0.398  (3  s.f.) 80 − 1  80 

(ii) 90% CI:
11  (i) H0:  λ = 18 ,  H1:  λ > 18
2.59
11.63 ± 1.645 ×
(ii) X  ~ Po(18),   X ≈ N(18,18) 80
11.63 ± 0.296

5% [11.3, 11.9]

2.59
(iii) 2 × 1.645 ×  0.4
18 k n
13.24...  n
175.2...  n
∴ n  176
78
3 s = 2.6,  n = 90,  z (98%) = 2.326 7 pˆ = 0.307
2.6 95% CI:
S2
Width = 2 × 2.326 ×
90
0.307(1 − 0.307)
= 1.27 (3  s.f.) 0.307 ± 1.96
500
18 0.307 ± 0.0404
4 (i) pˆ = = 0.4
45 [0.267, 0.347]

95% CI for p:
0.4(1 − 0.4) 
Since 0.288 (or 28.8%) is in the CI, and the probability
0.4 ± 1.96
45 is 95% that this interval encloses the population
0.4 ± 0.143... mean, there is no evidence of a significant change in

S2 Statistics 2 Answers
[0.257, 0.543] (3  s.f.) market share.

0.4(1 − 0.4) 8 (i) x = 55


(ii) 1.96 ×  0.1
n 
94% CI:
0.24 0.1 15
 55 ± 1.881 ×
n 1.96
15
2
0.24  0.1  55 ± 7.285

n  1.96 
2 [ 47.7, 62.3]
0.1 
0.24 ÷  n
 1.96 
(ii) There is a probability of 0.94 that this interval
92.198...  n
encloses the true population mean µ .
∴n  93
3
(iii) Increase the sample size. 
or
Reduce the level of confidence. If we repeated the survey a large number of
2.35 + 4.45 times, 94% on average of CIs we constructed
5 (i) x= = 3.4
2 would contain µ .
(ii) 4.45 − 3.4 = 1.05
s (iii) Every member of the population has an equal
∴   1.96 × = 1.05
45 chance of being selected in the survey.
s = 3.59... 27
9 pˆ = = 0.5625
s = 12.9 (3  s.f.)
2
48
98% CI:
(iii) Since width ∝ 1
n 0.5625(1  −   0.5625)
0.5625 ± 2.326 ×

Half width ⇒ 4 × sample size 48
0.5625 ± 0.1665
= 4 × 45
= 180 [0.396, 0.729]


or

3.59 1.05
Stretch and challenge
1.96 × 
n 2 1 (i) X : no. intervals that contain µ
13.40  n X ~ B(10, 0.95)

179.63...  n E( X ) = 10 × 0.95 = 9.5
n  180 Var( X ) = 10 × 0.95 × 0.05 = 0.475
1  (1295)2  SD( X ) = 0.457 = 0.689 (3  s.f.)
6 (i) s2 = 33544   −  
50 − 1  50 
= 0.07143 kg (ii) P( X = 10) = 0.9510 = 0.599 (3  s.f.)

(ii) 
95% CI: (iii) c 10  0.95
1295 0.07143 c  10 0.95
± 1.96 ×
50 50 c  0.995
25.9 ± 0.0741 ∴  99.5%
[25.83,  25.9
97]
2 (i) E( X1 − X 2 ) = µ1 − µ2
(iii) Since 25 is not in the CI, we can conclude that σ 12 σ 22
Var( X1 − X 2 ) = +
there is evidence that the machine is overfilling n1 n2

the bags.  σ 2 σ 2
∴ X1 − X 2 ~ N  µ1 − µ2 , 1 + 2 
 n1 n2 
79
(ii) x N − x T = 72.5 − 64.3 = 8.2  1.62 
S2 H0:  x N − x T = 0 H1:  x N − x T > 0
X ~ N  9,
 5 
12.8
2
12.8
2  
Var( x N − x T ) = + = 11.6736 10.46 − 9 
32 25 P( X  10.46) = P  Z 
 1.6 
Under H0 ,  5 
x N − x T ~ N (0, 11.6736) = P( Z  2.040)

= 0.021 (3 s.f.)

XN – XT
Since 0.021 , 0.025, reject H0. There is evidence that
the mean time has changed.
S2 Statistics 2 Answers

0 8.2 52 800
2 (i) x= = 352 ml
150
 8.2 − 0 
P( x N − x T  8.2) = P  Z   1  (52 800 ) 
2
 11.6736  s2 = 18586776   −  
= P( Z  2.40) 150 − 1  150 

= 0.0082 = 7.89 mll


7.89 
Since 0.0082 , 0.01, reject H0, conclude (ii) X ≈ N  352,
 40 
that there is evidence that there has been
improvement in the test scores.

2 2
(iii) 2 × 1.96 × 12.8 + 12.8  3
n n
327.68 3 351 352

n 2 × 1.96
327.68 × 2 × 1.96  
 n  351 − 352 
3 P( X < 351) = P  Z < 
 7.89 
23.65...  n
 40 
559.47  n
= P( Z < −2.251)
n  560
= 0.0122

(iv) E(P1 − P2 ) = pˆ 1 − pˆ 2
(iii) Yes, as the distribution of volumes is not given.
pˆ 1(1 − pˆ 1) pˆ 2 (1 − pˆ 2 )
Var(P1 − P2 ) = +
n1 n2 3 (i) X : time to complete exam
 pˆ (1 − pˆ 1) pˆ 2 (1 − pˆ 2 )  X ~ N(73.4,  3.22 )
P1 − P2 ≈ N  pˆ 1 − pˆ 2 , 1 + 
 n1 n2 H0:  µ = 73.4, H1:  µ > 73.4

Exam focus
1 X : time taken by rats to complete a maze

73.4 74.2

 3.22 
X ~ N  73.4,
 n 

9 10.46 74.2 − 73.4
Z= = 1.875
X ~ N (9, 1.62 ) 3.2
n
1.875 × 3.2
H0: µ = 9, H1: µ ≠ 9 0.8 =
n
n = 56.25
From the sample
i..e.  n = 56  or  57
x = 10.46 min
We are assuming the standard deviation for
this group is the same as the population value
of 3.2 min.

80
(ii) Z crit = 1.751 P( X  9800)

Since 1.875 . 1.751, reject H0 and conclude  


S2
9050 − 8500 
= P Z 
there is evidence that teaching method is  2100 
 80 
causing the students to take longer.
= P( Z  5.537)
4 X : length of butterfly =0
X ~ N( µ , σ )
2
Since 0 , 0.02, reject H0, there is evidence that
774 the mean fees have almost certainly increased.
(i) x= = 6.45 cm
120
Assume that the standard deviation from the
1  774 
2

S2 Statistics 2 Answers
s =
2
 5087  −   = 0.796 cm2 population can be used for the sample.
120 − 1  120 

(ii) Yes. The original population distribution
(ii) 98% CI is was unknown.
0.796
6.45 ± 2.326 × (iii) P (Type I error) = 0.02
120
6.45 ± 0.189
(iv) P (Type II error)
[6.26, 6.64] (3 s.f.)

(iii) 0.023 = 0.000 008

5 (i) H0: µ = 8500, H1: µ > 8500


8982.3 9500
 21002 
Under H0 ,  X ≈ N  8500,  k − 8500
 80  = P( X  8982.3    µ = 9500) = 2.054
2100
  80
8982.3 − 9500 
= P Z  k = 8982.3
 2100 
 80 
= P( Z  −2.205)
= 0.0137 (3 s.f.)
8500 9800

81
S2 S2 Past examination Chapter 9 The Poisson distribution

questions 1 X + Y ~ Po(5.6)


P( X + Y  3) = 1 − P( X = 0,1, 2, 3)
 5.6
2
5.6 
3
= 1 − e −5.6  1 + 5.6 + +
Chapter 8 Hypothesis testing using  2! 3! 

the binomial distribution = 1 − 0.191


= 0.809 (3  s.f.)
1 (i) X : number of heads in 10 tosses
X ~ B (10, 0.5) 2 X : number of emails in 1 hour
S2 Past examination questions

H0:  p = 0.5 , H1: p  ≠ 0.5


X ~ Po(1.27)
P( X = 0) = P( X = 10) = 0.510 = 0.000 976

( )
P( X = 1)  = P( X = 9)  = 10
1
0.51 0.59 0.00976
(i) λ = 5 × 1.27 = 6.35

P( X = 0) + P( X = 1) + P( X = 9) + P( X = 10) Y : number of emails in 5 hours


= 0.021 47  10% P(Y  1) = 1 − P(Y = 0   or  1)

P( X = 2) = P( X = 8) = 10
2 ( )
0.520.58 = 0.0439 = 1 − e −6.35 (1 + 6.35) 

P( X = 0,1, 2, 8, 9,10) = 0.1093  10% = 0.987 (3  s.f.)


(ii) T : number of emails in 700 hours
∴ Acceptance region is 2  x  8.
T ~ Po(889)

(ii) P (Type II error) T ≈ N (889, 889)

= P(2  X  8     p = 0.7)

= 1 − P( X = 0,1, 9,10)    p = 0.7 

= 1− 
( )
 0 1 ( )
 10 0.700.310 + 10 0.710.39 

( )
10 10
( ) 0
 + 9 0.7 0.3 + 10 0.7 0.3 
9 1 10 850 889
 850.5 − 889 
= 1 − 0.149 P(T  850) = P  Z 
 
= 0.851 889 
= P( Z  −1.291)
2 H0:  p = 0.5, H1:  p  0.5
= 0.902  (3  s.f.)
H ~ B (12, 0.5)
P( X  9) = P( X = 10,11,12) 3 X : number of people who do not arrive

( )
= 12
10
0.5100.52 + 12
11 ( ) ( )
0.5110.51 + 1122 0.512 X ~ B  213, 
 50 
1

= 0.0193
(i) X ≈ Po( 4.26) since np < 5
∴ Significance level is 1.93%
P(overbooked) = P( X = 0,1, 2)

3 (i) X : number of packets that contain a gift  4.26 
2
= e −4.26  1 + 4.26 +
 2! 
X ~ B (20, 0.25)
= 0.202 (3  s.f.)
H0:  p = 0.25 , H1: p
   0.25

( )
P( X  1) = 200 0.2500.7520 + 20
1 ( )
0.2510.7519
(ii) 
Y : number of people who don’t arrive on other
flight
= 0.0243
Y ~ B  135, 
1
P( X  2) = 0.0243 + 20
2 ( )
0.2520.7518

 75 

= 0.0913  5% Y ≈ Po(1.8)
∴ Criticcal region is X  1 X + Y ≈ Po(6.06)
e −6.06 6.065
(ii) P (Type I error) = 0.0243 P( X + Y = 5) =
5!
= 0.159  (3  s.f.)
(iii) 2 is outside the rejection region, so accept H0.
There is no evidence that the proportion of
packets with free gifts is less than 25%.

82
 2 
(iii) λ = 0.8n
4 X ~ B  6000,
 10 000 
L: number of lions seen in n days
S2
2
E( X ) = np = 6000 × = 1.2 P(L = 0)  0.1
10 000
Since np  5 use Poisson e −0.8 n  0.1
− 0.8n  ln 0.1
X ≈ Po(1.2)
ln 0.1
P( X  2) = 1 − P( X = 0,1, 2) n
−0.8
 1.2 
2
n  2.878...
= 1 − e −1.2  1 + 1.2 +
 2 
Smallest value of n is 3.
= 0.121 (3  s.f.)

S2 Past examination questions


5 λ = 2.1 / month, λ = 6.3 (3 months) 7 (i) R = X1 + X 2 + X 3
E(R ) = E(X1 + X 2 + X 3 ) = 3 × 1.6 = 4.8
(i) P( X  1) = e −6.3 (1 + 6.3) = 0.0134 P(R  4) = P(R = 0, 1, 2, 3)
 6.3 
2

P( X  2) = e −6.3
 1 + 6.3 + 2  = 0.0498 (. 0.02) 4.82 4.83 
= e −4.8  1 + 4.8 + +
 2! 3! 
Critical region is X  1 = 0.294  (3  s.f.)
(ii) S = X1 + X 2 + ... + X n
(ii) P (Type I error) = P( X  1) = 0.0134
E( S ) = 1.6n
(iii) H0: λ = 6.3, H1: λ  6.3
e −1.6n (1.6n)4 16e −1.6n (1.6n)2
= •
3 is outside the critical region, so accept H0 . 4! 3 2!
There is no evidence that the mean has 16 × 4 !
(1.6n)2 =
3 × 2!
decreased.
2.56n2 = 64
(iv) λ = 25.2 (12 months) n2 = 25
n=5
Since λ . 25 use normal approximation.
(iii) T = X1 + X 2 + ... + X 40

E(T ) = 40 × 1.6 = 64
T ~ Po(64)
Since λ . 15 use normal approximation.
20 25.2
T ≈ N(64, 64)
 19.5 − 25.2 
P( X  20) = P  Z    75.5 − 64 
 25.2  P(T  75) = P  Z  
 64 
= P( Z  −1.135)
= P(Z  1.438)
= 0.128  (3  s.f.)
= 0.0753 (3 s.f.)
6 X : number of lions seen per day standard X ~ Po(0.8)

Y : number of lions seen per day off-road Y ~ Po(2.7)

(i) P( X  2) = 1 − P( X = 0, 1) 64 75
= 1 − e −0.8 (1 + 0.8)
= 0.191 (3  s.f.)
(ii) λ = 3 × 0.8 + 2 × 2.7 = 7.8

P(total , 5) = P(0, 1, 2, 3, 4)

 7.82 7.83 7.8 4 


= e −7.8  1 + 7.8 + + +
 2! 3! 4 ! 
= 0.112 (3  s.f.)

83
Chapter 10 Continuous random    P(12 < X < 30 ln 1.5)
S2 variables 30 ln1.5


30
   =
∫ 2 dx
28
1 (i) k dx = 1
12 x
20 x2 30 ln1.5
= − 
30
k∫
28
x −2dx = 1  x 12
20

− − 
28 30 30
 −1  =−
k x  = 1 30 ln 1.5  12 
 −1 20
= 0.0337 (3 s.f.)
28
 
k − 1  = 1 3 (i) (a) 
X
 x 20
S2 Past examination questions

  
k − 1 − − 1  = 1 (b)  V – there are high and low values likely
 28  20 
 
k 1 =1 (ii) A=
2+1
× 0.5
 70  2
k = 70 = 0.75
28


70 y
(ii) E( X ) = x× 2 dx
20 x
2
28


1
= 70 dx
20 x
1
= [ 70 ln x ]20
28

= 70(ln 28 − ln 20) 0
= 23.6 (3  s.f.) 0 1 2 x

(iii) P( X  E( X ) = P( X  23.6) or
23.6

∫ ∫
70 0.5
= 2 dx A= (2 − 2x )dx
20 x 0
23.6 0.5
= 70  − 
1 = 2x − x 2 
 x 20 0

= 2 × 0.5 − 0.52
 1 −1 
= 70  − −  = 0.75
 23.6  20  
= 0.534  (3 s..f.)

1
(iii) (a)  ax n dx = 1
0
(iv) Mean is greater than the median, since 1
 x n+1 
P ( X  E ( X ))  0.5 a  =1
 n + 10

a
1 
2 (a) g: Area . 1 =1
 n + 1
h: pdf can’t be negative a = n +1


1


15
(b) (i) E( X ) = x × 302 dx (b) E( X ) = x × ax n dx
0
10 x
= ∫
1

∫ ax n+1dx
15
1
= 30 dx 0
10 x
1
 x n+2 
= [30 ln x ]10
15
= a 
 n + 2 0
= 30 (ln15 − ln10)
a
 15  =
= 30 ln   n+2
 10 
= 30 ln1.5     a
=
5
n+2 6
6a = 5(n + 2)

m
30
(ii)  dx = 0.5
10 x2 6a = 5n + 10 and a = n + 1
m
 30  ⇒ 6(n + 1) = 5n + 10
− x  = 0.5
10 6n + 6 = 5n + 10
30  30  n=4
− − −  = 0.5
m  10  a=5
30
− + 3 = 0.5
m
30
− = −2.5
m
m = 12

84

Chapter 11 Linear combinations of
4 (i) ∫ 3

0
k sin x dx = 1
random variables S2
2
1 B: weight of a bottle B ~ N(1.3, 0.062)
k [ − cos x ]03 = 1
π

k  − cos π − ( − cos 0 )  = 1
2 R: weight of a rack B ~ N(2, 0.32)
 3 
k  − ( −0.5) + 1 = 1 (i) T = R + B1 + B2 + ...B12
1.5k = 1 E(T ) = 2 + 12 × 1.3 = 17.6 kg
2 Var(T ) = 0.32 + 12 × 0.062 = 0.1332 kg2
k=
3 T ~ N (17.6, 0.1332)


m
2
(ii) sin x dx = 0.5
3

S2 Past examination questions


0
m
 − 2 cos x  = 0.5
 3 0
17 17.6 18
− cos m −  − cos 0 = 0.5
2 2
3  3 
P(17 < T < 18)
2 2
− cos m + = 0.5
3 3  17 − 17.6 18 − 17.6 
= P Z 
2 1  0.1332 0.1332 
− cos m = −
3 6
= P ( −1.644  Z  1.096)
1
cos m = = (0.9499 − 0.5) + (0.8635 − 0.5)
4
m = 1.32 (3  s..f.) = 0.813
2π (ii) W1 − W2 ~ N (0, 0.0072)

3 2
(iii) E( X ) = x × sin x dx
0 3 P(W1 − W2  0.05) + P(W1 − W2  −0.05)
2
π  0.05 − 0 

2 3
= 2× P Z 
= x sin x dx  
3 0 0.0072 
= 2 × P( Z  0.589)
2 


3
=  − x cos x − − cos x dx  = 0.556
3 0 

2
2 π
=  − x cos x + sin x 03
3
2  2 
− π × cos π + sin π  − 0 + sin 0  ( )
2 2
=
3  3 3 3  
–0.5 0 0.5
2 2 1 3
= − π×− +
3  3 2 2  2 E(3 X − Y ) = 3 × 6.5 − 7.4 = 12.1
2 π 3
=  + Var(3 X − Y ) = 32 × 14 + 15 = 141
3 3 2 
3 X − Y ~ N (12.1,141)
2π 3
= + = 1.28  (3  s.f.)  20 − 12.1
9 3 P(3 X − Y  20) = P  Z  
 141 
5 y = P( Z  0.665)
1
  = 0.747 (3 s.f.)

1 m 2x

1 1 1
m× m =
2 2 2
1 2 1 12.1 20
m =
4 2
m2 = 2 3 (i) M : number of min for which bike is hired
m = 2 = 1.41 C: cost of hire
C = 500 + 3M
OR
m E(C) = 500 + 3 × 142 = 926c or $9.26

 1 x dx = 1
0 2  2 Var(C ) = 32 × 352 = 11 025
2 m sd(C ) = 11 025 = 105c orr $1.05
x  1
 4 =2
 0
2
m 1
=
4 2
m2 = 2
m = 2 = 1.41 85
(ii) T : total amount paid by 6 people 332 + 334 + ... + 333
S2 E(T ) = 6 × 926 = 5556c
3 (i) x=
8
= 331.125 ml

Var(T ) = 6 × 11 025 = 66 150 = 331 ml (3 s.f.)



   sd(T ) = 66150 = 257c  2
s 2 = 1 877 179  −   (2649) 
                        = $2.57 ((3  s.f.) 8 − 1 8 

= 4.125 or 4.13 (3 s.f.)


Chapter 12 Sampling
1 Children will be excluded. (ii) 98% CI:

4.20
S2 Past examination questions

People without a phone are excluded. 331 ± 2.326 ×


50
People with unlisted numbers are excluded. 331 ± 0.674

More than one person may live in the house. [330, 332] or [330.3, 331.7]

2 Doubling the first digit will always give an even (iii) 333 is not contained in the CI from (ii).
number so odd numbers are excluded.
So claim is not supported.
3 (i) Larger properties are more likely to be picked.
4 (i) H0:  µ = 750 , H1 :  µ  750
Some areas (e.g. edges) are less likely to be picked.
X: weights of packets
(ii) Make a list of the houses and number from 1.
X ~ N ( µ , 112 ) 740 750
Use random number table to select houses.  11 
2
Under H0 , X ~ N  750,
 20 

Chapter 13 Hypothesis testing and  


746 − 750 
confidence intervals using the normal P( X  746) = P  Z 
 11 
distribution  20 
= P( Z  −1.626)
1 (i) 
S: length of salmon
= 0.0520
S ~ N (65, σ 2 )
 4.9 
2
Since 0.0520 . 0.04, accept H0.
S ~ N  65,
 n 
64.3 65 There is not enough evidence to say the
64.3 − 65
Z= = −1.807 mean is less.
4.9
n
0.97 (ii) P ( X  745)  0.97
8.8543
             −0.7 = −
n 745 − 750
= −1.882
11
        −0.7 n = −8.8543
n
                 n = 160 745 750
−20.702
−5 =
(ii) H0:  µ = 65, H1: µ  < 65 n
5% n = 4.1404
Z1 = −1.645
n = 17.14...
Since −1.807 < −1.645 Z1 65 ∴ Smallest n is 18.

reject H0 , conclude that there has been a 5 X : number of questions correct
decrease in the length of the salmon. X ~ B (100, 0.2)

H0:  p = 0.2 , H1:  p  0.2
2 (a) 95% CI:
32.6 UnderH0 , X ~ B (100, 0.2)
41.2 ± 1.96 ×
50 X ≈ N (20,16)
41.2 ± 1.583  26.5 − 20 
P( X  27) = P  Z  
[39.6, 42.8]  16 
1 1 = P( Z  1.625)
(b) 2 × = = 0.125 = 12.5%
16 8
= 0.0521
∴ α = 100 − 12.5 = 87.5%
Since 0.0521 . 0.05, accept H0.
1 1
16 16
Claim is not justified at the 5% level.

86
6 (i) X : time to complete task 8 X : number of heads in 100 throws
X ~ N ( µ ,  3.52 )
H0:  p = 0.5 , H1:  p  0.5
S2
H0:  µ = 22 , H1:  µ > 22 5%
 3.5 
2
Under H0 , X ~ B (100, 0.5)
Under H0 , X ~ N  22,
 12  X ≈ N(50, 25) 50 x
x − 22
= 1.645 x − 50
3.5 = 1.645
12 25
5%
3.5 x = 58.225
x = 1.645 × + 22
12
x = 23.7 22 ? If x = 58, with continuity correction

S2 Past examination questions


P( X  57.5)  0.05
(ii) P (Type II error)
= P( X  23.7    µ = 25.8) If, x = 59, P( X  58.5)  0.05
 
 Rejection region is X  59
23.7 − 25.8 
= P Z   23.7 25.8
3.5 38

 
9 (i) pˆ = = 0.19
12 200
= P( Z  −2.116) using unrounded figures forr X. 99% CI:

= 0.0172 0.19 ± 2.576


0.19(1 − 0.19)
200
7 (i) X : lengths of insects 0.19 ± 0.071 46
x = 7520 = 50.1 [0.119,  0.261]
150
1  ( 7520 ) 
2
s2 =  413 540   −    0.19(1 − 0.19)
150 − 1  150  (ii) 2× z = 0.1
200
= 245 z × 0.0277 = 0.05
OR z = 1.802
 413 540 
s 2 = 150    −  50.12  x%
149  150 
= 245

X ≈ N  50.1,
245 
(ii) –1.802 1.802
 80 
  P( Z  1.802) = 0.0357
 53 − 50.1
P( X  53) = P  Z   P( Z  −1.802) = 0.0357
 245 
 x = 1 − 2 × 0.0357
80 
= P( Z  1.657) = 0.928
= 0.0488 = 92.8%

50.1 53

87

You might also like